Libro de Algebra Preuniversitaria Nivel Uni

November 9, 2017 | Author: carlos40083 | Category: Exponentiation, Algebra, Division (Mathematics), Rational Number, Multiplication
Share Embed Donate


Short Description

Download Libro de Algebra Preuniversitaria Nivel Uni...

Description

Álgebra INTRODUCCI ÓN La palabra Álgebra viene de "ilm al-jabr w'al muqabala" título árabe del libro escrito en el siglo IX por el matemático árabe Muhammad ibn Musa Al-Khwarizmi. Este título se traduce como "Ciencia de la restauración y la reducción". El álgebra es una rama de las Matemáticas que estudia la forma de resolver las ecuaciones. Por ello, todas las operaciones algebraicas, reglas, fórmulas, definiciones, etc. tienen un sólo objetivo: el cálcul o de incógnitas. Una de las características es que utiliza símbolos o letras para representar números. Por ejemplo la letra "x", puede representar el valor de una temperatura, una edad, una velocidad o la medida de un ángulo; pero el Álgebra no estudia estas magnitudes, nos muestra las operaciones en general sin precisar qué tipo de magnitud se está tratando. El Álgebra actual trata con estructuras más complejas que los números y sobre estas estructuras define operaciones similares a las operaciones aritméticas. Esta nueva Álgebra se debe a Evariste Galois. CONCEPTOS BÁSICOS EXPRESIÓN ALGEBRAICA Es un conjunto de números y letras relacionados entre sí por los operadores matemáticos de la adición, sustracción, multiplicación, división, potenciación y/o radicación, en un número limitado de veces, por ejemplo : P(x;y;z) = 5 x 2

F(x; y) H(x; y; z)

2x

3x 3 y

1 y

24 z

2yz ; llamada racional entera o polinomio..

7 ; llamada racional fraccionaria. 5x ; llamada irracional. y

(*) Magnitud : Todo aquello susceptible a ser medido. TÉRMINO ALGEBRAICO Es aquella expresión algebraica que no presenta operaciones de adición ni sustracción. ELEMENTOS DEL TÉRMINO ALGEBRAICO signo exponentes

- 7 x5 y 8

P(x;y) =

coeficiente parte literal

Parte Literal : Está formada por las letras con sus respectivos exponentes que representan ciertas magnitudes, como por ejemplo: P(x;y;z) = 6 x 4 y 3 z ; la parte literal es : x 4 y 3 z Coeficiente Numérico : Es el número que generalmente se coloca delante de la parte literal, cuando el coeficiente es entero positivo indica el número de veces que se repite como sumando la parte literal, así pues tenemos : y3

y3

y3

......

80 veces

y3

80y 3

También se puede tener un coeficiente literal , como por ejemplo : P(x) = ax 2

el coeficiente es "a".

TÉRMINOS SEMEJANTES Son aquellos que presentan la misma parte literal, como por ejemplo : 2y 3 z ;

3 3 y z; 5

7 y3 z

REDUCCIÓN DE TÉRMINOS ALGEBRAICOS Las operaciones de adición o sustracción entre términos algebraicos sólo se puede efectuar entre aquellos términos que sean semejantes, para lo cual se calcula la suma o resta de los coeficientes numéricos, permaneciendo invariable la parte literal, veamos algunos ejemplos : Ejemplo : *

9y 3z 6y 3z 15 y3z

*

2x 4 y 3

4 z 5x 4 y 3 10z

7x 4 y 3

6z

Capítulo

LEYES DEEXPONENTES ECUACIONES EXPONENCIALES

1

POTENCIACIÓN

TEOR EMAS

Es la operación matemática que tiene por objetivo encontrar una expresión llamada potencia (p), conociendo previamente otras dos expresiones denominadas base (b) y exponente (n).

1.

bn

b p ; donde n

base ; b R exp o nente ; n Z

p

Multiplicación : bases iguales. a m . an 4 2 Ejemplo : x . x

2.

x4

2

n

x6

División : bases iguales. am

potencia ; p R

a Así pues, en 23 = 8 : 2 es la base, 3 es el exponente y 8 es la potencia.

am

Ejemplo :

x10

x10

x7

am

n

7

n

;a=0

x3

DEFINICIONES 1.

3.

Exponente cero

(a m )n ao

2.

1; a=0

1 ; ( 3)o

Ejemplo : 5 o

1;

Ejemplo : (x 2 )5

7

o

1

4.

Ejemplo : 4

a m.n x 10

Multiplicación M ult p ultip : exponentes iguales. a n bn = (ab)n

Ejemplo :

a3 b3 c3 (abc)3

1

4

(x 2. y 3 )5

(x 2 )5 . (y 3 )5

x10 . y15

Exponente entero positivo 5. an = a.a.a. ...... . a ; n

División : exponentes iguales.

2

an

"n" veces

Ejemplo : 7 3 4.

x 2.5

Exponente uno a1 = a

3.

Potencia de potencia.

7 . 7 .7

bn

343

Ejemplo :

Exponente negativo.

a

Ejemplo : 2 1

1 21

n

x3 1 a

n

a b

y3

;a=0

1 ; 3 2

x4 2

1 32

x y

1 9

y3

2

3

(x 4 )2

x8

(y 3 )2

y6

n

;b=0

RAD ICAC IÓN

TEOREMAS :

Es una de las operaciones matemáticas inversas a la potenciación cuyo objetivo es encontrar una expresión llamada raíz (b), conociendo otras dos expresiones denominadas radicando (a) e índice (n).

1.

Multiplicación : índices iguales. n 3

Ejemplo : signo radical n

a

b ; donde

n

Índice ; n Z

a

Radicando

b

Raíz ; b R

2.

a

b

a

b

n

a

b

9 3

3

32

9

8

2

( 2)3

8

Observación : Debemos tener en cuenta que dentro del conjunto de los números reales no se define a la radicación cuando el índice es par y el radicando negativo, como en los ejemplos : 4

32 no existe en R.

n

am

m an

x y

y

Raíz de raíz.

a R

3

8

2

3

Ejemplo :

n

1.

a b

2.

amb

m

3.

m

mk

a)

an

x

3 2

m.n

a 6

x

a

x

b a

n

; ab

0

am b ; a > 0 a nk ; k Z

2

( 2)

Formando parte de algún exponente Ej. 5x

b)

Ejemplo :

3.

b

a ;b=0 b

Es aquella ecuación donde al menos uno de sus miembros no es una expresión algebraica, así pues tenemos:

Exponente fraccionario.

(

n

INTRODUCCIÓN UC A LAS ECUACIONES TRASCENDENTES DE EN

2004 existe en R.

2 8) 3

a

PROPIEDADES ADICIONALES

bn

a

a. b

xy

m n

Ejemplos :

2.

x

a, b R , n Z n

n

División : índices iguales.

Ejemplo :

3.

n

3

n

3

1.

x .3 y

n

Así pues : en 64 4 : 3 es el índice, 64 el radicando y 4 la raíz. DEFINICIONES :

n

a. b

1

x

125 ; 23

Como base y exponente a la vez Ej. 2x x 5 ; xx

4

c)

16

3

Afectada por algún operador Ej. Logx2 x 1 ; Cos(2x) 0,5

n Z

ECUACIÓN EXPONENCIAL : n

*

a

a ;n

n

|a| ; n

# impar # par

|a| : valor absoluto de "a", significa el valor positivo de "a".

Ejemplo :

3

x3

x ;

x2

|x |

Es la ecuación trascendente que presenta a su incógnita formando parte de algún exponente. Ejemplo : 5 x

2 1

25

Teorema :

Transformando al segundo miembro se tendrá : a

Ejemplo : 7x

1

x

a

y

5 x

x

y ; a > 0; a = 1

7 x 1 2x = 6 x=3

5 x

Obs e r va ci ón : Para resolver algunas ecuaciones trascendentes, a veces es necesario recurrir al proceso de comparación comúnmente llamado método de analogía, el cual consiste en dar forma a una parte de la igualdad tomando como modelo la otra. Veamos un ejemplo : Ejemplo :

xx

3

3

x

x

3

33 3

x

3

3

3

3 (representa un valor de "x").

Sin embargo, debemos indicar que el método de analogía sólo nos brinda una solución, pudiendo haber otras, sino veamos el siguiente ejemplo : En :

x

x

2 se observa que x = 2

Pero

2 =

4

x

4

x

4 , con lo cual tenemos :

4 de donde : x = 4.

EJERCICIOS PROPUESTOS 01. Calcular : A + B; sabiendo que : 1 ( ) 2

( 2 3 )0

A

1 ( ) 3

B

2

1 ( ) 2

a) 5 d) 20

2

6 5

06. Si el exponente de "x" en : 1

0

216

a

a2

1 4 2

a

xb

3

x b es 4, entonces el exponente de "x" en :

(x a 1)2b .

4 d) 16

b) 10 e) 25

c) 15

b) 2 e) 1

07. Sabiendo que :

a

Reducir : n

02. Reducir : 32

24 x

x 1

.3

8 3x

a

c) 8

n

0.

1

.

33

32 x

(3 )

a) a0

4 b) a

d) a2

e) a

c) a

1

08. Simplificar : c) 3 37

b) 318

a) 1 12

d) 3

33 33 3 3

e) 3 24

03. Reducir :

33

3

3

3 3n 3

"n" radicales 1

a) 48 d) 64

4

1 16

U

32

5

a) 3 d)

9

b) 9 e)

3

3

c) 27 3

09. Hallar el valor de " " , si el exponente final de "x" en :

b) 50 e) 32

c) 16

3

x

x

5

x

es la unidad. Además : 3

04. Simplificar : b

a) 2 d) 8

6 a. 16 b. 3a 2b

a) 10 d) 25

18 a b

b) 4 e) 12

c) 6

5

b) 15 e) 30

c) 20

10. Hallar el exponente final de : x x x ...... x x

05. Sabiendo que :

f(x)

3

3

x

x

100 radicales

x2 / 3

a) Calcular : M

.......

f

f(x) (x ) , para : x = 3. d)

a) 3

1/ 2

b) 3

d) 3

1/ 3

e) 31 / 2

c) 3 1

399 390 1 2100 1 100

2

1

b)

e)

299 299 1 3100 100

3

1

c)

2100 1 2100

11. Hallar "x" :

19. Resolver : x

4 .8 a) 1/3 d) 5/3

x 1

2

2x 1

.16

b) 2/3 e) 4/3

12. Al resolver : 163

2x

3x 2

x

84

2x

p . q

a)

5

5

d)

5

5

2

b) 3 e) 6

3

5 5

x

b) 1 e) 4

2

3

a) 2

b) 3

d) 0,3

e) 6

15. Calcular "x", si : a) -3 d)

240

1( ) b) ( ) 7 7

1 d) ( )7 7

e)

7

7

4 5

0

e)

16. Resolver : x x a) 12 d) 9

1 3

9

8

3

5 c) -1

1

1

9

3

1

1

9

3

3

c) 2

1 4

a) 9

b) 15 e) 18

x . 4

x

d) 27

1 3 e) 3 b)

5

4x 3

1 . 3

9

1

b) 3

2

d) 3

6

e) 3

9

c)

5

c) 3

3

1 9

23. Reducir :

c) 10

a) 3

45 x

.52

y 6 2

a) 1

b) 33

d) 4

e) 324

5

23 y

c) 318

24. Calcular :

18. Resolver : x 13

a) 25 d) 50

3

b) 1 e) 2

x

672 ; e indicar : E

17. Hallar "x", de : x x

2

22. Reducir :

b) 4

1 2

1 7

b) 20 e) 1

x

x

x 37

x

13

xx

4

77

a) 7

a) 0 d) -6

c) 0,5

2

3

c)

5

1

x

( 11)0

32x

5

21. Calcular :

c) 2

2

c)

e) 5

14. Resolver :

9x

25

1

13. Resolver :

a) 0 d) 3

5

7

c) 4

x 2 3x

5

b)

20. Resolver : x 7

Indique : p + q.

4

x

c) 4/5

se obtiene la fracción irreductible :

a) 2 d) 5

2. x

1 x

10 n

5n

82

3 1

n 1

c) 13 a) 2 d) 4

b) 8 e) 16

c) 64

1

25. Sabiendo que :

a) 6 d) 8

5x

5x

Calcular : N

P(5)

a) 5 1 / 5 d)

5

P(5)

x5

x

34 x. 96 a) 4 d) 7

. c) 51 / 3

b) 51 / 5

5

. 2710

x

814

32. Resolver :

a) 2

2x

274

2x

b) 4

c)

a

x b 1. x c es 5, entonces el exponente de "x" en : ab c

a) 1 d) 4

(x

d)

5a 1 a

)

b) 2 e) 5

1 4

e) 8

4x

27. Reducir : a

a) an

b)

d) a n

n2

e) a n

1

a) 0 d) 3

n n 1

a c)

a

n

1 2

33. Resolver :

c) 3

n

x

c) 6

813

26. Si el exponente de "x" en : a

x

b) 5 e) 8

3

e) 5

c) 21

31. Resolver :

3

P(x)

b) 3 e) 10

7 7

2 2x

x

b) 1 e) 4

c) 2

34. Resolver : a

4x

1

48 22x

3

n

a) 1 d) 4

28. Simplificar :

b) 2 e) 5

c) 3

35. Calcular "x", si : 55 55 5 5 5 5

..........

55

5

5

55n 5

6

5

3

x

5

"n" radicales

a) 5

b) 10

d) 5 5

e)

29. Si : a a a aa

c) 25

a) 1

b)

1 2

d) 3

e)

1 4

5

a 1 , entonces el equivalente reducido de :

(a 1)(a

1)

36. Hallar "x" : (2 x )2

es :

a) 1

b) a

d) a2

e)

a

c) 1/4 a

a) 4 d) 2

232 . b) 8 e) 32

6 3

x2

3

x2

3

x 2 .......

3

x2

3

n

y x

n . Calcular : (n+x). 2

515 5

xk

El miembro de la izquierda consta de "n" radicales. Si :

80

c)16

37. Hallar "x" en :

30. En la siguiente ecuación :

k

c) 2

a) 9 d) 6

b) 12 e) 10

x 1

5x 54

5 c) 92

38. Hallar "x" de :

44. Reducir :

a) 5

1

d) 5

4

b) 5

2

e) 5

5

1

625

xx

c) 5

3

n

6.3 x

x

a)

7

b)

8

d)

13

e)

15

64 c)

e) x

11

M a) x 2 d)

39

x

3

m

nx

px

x c) x

mnp

45. Efectuar :

40. Resolver : xx

p

mx b) 1

d) mnp

3 2 3x x

m p . nm . pn

Sabiendo que :

a) 2

39. Resolver : 3

mm . nn . p p

E

5

3

x

x 1 x1 x

x

1 x .1 x

1

b) x

xx

x

c) x x

e) x

x

46. Calcular : 2 6

1 a) 3 d)

3

b) 2

3

e)

9

c) 9

M

3 a) 2 2 d) 8

41. Simplificar : 2n 1. 4

M

8

16 . (2n )

a) 4,5

b) 3,5

d)

e)

3

2n 1

n 2

3

b)

47. Si : m + n = 2mn ; reducir :

42. Reducir : .

n

x

(0,5)

2

a) 2 d) 2 x

4

1

b)

2

2

e)

23

a) 2

d) 2

2

c) 2

2 1 2 2 2

2

2

a) 2 d)

b) 2 2

e) 2

2 2

4n

2m

m

2n c)

2

3

48. Calcular :

x2

43. Mostrar el equivalente de :

2

4m

b) 1 e) -4

33

12

c) 2

2

2

2

2

e) 4

c) 2,5

2x 3 4x 2

8

8

8

2

2

2

a) 2

b)

d) 8

e)

3

39 33 1 33

2 /2

2

39

c) 1/2

2

49. Hallar el valor de :

22

x 1

E c) 4

1 33

para : x

a) 4 d)

1 4

2

x

8x

.

x 1

x

8x

.

x 1

2

b) 16 e)

1 16

c)

1 2

.....

50. Simplificar :

55. Hallar "x" de :

x 2

7

22

23

7

n

7 ..... 2 7

4

42

7

43

7

7.....

4n

a)

4n

.

3

7

2

2 c) 4 2

b) 2 2

2

d)

2 )2

(x

2

2

e)

2 1

2

7 56. Resolver ecuación :

Señale el exponente de 7.

a)

2 2

4

b) 2n

n

1 d) 3n

e)

c) -

1 2

1 2

x2

3

x2

1 2

3

1 2

x2

Entonces el cociente de las soluciones es :

n

a) -1 d) 2

n 2n 1

b) 0 e) 3

c) 1

57. Calcular "x" en : 51. Hallar "x" en : 27 27 a) 6 d) -8

x 1

39

m

x 2

b) 7 e) -7

c) 8

x 1 3

a) 1/5 d) -2/5

. a

2x 1 4

. a

2 3x

b) 3/5 e) 1

2x 3

9 . 4

9x 4

a)

19 2

b)

d)

1 9

e) 2

1; a

0

c) -4/5

76 3

2 3

19 x

8 27

c)

2

a) n

b)

n

d) nn

e)

n

x

c)

n

n

n

1 ; y además :

a) 1/4 d) 1/2

x

x

x 1

x

x

x

b) 2 e) 1/8

c) 1

59. Hallar "x", en :

27

0

x

8 5

a)

d)

1 4

x 2x

b)

2 4

e)

2

2

2;x

1 2

0 2 2

c)

2

60. Hallar "x" : (x > 0).

2

2y

x y

4, y 2

x

6 , el valor de 2

y

2 es :

1/ 2

x a) -4 d) -2

xx

, siendo : m

Calcular : 2x.

54. Si : 2x

xn

x

53. Resolver : 2 3

x

xx

58. Si : x R / x

52. Indique "x" en : a

xn x

b) 4 e) 0

1 x1 x

x1 / 2

x1 / 2 x

c) 2 a) 2 d) 2

b) 4 5 e) 8

c)

5

4

Claves 01.

b

31.

b

02.

c

32.

c

03.

d

33.

c

04.

d

34.

b

05.

d

35.

c

06.

c

36.

c

07.

c

37.

e

08.

a

38.

e

09.

b

39.

b

10.

c

40.

a

11.

e

41.

a

12.

b

42.

d

13.

d

43.

a

14.

c

44.

b

15.

b

45.

d

16.

b

46.

d

17.

c

47.

d

18.

a

48.

a

19.

a

49.

b

20.

c

50.

c

21.

c

51.

d

22.

d

52.

c

23.

a

53.

b

24.

d

54.

b

25.

a

55.

b

26.

a

56.

a

27.

b

57.

c

28.

a

58.

c

29.

b

59.

c

30.

a

60.

c

Capítulo

2

POLINOMIOS

NOTACIÓN DE EXPRESIONES ALGEBRAICAS

P( 2 7)

2 ( 2)3

5 ( 2) 1

Se utiliza para indicar las variables de una expresión. Ejemplos : *

P(x)

variable : "x".

F(x ; y)

variables : x, y..

Q(x ; y ; z)

ax

by cz

"Q " de xyz

1.

Suma de coeficientes = P(1).

2.

Término independiente = P(0).

CAMBIO DE VARIABLE

"F" de xy

*

var iables cons tan tes

x; y; z a; b; c

Así como las variables pueden reemplazarse por números, también pueden ser reemplazadas por otros polinomios, así tenemos: 1.

VALOR NUMÉRICO (V.N.) Es el resultado que se obtiene al reemplazar las variables de una expresión algebraica por valores determinados.

Dado : P(x) = 2x+11 . Obtener P(x+7) Para obtener lo pedido, se reemplaza :

x por x 7 en P(x). P(x) 2 x

Ejemplo : 1.

x 7

Determinar el V.N. de la siguiente expresión : P (x; y; z) x 2 3yz y = -2; z = 3

para : x = 5; 2.

P(5; -2; 3) = 5 2

3 ( 2 )( 3 )

7

2 (x 7) 11 2x

x 7

25

Dado : P(x 3)

3x

4

Determinar P (3) , si :

Se reemplaza (x + 3) por (2x - 5) previa preparación del polinomio como :

P (x) x3 2x 10 .

P(x+3) = 3(x + 3 - 3)+4

En este caso, se pide el V.N. de P (x) para :

Ahora : P(2x-5) = 3(2x-5-3)+4 Luego : P(2x-5) = 6x - 20

x = 3.

P (3 )

33

2 (3 )

10

P(3) = 23 3.

P(x 7) P(x 7)

11

Determinar : P(2x 5) .

Reemplazando :

2.

27

PROPIEDADES : para un polinomio P(x).

"P " de x

*

P (5)

16 10 1

POLINOMIO

5x 1

E s t o d a e xpr e s ió n al geb ra i ca r a ci o nal y e nte r a . C uand o t i e ne un t é r mi no s e de no mi na monomio, con dos se denomina binomio, con tres trinomio, etc.

Para este caso, se resuelve la ecuación : x + 7 = 5; de donde : x = -2.

Recordemos que en una expresión Algebraica Racional entera :

Al reemplazar :

Ninguna variable está afectada por algún signo radical o exponente fraccionario.

Determinar P(5), si : P(x 7)

2x 3

Ninguna variable se encuentra en el denominador.

POLINOMIOS ESPECIALES

Ejemplo :

1.

P(x ; y)

3x

2

5 polinomio (trinomio).

7y

P(x;y;z) = 2 x

Ejemplo :

2y z no es polinomio..

P(x ; y)

GRADO : Es la categoría que se asigna a un polinomio; y depende de los exponentes de sus variables. GRADOS DE UN MONOMIO : Grado Absoluto : es la suma de los exponentes de sus varia bles.

4

Ejemplo : P(x; y) 2a x y

2x 4 y 3

x5y2

5x 6 y

7

7

7

Homogéneo de grado 7. 2.

Grado Re lativo : es el exponente de la varia ble en referencia. 3

Polinomio Homogéneo : cuando sus términos son de igual grado absoluto.

Pol i no m io Co m pl e to : cuando tiene todos los exponentes de la variable en referencia, desde el mayor hasta el cero incluido. Ejemplo : "x" tiene exponente "1"

5

P(x; y) 2x y

3

"x" tiene exponente cero

2 4

7x y

5y

G. A. = 5 + 4 G.R. (x) = 4

completo con respecto a "x" .

G.R. (y) = 5

Propiedad : para un polinomio completo P(x).

G R AD OS D E UN POL I N OMI O DE D O OS S Ó M MÁS TÉRMINOS : Grado Absoluto : es el mayor grado absoluto de uno de sus monomios.

# términos = Grado + 1 3.

Grado Relativo : es el mayor exponente de la variable en referencia.

Polinomio Ordenado : es aquel cuyos exponentes de la variable en referencia (ordenatriz) van aumentado (orden creciente) o disminuyendo (orden decreciente). Ejemplo : aumenta

Ejemplo : mayor

P(x;y) Grados

3

4 5

2x y 7x y 4

9

6 2

6x y

Q(x)

mx 3

3

7

6x y

9

5 xy

20

ordenado ascendentemente respecto a "y". POLINOMIO IDÉNTICAMENTE NULO Es aquel polinomio cuyos términos presentan coeficientes iguales a cero, como por ejemplo :

POLINOMIOS IDÉNTICOS Dos polinomios son idénticos si sus términos semejantes tienen igual coeficiente, así pues : ax 3

4

4x y

8

G.A. = 9 G.R. (x) = 6 G.R. (y) = 5

P(x )

P(x; y)

mayor

bx nx

P(x)

ax 3

bx 2

c

será idénticamente nulo, si : a = 0; b = 0; c = 0.

c p

son idénticos, si : a = m; b = n ; c = p. Propiedad : dos polinomios idénticos tienen el mismo valor numérico para cada sistema de valores asignados a sus variables.

Propiedad : todo polinomio idénticamente nulo tiene valor numérico igual a cero para cualquier sistema de valores asignados a sus variables.

EJERCICIOS PROPUESTOS 01. Hallar : P[P(3)]. Si : P(x) = 2x - 5. a) 1 d) -1

08. Dado el polinomio :

b) 3 e) 5

Si : P(x ; y)

02. Si se cumple : P(x ) P(x

a a) 8 d) 14

Calcular : P(4) P(0) . b) 10 e) 15

c) 20

P(x) ax b

Q (x )

bx a

R(x )

P(x ; y) 4m x Si : GA(P) = 10

y

P(x )

c) 16

4x5ym

4

3x 4 y m

5

x6ym

2

Si : GR(x) + GR(y) + G.A. = 32. Entoces el valor de "m" es : b) 5 e) 8

c) 6

xa

b

x7 y b

a

x 20 z12

b) 9 e) 1

c) 5

07. Determinar cuál es la suma de los coeficientes "m" y "n", de modo que para cualquier valor de "x", se cumple:

7 x b) 1 e) 2

9

x m)3

c) 624

y (n 3) x n 8 y 2

m (x 1) n (x 2) c) -2

......

es ordenado y completo. Hallar el número de términos. a) 7 d) 5

b)9 c) 13

c) 11

12. Si :

P(x

2)

6x 1

12x 17

Obtener : F(10) . a) 23 d) 21

es homogéneo. Calcular : (a b)2 .

a) -1 d) 0

(n 2) x n

P(F(x) )

06. Si el polinomio :

a) 16 d) 3

b) 243 e) 64

(n 4) x n 7 y 3

05. Dado el polinomio :

R (x ; y ; z )

3)m (mx 5 1)5 (2x m

11. Si :

b) 64 e) 2

3

(2x 4

a) 1024 d) 512

GR(x) = 7.

Calcular su coeficiente.

a) 4 d) 7

c) 17

Indique el coeficiente principal, si el término independiente es 72.

n 2m 3n 5n m

7x2 ym

b) 19 e) 13

c) 1

04. Dado el polinomio :

P(x, y)

c) 20

10. Dado el polinomio : b) a e) ab

a) 5 d) 8

b) 18 e) 28

a) 15 d) 21

Q (P(x ))

Hallar : P(Q(1)) . a) b d) -b

ab

P(x ) (2x 1)n nx con "n" impar, si la suma de sus coeficientes aumentado en el duplo de su término independiente resulta 16, entonces "n" es :

b) . Además :

P(Q( x) )

b

09. Sea el polinomio :

03. Sean los polinomios : siendo : (a

0 . Calcular :

x

1)

para algún polinomio no constante.

a) 9 d) 0

(a 4)xy 2 (20 b) x 2 y ax 2y

P(x ; y )

c) -3

b) 20 e) 19

c) 22

13. Dada la expresión : P(x) , tal que :

P(x )

P(x

P(2)

4 . Calcular : P(P(P(0))).

a) 7 d) 1

1)

P(x

2) ,

además : P(1)

b) 4 e) 14

3;

c) 3

14. Dado el polinomio : a 1

7 a

P(x ) x 3x 5x 7 Hallar la suma de valores que puede asumir "a". a) 6 d) 18

b) 11 e) 21

P(x, y, z) (xy )3a Calcular : a + b + c.

b a

yb

x2

R (x )

5x 2 P(x 1)

3x (x

2z c

ax 2

b) 32 e) 26

5)

b) 243 e) 1

c) 1024

b) 2 e) 4

c) 6

19. Si la siguiente expresión matemática se reduce a un polinomio : a

b

P(x, y, z) a x b b x c Hallar el valor de : a - 2c + b. b) -2 e) 0

c

c xa

c) 1

xa

a 1 4 4 y

x11

a

f(x ) f( y) ; f (1)

c) 4

P(x; y) nx n m y x r 1y my m 5x 3 es homogéneo y con grado relativo respecto a "y" igual a 3. Hallar el grado relativo de "x". a) 3 d) 9

b) 5 e) 11

2

c) 7

25. Sean los polinomios : P(x )

ax 3

R(x )

ax

Si : P(0)

bx 2

cx d ; Q(x )

ax 2

d;

b. R ( 2)

1.

Hallar "x", tal que : R(x)

0.

2 ; Q (1)

a) -3 d) 1

b) -1 e) 3

c) 0

26. Determinar en cuanto difieren "p" y "q" para que con cualquier valor de "x" se cumpla que :

a) 7 d) 3

p (x 4) q (2x 3)

b) 5 e) 2

c) 1

27. Hallar : m . n, si el polinomio es homogéneo.

2 ... 10) .

P(x ; y) a) 220 d) 55

1

b) 8 e) 3

27 8 x

20. Sea "f" una función definida en el conjunto de los números reales tal que verifica las siguientes propiedades : Calcular : f(1

x a 5y 2

24. Si el polinomio :

[F(1) F(2) F(3) ... F(99) ]

y)

c) 30

a

R ( x ; y) a) 7 d) 6

F ( 5)

f(x

c

c) -7

Q(x) x m 10 x m n 5 x p n 6 es completo y ordenado en forma decreciente.

a) -1 d) 2

24 x 2

23. Indique el grado de :

18. Hallar : m - n + p; si se sabe que el polinomio:

a) 8 d) 10

8x 4

b y P(P(x ))

a) 28 d) 31

2) q(x )

Hallar :

a) 0 d) 23 499

c) 87

El valor de : a + b + c, es :

x 3 (x18 125x15 ) 2 (x

K

6x 1

22. Si : P(x)

c) 7

b) 9 e) -6

17. Si : F(x )

3x 2

2)

b) 78 e) 99

Hallar la suma de coeficientes del polinomio R (x) . a) 11 d) 13

2x 4

2)

Hallar : H(5).

16. Si se cumple : P( x )

f(x ) g (x )

1)

a) 62 d) 93

a b

b) 5 e) 15

Donde : f(x g (x

c) 13

15. En el polinomio homogéneo :

a) 3 d) 9

H (x

21. Si :

a 5

b) 20 e) 110

x n 3 y7 (x 2 y 2 )4

x my 4

c) 40 a) 100 d) 140

b) 124 e) 70

c) 144

28. El grado de homogeneidad del polinomio : P(x ; y) x a y 2b c x a b y 2c x a 2c y a es 6. Calcular el valor de : E = a + b + c. a) 9 d) 3

b) 7 e) 11

34. Dado el monomio : 2b

M(x; y) 4 a b x 2a 3 b y 5 b se tiene : GA(M) = 10; GR(x) = 7. Señalar su coeficiente.

a

c) 5 a) 2 d) 16

b) 4 e) 64

c) 8

29. Sea el polinomio : P(2x)

2a1x 2

a0x

22 a 2 x 3

... 25 a5 x 6

Hallar la suma de coeficientes de P(x ) , si su término independiente es a5

2 y además:

a0 a1 a 2 a 3 a 4 a) 3 d) 2

8 ; a0

b) 5 e) 1

0

a (x 1)(x

x2

g (x )

Si : f(x)

g (x ) ;

a) 4 d) 7

b) 5 e) 8

b(3 2x)8

5

c) 6 R.

P(x ) (x n 2) (x n 3)3 2 en el cual el término independiente es 17. Calcular "n".

2)(x

3) c (x 1)(x

3)

9

a) 1 d) 5

b) 4 e) 3

c) 2

37. Hallar : m - n + p; si se sabe que el polinomio:

x R

Q(x) x m 10 x m n 5 x p n 6 es completo y ordenado en forma decreciente.

Determine el valor de : a+b+c. a) -1 d) 2

a(2 x)10

4

2) b (x

2x

P(x) Hallar : a + b.

36. Definimos un polinomio P(x) x

c) 7

30. Dados los polinomios : f(x )

35. Si la suma de coeficientes del polinomio P(x) es 13.

b) 0 e) 1/2

c) 1

a) 8 d) 10

b) 2 e) 4

c) 6

38. Sabiendo que el polinomio : x c 31. Si : f(x ) x 1 f(f(x)) será :

a)

c

x

b)

x 1 d) 1

A (x; y) 7x a 2 y b 3 8 x c y d es homogéneo. Hallar "a".

1.

1; c

x

c) c

x 1 e) x

a) 0 d) -3

h(f(0)) h(

a) 82 d) 28

2) 5)

5 x 2a

3 b 1

y

c) 1

39. Si el polinomio : R(x )

32. Si : f(x

b) 2 e) -4

1

x 2 1 y h(x

1)

3x 1 , se tiene que

es : b) -17 e) -4

c) 193

(a

b 2) x 2 (a c 3) x (b c 5)

se anula para : x = 2001; x = 2002; x = 2003; x = 2004. Hallar : a-b+c. a) -1 d) 0

b) 2 e) 2001

c) 1

33. Hallar "n", si el grado de :

x xn 3

a) 5/3 d) 56/5

b) 56 e) 5/6

x

40. Sea P(x) un polinomio mónico de grado 3; halle la suma de coeficientes del término cuadrático y lineal, siendo su término independiente igual a 5. Además :

es 5

P(x c) 56/3

a) 1 d) 3

1)

b) 0 e) 4

P(x ) nx 2 c) 2

41. Dado un polinomio lineal P(x ) , que presenta resultados mostrados en el cuadro :

x

a) 7 d) 5

b) 9 c) 13

48. Dada la función "f", tal que :

1 2

P(x) 4 6 Calcule : P(5)

b) 16 e) 8

f(x 2 2x 1)

42. Si :

a) x 2

f

P(0) .

a) 18 d)14

x

2

b) x 2

c) x

2 x

2

e) x

2 x

2

d) (x

4

Calcular :

c) 12

3 , entonces f(x

2x 2

2)

2 x

2

2

1

2)

es:

b a b

y

f(1) f(

b) 7 e) 8

3x 2 m

n 4

.y m

n 2

P(x ; y) (m 3)x a) 4 d) 10

b 2y 4

7x 2m

n 2

.y m

n

b) 9 e) 18

mx

b) 6 e) 12

Obtener : P

m 2

.y 3

y17

2m

c) 8

y

c) 14

51. Si : f(x

1 ax 1

b) 2 e) 0

3a 1

c) -3

4 ; y f(0) = 2,

f(x ) 2x

1)

a 2x

1 2

a) 1 d) -2

n 3 m n 1

es de grado 10 y la diferencia entre los grados relativos a "x" e "y" es 4. a) 6 d) 15

9 m

50. Siendo :

c) 3

7 x 2m

c) 10

m

44. Calcular : m - n, si el polinomio :

P(x; y)

1)

P

b) 2 e) Más de 4

x R

2

es homogéneo? a) 1 d) 4

18

49. Proporcionar la suma de coeficientes del siguiente trinomio :

43. ¿Para cuántos valores de "b" el polinomio : ab x a

2x 2

3 x 3 2

a) 11 d) 9

4

P( x ; y )

c) 11

entonces f(1) f(

1)

a) 0 d) -2

b) 2 e) -6

vale : c) 6

45. Si el polinomio P(x;y) es idéntiamente nulo. Hallar : ab. P(x ; y)

(a

a) 10 d) 60

b)x 3 y

2x 4 y 5 18 x 3 y (b a)x 4 y 5

b) 20 e) 80

52. Si : f(x x )

c) 40

xx 1

Además : f(x x Calcular : P

xx

2x 2

3125 .

1)

f (x

2)

.

46. En el polinomio : P(x 1) (2x 1)n (x 2)n 128(2x 3) donde "n" es impar, la suma de coeficientes y el término independiente suman 1, luego el valor de "n" es : a) 5 d) 11

b) 7 e) 13

c) 9

47. Si :

a) 16 d) 14

b) 10 e) 12

c) 18

53. Q(x) es un polinomio que cumple las siguientes condiciones : I. Q(3) = Q(5) = 0 II. Grado mínimo III. Suma de coeficientes 16. Calcular el término independiente de Q(x).

P(x )

(n

2) x

n 9

y (n

3) x

n 8 2

y

(n

4) x

n 7 3

y

...

es ordenado y completo. Hallar el número de términos.

a) 18 d) 45

b) 15 e) 32

c) 30

54. Sabiendo que : P(x; y) (5x 3 y)n 1 5n es tal que la suma de coeficiente es igual al término independiente aumentado en 1024. Hallar "n". a) 6 d) 9

b) 7 e) 10

58. Si : f(x

Hallar : f

x2 1 x

a)

a

b

c

F(x ) xa b xb c xa c es homogéneo de grado (10), de qué grado es el monomio.

a) 7 d) 33

a

e)

b

x b . c ya . zc

b) 13 e) 30

Si : a

b

c(x a

x b ) a(x b

x c ) b(x a

1 x2

(x 2

2

x 1

b)

x2

d) (x 2

x 1)2

x 1)2

(x 2 x 1)2

x c ) abc

P(x)

ax 3

bx 2

cx d

Q (x )

2x 3

x2

3x 1

Si : P(x)

Q (x

1) ,

determinar el valor de : a+ b + c + d

c.

a) 6 d) 15

b) 9 e) 18

c) 12

a) 0 d) 3

b) 1 e) 5

60. Si : R ( x 3)

57. El polinomio : m

n

p

q

A(x) ax bx cx dx mp es completo y ordenado, con suma de coeficientes igual a 13. Indicar : a + b + c + d. a) 5 d) 6

1 x2

0

59. Sean : P, Q dos polinomios dados por :

c) 27

56. Calcular la suma de coeficientes del siguiente polinomio completo : P(x )

c)

, x

x2 1 x

c) 8

55. Si el trinomio :

S(x ; y; z)

x2

1)

b)10 e) 9

c) 8

5

c) 2

x 1

Además : R (F 2 x (

9

7)

)

20 x

1

Calcular : F(x) . a) 15x - 9 d) 18x - 29

b) 8x - 129 e) -18x + 129

c) 18x - 129

Claves 01.

c

31.

c

02.

b

32.

e

03.

c

33.

c

04.

d

34.

c

05.

c

35.

e

06.

b

36.

b

07.

a

37.

c

08.

d

38.

c

09.

c

39.

c

10.

a

40.

a

11.

a

41.

d

12.

e

42.

c

13.

a

43.

c

14.

d

44.

c

15.

c

45.

e

16.

d

46.

c

17.

e

47.

b

18.

c

48.

e

19.

e

49.

d

20.

e

50.

a

21.

d

51.

c

22.

e

52.

a

23.

b

53.

c

24.

b

54.

d

25.

e

55.

c

26.

b

56.

e

27.

c

57.

a

28.

c

58.

e

29.

b

59.

b

30.

c

60.

c

Capítulo

3

PRODUCTOS NOTABLES

MULTIPLICACIÓN ALGEBRAICA Es la operación que tiene como objetivo determinar una expresión algebraica llamada producto, dadas otras expresiones algebraicas llamadas multiplicando y multiplicador, la igualdad obtenida es una identidad. Ejemplo : (x+2) (2x+1) = 2x2 + 5x + 2

multiplicando y multiplicador

identidad

producto

PRODUCTOS NOTABLES O IDENTIDADES ALGEBRAICAS 1.

Binomio al cuadrado b)2

a2

2ab

(a b)2

a2

2ab b 2

(a

Nota : (a b)2 2.

(b a)2 en general : (a b)2 m

(a

b)2 (a b)2

2 (a 2

b2)

4 ab

Diferencia de cuadrados (a b)(a b)

4.

((b b a)2 m ; (m Z)

Identidades de Legendre

(a b)2 (a b)2

3.

b2

a2

b2

Binomio al cubo (a

b)3

a3

3a 2 b 3ab 2

b3

ó

(a

b)3

a3

b3

3ab (a

b)

Identidad de Cauchy

(a b)3

a3

3a 2 b 3ab 2

b3

ó

(a

b)3

a3

b3

3ab (a

Identidad de Cauchy

5.

Identidades de Steven (x

a)(x

b)

x2

(x a)(x b)(x c)

6.

(a x

3

b)x

(a b c) x 2 (ab ac bc) x abc

Suma y diferencia de cubos b)(a 2

ab b 2 )

a3

b3

(a b)(a 2

ab b 2 )

a3

b3

(a

ab

b)

7.

Trinomio al cuadrado (a

8.

c )2

b

a2

b2

c2

2ab

2 ac

2 bc

Trinomio al cubo (a

c)3

b

a3

b3

c3

3a 2b

3 a 2c

3 b 2c

3 b 2a

3c 2a

3c 2b

6 abc

ó (a b c)3

a3

b3 c 3 3 (a b)(a c)(b c)

IDENTIDADES ADICIONALES 1.

Identidad de Argan'd

(a 2n * 2.

3.

a n bm

b2m )(a 2n

a n bm

Caso particular : (x 2

b 2m )

a4n

x 1)(x 2 x 1)

a 2n b 2m

x4

b4m

x2 1

Identidades de Lagrange (a 2

b 2 )(x 2

(a 2

b2

y 2)

by)2 (ay

(ax

c 2 )(x 2

y2

z2 )

bx )2

by cz)2

(ax

(ay bx )2

(az cx )2

a3

c3

Identidad de Gauss

b c )(a 2

(a

b2

c2

ab ac

bc )

b3

3abc

de donde : 1 (a b c)[(a b)2 (b c)2 (c a)2 ] 2 4.

a3

b3

c3

3abc

Otras identidades : (a

b c )(ab ac

(a b)4 (a b)4

(ab ac

bc)2

bc )

(a

8ab (a 2

a 2b 2

b)(a c )(b c) abc

b2)

a 2c 2

b2c 2

2abc (a

Algunas Relaciones Condicionadas : I.

(bz cy)2

Si : a + b + c = 0 1.

a2

b2 c 2

2.

a 3 b3 c 3

3abc

3.

a4

b4

c4

1 2 (a 2

4.

a5

b5

c5

2 (ab ac bc)

c 2 )2

5abc (ab ac bc)

II.

Si : x; y; z R / x 2 y 2 entonces : x = y = z.

II.

Si : x; y; z

R

b2

z2

xy

yz

m; n; p Z / x 2m

entonces : x = 0; y = 0; z = 0.

zx ,

y 2m

z 2p

0,

b c)

EJERCICIOS PROPUESTOS

x3

01. Si :

y3

20 ; xy

08. Si :

5

Calcular : M a) 40 d) 30

y)3 15(x

(x

y) 15

b) 35 e) 15

c) 20

2

1 a

a

a) 27 d) 4,3758

b) 6 e) 0

c) 12

E (a

b)(a 2

b)(a

b 2 ) (b4

b) 2b 2

a) 2a 2 4

a4)

c) 2a 4

e) 0

E

a) 6 d) -6

x3

y3

5

x2

y 2 16

c) -9

2

x

)(x 3

x

3

)

1 x

c) 728

x y

d)

e) 11

3

y)2

(x

0; y

0

1 2

1 2

y x

x2 a) (a-2)(a+2) 2)

x

2

y3

x y

a) 3 d) 6

167

y 2) , el valor de :

3x 2 y 5x

2

6y es : 2x y

b) 4 e) 2

V

1

c)

2 (x 2

3x 3

3 ; determinar el valor de :

1

1

3

x2

x

b) 36 e) 23

2

2

c) 5

12. Calcular :

si :

1 x

1 y

x2

y2

x

xy

2y 2x

2y x 3y

4 x y

c) 25 a) 2 d) 4

07. Determine :

e) a

167 ; x

b) 13

11. Si :

E x3 x 2

2

y x

a) 12

E

06. Sabiendo que : x

2 )(a

c)

E

b) 240 e) 3

a) 49 d) 18

1

Calcular :

4

a) 243 d) 120

c) (a

b) 1 e) 4

c) -3

b) -7 e) 10

Hallar : (x 2

n 3)

y a.b = 3.

a) 6 d) 12 1 x

a) 2 d) 3

x y

Entonces (a b)2 es :

05. Si : x

3

Si : mn = 2 y m+n = 2 2 .

64

b) -4 e) 2

b

(m

10. Si :

03. Si : x+y = 4; calcular :

04. Si : a

es :

a3

09. Hallar el V.N. de :

02. Efectuar :

d) 2b

1

3 3 , entonces a

1 x

; si : x

a

b) 3 e) 6

13. Calcular : 3

b) a 2

2

d ) (a

2 )(a

2)

c) 1

a) x - 3 d) -3

x2 x

x5

27

b) 3 e)

3

x2 x

x5 c) x

3 5

x

27

14. Calcular : b)(a 2 ab b 2 ) (a

(a

a) 4 a 3

b) 4 b3

d) 2b 3

e) b3

b)(a 2

ab b 2 ) 3a 3

c) 5a

3

b

) (a b a

b

1

1 d) 6

c) a

6b

e) a 6 b

4b

)

b) (a b

a

a6 b

d) a 6 b

a

c) 3

1

1

e) 5

a3

) es :

a

b3 c 3

2

b

2

c

3

2

2

b 6

)

(a

E

6b

6b

a

n ; n {1, 2 , 3}

21. Sabiendo que :

) (a 4 b 1 a

b 6

a

cx .

b) 2 1

Calcular : a) (a b

bx

Calcular : abc, además : F(n) a) 4

15. La expresión simplificada de :

(a b a

ax

20. Sabiendo que : F(x )

a) 1/3 d) 1/2

16. Hallar el V.N. de :

b c)(2 ab ac 1 abc

b) 3 e) 1

bc )

c) 2

22. Evaluar : E

(a

b)

2

2

(a

c)

(b c)

2

(a

b c)

para : a

5

3; b

a) 0 d) 50

7; c

5

40 2 5

b) 10 e) 40

c) 47

17. Sabiendo que : x + y + z = 1 Calcular : 3

3

x y z 1 xy yz zx xyz

M a) 1 d) 3

3

b) -1 e) 2

18. Si :

c) -3

x+y+z=3 xy + yz + xz = 0

a) 3 d) -1

x3

y3

z3

c) -2

19. Calcular el producto abc, sabiendo que :

a

3

b

3

c

3

b2

c2

645

a) No se puede determinar. b) 80 c) 70 d) 60 e) 75

a) 2 d) 16

b) 4 e) 32

c) 8

a) 4

b)

c) 2

d) 2 2

e) 4 2

23. Si :

a

4

b

4

8

4

2

8

4

2

a b

Calcular :

3xyz

b) 2 e) 1

a + b + c = 15; a 2

16 3 . 5 .17. 257 1

24. Si : m2 Calcular :

Calcular : 3

2

93

b a

2

n2

m2

n2

R

m2

n2

a) 2

2 b) n

d) m2

e) 0

n2 m2 n2 c) 1

25. Si : mn n2 3n m

n

nn

nn

mn

n

nn

nn

mn m 2

m5

n5

mn n 2

m2

m3

mn

Calcular el valor de :

a) 1 c) n

b) 0 2

e) n - 1

c) m + n

26. Reducir :

31. Sean "a" y "b" números reales positivos, tales que : K

3

(m

4)

(m

a2

3)(m 4)(m 5)

13 y b.a = 13 ( )

b2

1

Simplificar la expresión : 2

a) m d)m+4

b) m e) m+8

c) m+3

a

13

a

27. Determinar el valor numérico de :

y x 1 y 1 x ( )( ) ( )( ) x y 1 x 1 y Siendo : x

4

9

2; y

a) 1

b) -1

d) -2

e) 2 2

3

4

a)

4 c) 2

M

b2 c 2 a

a) 1 d) -1

c2 a2 b

c2

a2 c

y

a)

(s

p)2 (s

p)2

b)

s4

2ps 2

3p 2s

c)

s4

ps (1 s)

d)

s4

ps

e)

0,25 s 4

2 (a1 n

n

a

a2

a) 1 d) 4

ab bc

c) 3

b) 2 e) 6

c) 3

(a b)[(a b)2 2ba (a b)2 ] 2b3

3 2 p 2

para : a

3

3; b

a) 4 d) 7

p2

23 3

2 1

b) 5 e) 8

c) 6

35. Dado : M3

2 2 2 2 a a ... a 1 2 3 n

a) 2a d) 8a

c) n

b) n

2 (a b)[(a b)2

2ab (a b)2 ] (a b)

[(a b)2 4(a 2 b 2 ) (a b)2 ] Hallar el valor de "M".

a 3 ... a n )

(x a1 )2 (x a 2 )2 (x a3 )2 ... (x an )2

d) n - 1

ac

34. Hallar el valor numérico de :

p4

Calcular :

a) 0

ac ab bc

(x 1)3 (y 2)3 (z 3)3 (x 1)(y 2)(z 3)

30. Sabiendo que : x

ac) ; a, b , c R

33. Si : x + y + z = 6, calcular :

es igual a :

3 2 p 2

ps 2

3 (ab bc

b) 1/2 e) 0

2

2

1

1)

bc ab ac

a) 2 d) 1

29. Si se tiene como suma "s" y producto "p" de dos cantidades x, y, entonces :

x

2(

b c )2

A

b2

c) 3

2

c)

Calcular :

b) 0 e) 2

2

e)

2

(a

b2

b 3a x

32. Si :

28. Si : a +b+c = 0, reducir : a2

ba x

3 x

b)

5

d)

x 1

2

b) 2b 3

e) 8 b

c) -2ab 3

36. Dado el polinomio : P(x)

(x 2 1)(x 2 x 1)(x 2 x 1)

obtener :

2

e) (n 1)

P( 4 a) 0 d) 215

15

b) 217 e) 218

4

15 ) c) 216

37. El valor numérico de : 3

(x 3

44. Sabiendo que :

3 x)2 (3x 2 1)2 ;

38. Si : n

3

2 1

Calcular : a)

b) 991 000 e) 998 000

R

n

3

d) 2

a) 2 d) -1

3n 2

b2

a) 1 d) 4

3

a (a 2b) b2

1

c

(ab)4

23 2x , calcular el valor de : x 8

2

42. Si : b 3 1

c)

8

a2

e)

2

b2

c2) c) 18

z2

x bc

b2

1

(x

y z)(xyz)

x

3

y3

z3 c) 1

x ac

1 a

1 b

x ab

c2

1

1

5abc (a

c2

d) 3abc

1 c

a b c

a 1bc b 1ca c 1ab

Calcular : a3

a) 0 d) 1

b) 1 e) -2

c) -1

49. Si : a 4

x 1 0 , hallar : 31

1

b) ab+bc+ca

b2

c) 5

5 Obtener : 1 b . 4 b

b) 1 e) -10

2y 3z) 14

b) 4 e) 2

1

x

2 (x

48. Si :

b) 2,5 e) 4,5

43. Si se cumple : x 2

a) 3 d) -1

c) a 2

b 2)

b

y2

a) a+b+c

b3 ) .

0;

(bc )4

47. Si : a+b+c = 0, hallar el valor de "x".

2x

ab (a b) 1

a) 3 d) 4

(ac)4

a 2 b 2c 2 (a 2

M

e) 5

Hallar : a 3b 3 (a 3

0 , calcular :

Calcular : c) 3

2

a 2b 2 (a 2

1

b) 6 e) 3

x2

b) 4

5

a) 2 d) 4

4)

b) 2 e) 5

a) 3

c) 1/2

46. Siendo : x, y, z R, tales que :

E

a) 2 d) 3

b

R

1

40. Sabiendo que : x

a) 0 d) 2

1

c) 0

(x 1)(x 2 x Calcular : ab 1

41. Si :

45. Si : a

2

a (a 2b) b 2

3

Donde : a 2

d)

b) 1 e)-1/2

e) 1

39. Si : x

y 2 (y 1)

2 1

b)

3

x 2 (x 1)

K

c) 100 000

1

3

(x 1)(y 1)

Calcular :

para : x = 999 es: a) 19 990 d) 999 000

y)2 1

(x

x

b 3 c 3 3abc a b c

b) 1/3 e) -1

b4

47

ab N

c) 2/3

1 , reducir : a6

b6

2

10

c) -1

a) 3 d) 10

b) 14 e) 18

c) 20

b

1

c 1)

50. Hallar el valor de "E" : E

(x

3

y

3 z .(x+y+z)(x+y) (x

z)

a) x 3

b) y 3

d) 0

e) z 3

51. Si : H

d)

2x 1 2

52. Si : (xy

2) 196

x 1 2

b)

c) x + 2

e) 2x - 1 1

1)(yz

Calcular : R

1

a) 2 d) 4

y

1

1)

1

z)(x

y

8 1

b) 3 e) 1 ax

53. Si : P(x)

3m2

c3

7m 3

b6

z 1)

13m 3

3 b) 6m

d)

m3

e) 7 m 3

a b

57. Si :

c) 2m 3

1

a b

7

8

a b

1

a b

8

b) 3 e) 1

c) 5

58. Si se cumple :

x n y n ( ) ( ) 62 y x Entonces el valor numérico de :

1 , entonces : 1 p(10) 2

p(4)

a)

a) 2 d) 4

c) 2

bx, a6

b2 c 2

Calcular : R

1)(zx

(x

a2

S = (a+b-c)(b+c-a)(c+a-b)

H 16,25

a) 2x + 1

y)

56. Siendo : a+b+c=m

a 3 b3 Calcular :

c) 3z

(x 5)(x 6)(x 1)(x

Hallar : R

3

3

a) 4 d) 2

p( 2)

xn

yn

x ny n

b) 8 e) 1

c) 16

es equivalente a : 4 4

2 2

a) a b

b) ab

c) 4 a b

d) 2a 4 b 4

e) a 2b2

59. Sabiendo que : x 2 Obtener :

5x 1 3

(x 3 140) x11

A

3

54. Evaluar :

4 ab (3a 2

E

b 2 )(a 2

3b2 )

Para : 3

a

3

3

2

2

a) 2 d) -4

;

3

b

3

3

2

2

b) 3 e) 5

a) 1 d) 3

x

y z

a

y3

z3

2

2

2

x

a) 0 d) 3

y

z

x

z y

xy xz yz

4 xyz xy

xz

b) 1 e) -3

1

a2

Donde : x3

b) 2 e) 1/3

a b c

c) -3

y z x

yz 1

5

1

b b2

c

c) 1/2

2

1

c2

Calcular : a 3

5 2

b3

2 5

c3 .

a) 4

10

b) 7 2 10

c) 5

10

d)

e) c) -1

1

60. Si :

55. Calcular el V.N. de :

R

x4

5

2

5

2

Claves 01.

b

31.

c

02.

c

32.

b

03.

a

33.

c

04.

b

34.

c

05.

c

35.

a

06.

c

36.

d

07.

c

37.

e

08.

e

38.

c

09.

c

39.

d

10.

b

40.

d

11.

c

41.

b

12.

d

42.

c

13.

d

43.

c

14.

c

44.

b

15.

d

45.

a

16.

d

46.

c

17.

d

47.

b

18.

a

48.

a

19.

b

49.

e

20.

d

50.

e

21.

b

51.

d

22.

a

52.

b

23.

e

53.

e

24.

a

54.

e

25.

a

55.

e

26.

d

56.

a

27.

e

57.

e

28.

b

58.

d

29.

e

59.

e

30.

b

60.

b

Capítulo

4

DIVISIÓN ENTRE POLINOMIOS DIVISIBILIDAD ALGEBRAICA COCIENTES NOTABLES

DIVISIÓN DE POLINOMIOS Es la operación que tiene por objetivo determinar un polinomio llamado cociente (q) y otro polinomio denominado resto o residuo (R), conociendo otros dos polinomios llamados dividendo (D) y divisor (d). Esquema clásico : D R

de donde :

d q

D = dq + R

(Identidad de la División).

Propiedades : Siendo el grado del dividendo mayor o igual que el grado del divisor, con respecto a una variable en particular, es decir : [D]° [d]°. Se cumple : 1.

El grado del cociente es la diferencia entre el grado del dividendo y divisor.

[q]º = [D]º - [d]º 2.

El máximo grado del resto es igual al grado del divisor disminuido en uno. [R] ºmáx= [d]º - 1

MÉTODOS DE DIVISIÓN Para todos los métodos, el dividendo y divisor deben estar completos (si falta algún término se agrega "cero") y ordenados en forma decreciente. I.

MÉTODO DE HORNER Para este método sólo se utilizan coeficientes, colocándolos en el siguiente esquema :

Primer coeficiente del divisor los demás coeficientes del divisor con signo cambiado

d i v i s o r

D I V I D E N D O # lugares = dº

COCIENTE

R E S T O

Ejemplo :

x5

Dividir :

4x4 4x

2

6x 2 4x

6x 1 2

Colocando según el esquema, los coeficientes del dividendo y divisor :

# lugares = dº = 2

4

8

4

4

0

8

-4

6

12

6

-6

-2

8

-4

por

2

3

-1

2

2

Coeficientes del "q"

8

-4

10

-5

Coeficientes del "R"

sólo se obtienen coeficientes. La variable se agrega de acuerdo al grado . Así tenemos : q° = 5 - 2 = 3 ; Rºmáx = 2 - 1 = 1.

q R II.

2x 3 3x 2 10x 5

2x 2

MÉTODO DE RUFFINI Al igual que en Horner, sólo utilizan coeficientes. Ruffini se aplica únicamente cuando el divisor es de la forma : x + b. Esquema de Ruffini :

-b

DIVIDENDO COCIENTE

siempre es un número

R

valor de "x" al igualar el divisor a cero.

Ejemplo :

3x 4

8x 3 5x 2 x 2

5

Colocando los coeficientes en el esquema de Ruffini : x

2

0

3

2 por

3

8

5

0

5

6

4

2

4

2

1

2

9

coeficientes de "q"

Las variables de "q" se agregan de acuerdo al grado : q° = 4 - 1 = 3.

q R

3x 3 9

2x 2

2x

2

R

Observación : si el divisor es ax + b (a "a".

Ej. :

3x 4

7 x 3 3x 2 3x 2

1), luego de realizar la división, los coeficientes del cociente se dividen entre

x 7

3x - 2 = 0 2 3

3

7

3

1

7

2

6

2

2

3

9

3

3

9

1

3

1

1

3

qº = 4 - 1 = 3

q

x3

R

9

3x 2

x 1

TEOREMA DEL RESTO El resto de dividir el polinomio P(x) entre (x-a) es P(a). Observación : *

Si el divisor no es de primer grado, se calcula alguna expresión según el caso y tal cual, se reemplaza en el dividendo. Ejemplo : Hallar el resto : x 50

3x 21 7 x x 1

Por T. resto :

x+1=0

Reemplazando en el "D" :

2

x = -1

R

( 1)50

3 ( 1)21 7 ( 1) 2

R=1-3+7+2 R=7 Ejemplo : Hallar el resto :

x 20 7x 5 x Por T. resto :

x2 1

6x 4 2

x3 1

1 0

x2

Formando " x 2 " en el dividendo :

1 (no se calcula "x").

(x 2)10 7 (x 2 )2 x 6 (x 2 )2

Reemplazando :

x2

1

R (1)10 7 (1)2 x 6 (1)2 (1) x 1 R = 1 + 7x - 6 + x + 1 R = 8x - 4

x 2. x 1

DIVISIBILIDAD ALGEBRAICA Se dice que un polinomio es divisible entre otro, si el resto de dividirlos es cero; es decir : Si en : P(x)

f(x)

R=0

Entonces P(x) es divisible entre f(x). Propiedades : 1.

Si un polinomio es divisible entre otros polinomios por separado, entonces será divisible entre el producto de dichos polinomios, siempre que estos sean primos entre sí, (no deben tener ningún factor en común); es decir : Si en :

* 2.

P(x) P(x)

f(x) g(x)

R=0 R=0

P(x)

f(x) . g(x)

R=0

f(x) y g(x) son primos entre sí.

Si un polinomio es divisible entre un producto de varios polinomios, entonces será divisible entre cada uno por separado; es decir : Si en :

P(x)

f(x) . g(x)

R=0

P(x) f(x)

R

0

P(x) g(x)

R

0

COCIENTES NOTABLES (C.N.) Se llama, así, a los cocientes exactos obtenidos de la división de binomios de la forma :

xn x

an a

Condiciones :

R n

0 entero y positivo

Propiedades :

1.

En :

2.

Si :

xn an , el número de términos del cociente será "n". x a

xm

an

p

aq

x

es un C.N., entonces se cumple que : m p

n q

# tér minos del cociente

FÓRMULAS DE LOS COCIENTES NOTABLES 1er. Caso : n

par o impar

xn an x a

x n-1 x n 2a x n 3a 2 .... a n

1

2do. Caso : n

impar xn x

3er. Caso : n

an a

x n-1

xn

2

a

x n 3a 2 ... a n

1

par xn an x a

x n-1

xn

2

a x n 3a 2 ... a n

xn an no genera un C.N. pues R x a

Observación : La forma

1

0.

TÉRMINO GENERAL(Tk ) Se llama así a un término cualquiera del C.N. se representa por Tk . La fórmula para obtener el término general en: xn an x a

es :

Tk donde :

k x, a n

x n k a k-1

lugar de término.. términos del divisor (denominador). exponentes que se repite rrep re epite epi pite pi ite te en el dividendo..

Importante : para aplicar la fórmula, la división debe tener la forna de C.N.

Ej.

Calcular el T17 en :

x120 x

y180

2

y3

Solución : x120

y180

x2

x2

y3

x2

no tiene forma

T17

(x 2 )60

17

(y 3 )17

60

y3

60

y3

tiene forma de C.N.

1

T17

x 86 y 48

Observación : la misma fórmula puede aplicarse para los casos : xn an xn y x a x

así tendremos :

an , pero colocando el factor ( 1)k a

Tk

( 1)k 1 x n k a k-1

1

EJERCICIOS PROPUESTOS 01. Sea : Q(x) el cociente y R (x ) el residuo de dividir :

6x 4

7x 3 3x

Indicar : Q(x )

4 x 2 10x 3 2

x3

R (x ) .

6x

b) 2x 2

c) 2x 2

3x 2

d) x 2

e) 2x 2

2

a) -2 d) 2

b) -1 e) 3

x4

5

9x

6x 3

d)

x

e)

x 2 2x

3

x

2

a) -8 d) 1

x

3x 2 1

b) x

a) -2x+1

2

b) -4 e) 9

2x 1

c) 2x+1

03. El residuo de dividir :

4x3

Ux 2

2x 3

x2

Calcule el valor de : P(10001) .

a) 20 d) 50

Nx

I

10. Calcular el residuo de dividir : (x 2

c) 40

a) 88 d) 95

3

4x

b) 2 e) -1

2

5x 6 c) 4

06. Al efectuar la división : 3x

4x

3x

2

2 2x

b) 4 e) 12

c) 87

Cx 3 4x

3

a) 41 d) 10

27x 2 19x 5 3x 1

b) 21 e) 40

2

2x 6

c) 6

c) 11

12. Señale la relación necesaria de "a", con "c", tal que la división :

2a 2x 5

4 abx 4

2b2x 3 a 3x 2 a 2x ax 2

presente un resto : 4 a 2x 3

Indicar el producto de todos los coeficientes del cociente. a) 2 d) 8

x

es exacta. c) 14

05. El residuo de dividir : 3x entre 3x + 2 es :

4

Bx 4

Ax B es exacta, entonces el

b) 9 e) 20

a) 0 d) 1

3)5

b) 89 e) 98

A Ax 5

3x 2 2x 1 valor de : N = A+B, es : a) 5 d) 19

3 x 1)4 2 (x x 4

11. Calcular : (A+B-C), si la siguiente división:

04. Si la división :

25x

c) -1

U.N.I .

b) 30 e) 60

2

b) -2 e) 1

3

es : 5 x 2 11x 7 . Calcular :

3

x 3 10000x 2 10002x 9999

P(x )

a) -3 d) 0

8x5

c) -1

09. Dada la función polinomial :

2x 1

16x

5x 2 4 x m

sea divisible por : x+1, el valor de "m" debe ser :

12x

6x

c) 1

08. Para que la siguiente ecuación :

6x 2

02. Hallar el residuo de dividir :

4

nx 2 nx n 2 x n 2

x 2

a) 2x 2

2

07. Calcular "n", para que el residuo de la división sea : 3n+2.

bx c 2c 2

a 2c .

a) 3a = 2c

b) 2a = 3c

c) a = c

d) 3a

e) 3a = -2c

2c

2a 2 b

13. ¿Para qué valor de "m", la división :

5x 3

m (x 2

5x

2

x 1)

es exacta?

2x 4

a) 5 d) 8

a) 5 d) 32

b) 6 e) N.A.

b)

2 +7

d) 13 2

e) 9 2

c) 7 2

3 ) x3

(1

2 3 (x 2 1) A

x 1

2x

3

c) 9

16. Calcular el resto de dividir : 7

(x

n) x

a) 0

b) 126n7

d) 62n7

e) 128n7

7

21. Hallar la relación entre "b" y "c" para que :

bx c ; sea divisible entre (x 1)2 .

xa

b) b = c + 1 d) 2a = c + 1

22. Si en la división : 1

(2a 1) x a

2

(3a 2) x a ax 1

x 2n

n

b) 9 e) 3

x7

c) 3n7

8n

x7

x15

x5 1

x 1

b) -x e) 0

c) x+1

18. Indicar el residuo obtenido al efectuar la división :

mx 3m

2

nx 3n

1

px 3 p

x2 1 x a) (m - p)x + m - n c) (n - m)x + p - m e) (m+1)x + n - p

b) mx - n + p d) (m + p)x - n

nx 1

x2 1 Calcular : n6 .

10 n

x7

; es : (-4x+1).

7n

... " n" sumandos 1

b) n2

a) -n

e) - n

c) 0

2

24. Calcular la suma de los valores de "a" que hacen al polinomio : P(x)

xn

ax n

1

a) 4 d) 7

ax 1 ; a Z

divisible por (x 1)2 .

b) 5 e) 8

c) 6

25. Calcular el resto en : [(x 2)n

2

2x 2n 1 ] (x 3 x

19. Si el resto de dividir :

6x 3

9n

2003 .

d) 2003 a) x d) 1-x

c) 8

x7

n N/n

x2

... (a 2 a 1)

23. Calcular el resto de la siguiente división :

17. Hallar el resto en :

(x 3 1)29

3

el cuádruple del resto es igual a nueve veces la suma de coeficientes del cociente. Hallar "a". a) 10 d) 6

7

1

c) -1

es 2. ¿Cuánto vale A? b) 6 e) -6

3x 5 1

b) 1 e) 3

ax a

a) 18 d) 8

3

es 8 x 2 px 5 . Calcule : m + n + p

a) a = c - 1 c) 2a = c - 1 e) a = 2c - 1

15. El resto obtenido en :

nx6 x

a) 0 d) 2

2 2.

a) 8 2

3x4

mx 8

c) 7

x 5 (2 2 2) x 4 4 2x3 5x 3 2

Para : x

c) 16

20. Si el residuo de la división :

14. Calcular el valor numérico de :

P(x)

b) 15 e) 64

a) 0 d) x

2

x 1

b) 1 e) -x

2)2n

x 2n

1

;n Z

c) 1

26. Obtener el término independiente del cociente de : x18 (x 3 1) 5 x14 x 1 a) 10 d) 6

3x 4

b) 8 e) 2

11

a) 2 d) -2

c) 4

27. Si se divide el resto de la siguiente división: x7 n

x6 n x

entre x 2

3

n 1

2x 5 n x

n 2

1

3x 4 n

3

... x 1

2 ; se obtendrá como resto :

a) x d) -1

b) x + 1 e) 0

c) 1

28. Calcular el valor de "n" para que : (x 1)n (x 3 (x

2

2x

8)

2

.(x 2 8 x 16) 29x 4 (2 x)4

2)

presente un resto de 11 200. a) 6 d) 3

b) 5 e) 4

c) 2

29. Calcular el residuo que se obtiene al dividir: (x 9

2x 4 (x 4

x)(x

2)(x

2)

b) 5x 2

c) x 2

2 d) 5x 14x 8

6x 8

e) 3x 2 12x 6 30. Deteminar: a+b+c, de modo que : (x 1)5

a(x 1)3

a) 40/3 d) 184/3

31. Si al dividir :

bx

3 c ; es divisible por (x 1) .

b) 70/3 e) 52

P(x) x 2

c) 94/3

. El residuo es 8 y el cociente (x 2 1) ,

hallar : P(4 ) . a) 40 d) 32

b) 42 e) 18

32. Si al dividir P(x) entre (x

c) 30

2

x)(x

3) , se halla por resto

(6x +5), hallar el resto de dividir P(x) entre x - 3. a) 20 d) 12

b) 23 e) 18

b) -4 e) 0

c) -1

34. Un polinomio P(x) de tercer grado es divisible por separado entre (x - 2); (x+1) y (2x+1). Si la suma de sus coeficientes es -30, hallar el cociente de dividir P(x) entre el producto (x-2)(x+1)(2x+1). a) -4 d) -6

b) x + 1 e) 6

c) 5

35. Un polinomio es dividido en forma separada entre (x-4), (x+4) y (x-1); obteniéndose el mismo residuo 5. Hallar el residuo que se obtiene al dividir dicho polinomio entre (x 3 x 2 16 x 16) . a) 2 d) 0

b) 5 e) 4

c)10

36. Un polinomio de tercer grado cuya suma de coeficientes es -76, es dividido en forma separada entre (x+1), (x+3) y (x-3); obteniéndose el mismo residuo 4. Calcular su término independiente. a) -31 d) 19

b) -37 e) 21

c) -41

2)

a) 5x + 4

2x 6

33. El polinomio P(x) es divisible en forma separada entre (x-2), (x+4) y (x+1). Hallar el residuo que deja la división de P(x) entre (x 3 3 x 2 6 x 8) .

c) 2

37. Si A(x) es un polinomio de segundo grado, tal que al dividirlo entre (x-5) y (x+3) en forma separada deja residuo igual a 7. Calcular el residuo de A(x) (x 1) , si : A(x) (x-4) deja residuo -7. a) -17 d) -10

b) 15 e) -6

c) 12

38. Al dividir un polinomio mónico P(x) de tercer grado por separado entre (x 2

2) y (x + 1) da el mismo P(x ) resto 8, hallar el resto de dividir : . x 3 a) 24 d) 15

2x

b) 12 e) 17

c) 28

39. Se divide P(x) entre (x+1) y (x-1), los restos respectivos son 2 y 4. Hallar el resto de dividir dicho polinomio entre x 2 1 . a) x + 2 d) x + 3

b) x e) -x + 3

40. El polinomio : (x

c) -2

2)51 (x 1)40

No es divisible entre : x

2

7.

3x 2 .

Indique su residuo. a) 2x + 1 d) 2x + 4

b) 2x - 1 e) 2x

c) 2x - 4

41. Si al dividir : P(x) entre (x - b) da como resto "a" ; al dividir P(x) entre (x - a) da como resto "b". Hallar el resto que resulta de dividir :

P(x ) (x a) (x a) x + ab c) -x - a + b e) -x + 2ab

b)

(a

b)

b) -x + ab d) -x + a + b

a) 27 d) 512

42. Al dividir el trinomio : ax 2 bx 2 entre (x-1) y (5x-13) dio como restos -1 y 15, respectivamente. Hallar el valor de : (a - b). a) 13 d) -1

b) 10 e) -13

b) -x e) -x - 1

b) 15

d) x + 3

e) x 2

x2

d) 4 x 2

e)

3x 2

c)

2x 2

46. Se tiene un polinomio de segundo grado que es divisible entre (x - 1). Se sabe además que su término independiente es -3 y que al dividirlo entre (x + 1) se obtuvo como resto 8. Hallar el resto que resulta de dividir el polinomio entre (x - 3). b) 22 e) 56

c) 36

47. Los restos de las divisiones de un polinomio entero en "x" por los binomios (x+3), (x - 2), (x - 1) son 16, 11 y 4 respectivamente. Entonces el residuo de la división de dicho polinomio entre x 3 7 x 6 será :

d) x 2

c) x 2 1

b) 2

x 1

e) 2x 2

c) 2300

2

como residuo 3x. Hallar el residuo que origina S(x ) entre (x 2

x 1) . b) 3x - 3 e) 9x - 9

x 1

c) 3x + 3

51. Un polinomio P(x) , al ser dividido entre (x 2 1) , da como residuo (-x + 1). ¿Cuál será el residuo en? [P(x )]7 x2 1

3x 1

b)

a) 1

b) 7200 e) N.A.

50. Al dividir un polinomio S(x) entre (x 3 1) se obtuvo

a) x + 4 d) 6x - 6

45. Al dividir un polinomio mónico de tercer grado entre (x-2) y (x-4) en forma separada se obtuvo el mismo residuo -8, si su término independiente es 16. Hallar su término cuadrático.

a) 10 d) 48

a) 710 d) 1221

c) x 2 1

a) 3x 2

c) 427

c) x + 1

44. Los restos de la división de un polinomio entero en "x", por los binomios x+1, x-1 y x-2 son, respectivamente 5, -1, -1. Hallar el resto de la división del polinomio por el producto : (x 2 1) (x 2) . a) 0

b) 501 e) 511

49. Calcular el resto de dividir un polinomio P(x) del sétimo grado entre (x + 2), si se anula para : x = 3, x = 2, x = 1 y es divisible entre (x 2 1) y (x + 5). Además el, resto de dividirlo entre (x + 1) es 960 y su término independiente es 60.

c) -10

43. Dado el polinomio P(x) , si P(x) - 5 es divisible por (x + 5) y P(x) + 5 es divisible por (x - 5). ¿Cuál es el resto de 2 25) ? dividir P(x) entre (x a) x d) x - 1

48. Un polinomio P(x) de noveno grado, tiene raíz cúbica exacta, se anula para x = 2 es divisible entre (x + 2), el resto de dividirlo entre (x + 1) es 729, la suma de sus coeficientes es 27. Señala el término independiente de dicho polinomio.

a) x - 1 d) 8(x - 1) 52.

b) 4(x + 1) e) 4(x - 1)

c) 8(x + 1)

Se sabe que el polinomio F(x) es divisible por (xn - 1). Si se divide F(x) entre (x-1), se puede afirmar que : a) b) c) d) e)

Es exacta. La suma de los coeficientes del cociente es cero. La suma de los coeficientes del resto es cero. a ó c. Hay 2 correctas.

53. Se tiene un polinomio P(x ) que, al dividirlo entre : x 5 15x 4 85x 3 225x 2 274x 120 , se obtiene como resto : 3x - 1 y un cociente Q(x ) . Se pide calcular el resto de dividir P(x ) entre (x - 4), sabiendo que al dividir Q(x) entre (x - 4) se obtuvo como resto 1. a) 11 e) 20

b) -10 e) -11

c) -20

54. Al dividir P(x) entre (x + a) deja como resto 4bc. Al

60. Si "N" es el número de términos que genera el desarrollo del cociente notable :

2 2 dividir Q(x) entre (x + a) deja como resto b c . Hallar el resto que se obtiene al dividir :

P 2(x) Q (x )

a) 7 d) 13

P 2(x) es divisible entre Q(x) .

x

x2 a

2x a

56. Siendo: P(x )

x

x

3

c) x - 5

nx n divisible separada-

el residuo de dividir P(x) entre :

a) 2 d) -1

1

1

b

c

1

b) 0 e) -2

d 1)

57. Encontrar el término central de un polinomio de la

a) 3 d) 7

x n , sabiendo

que el resto que resulta de dividirlo entre (x - 1) es 153. a) 10x 10

b) 9x 9

d) 13x13

e) 7 x 7

c) 12x12

58. Si el cociente notable :

x 30

xm

xn

y2

b) 21 e) 50

bn

a3

b2

y3

a) 6 d) 13

b) 12 e) 27

c) 15

a)

x15 x

b)

c)

3

y 20 y4

y 20

m 25

y4

m5

a 40

b 28

a10

b7

x 21 1 x3 1

a)

m72

m54

b)

y8

c)

x 35 x 30 x 25 x 20 x15 x10

x 8 y6

m 36

m18 1

x16 y 4

x 24 y 2

x 32

x5 1

65. Hallar el vigésimo tercer término del desarrollo del cociente :

5

x120 x5

y 96 y4

Señalar la suma de exponentes.

m n b) 3 e) 7

y 2m

genera un cociente notable. Hallar el número de términos de dicho cociente notable.

tiene 9 términos en su desarrollo, calcular :

a) 1 d) 5

27

64. Indicar el C.N. que origina a :

59. Siendo que el C.N. 2

2 81

x

tiene 10 términos,

c) 25

am

c) 6

62. Si la siguiente división :

d)

hallar el valor de (m+n). a) 23 d) 35

a

6)

n 2

b) 5 e) 9

forma : 1

a5(n

63. Desarrollar los siguientes C.N. :

c) 1

nx (n 1) x 2 (n 2) x 3 ... 2x n

3

n 1

xm

mente entre los binomios (x-a), (x-b), (x-c), (x-d), señale

(x a

c) 11

15 a

a ) y (x + 3), entonces también

b) x - 3 e) x - 4 4

y10

x5n

será divisible entre : a) x + a d) x + 5

x5

5

b) 9 e) 28

c) 2bc

2x 2 15 x

es divisible entre (x

y5a

61. Hallar el número de términos del desarrollo del C.N. :

b) b 2c 2 e) 4

55. El polinomio : x 3

1

Indicar el valor de : "a + N".

entre (x + a). Se sabe además que :

a) 4bc d) 16

x 3a

c) 4

a) 91 d) 97

b) 93 e) 99

c) 95

66. Evaluar el quinto término del C.N. obtenido a partir de: x 36 x

y12

6

y

2

, para : x

8

2

4 a) 2

b) 2

d) 28

e) 1

26 .

e y

10

c) 24

x

x 5m a) 6 d) 18

y

260 n

y4n

es x 345 y 984 . b) 12 e) 24

68. Si : m - n = 27; y

c) 15

xm

yn

7

y4

x

genera un C.N.

b) 39 e) 42

x125

P(x ; y)

x

y3

x120

y 40

x3

90 m .

x y

y

Hallar : "m.p".

b) 110 e) 90

c) 132

71. Hallar el término central del desarrollo del siguiente cociente notable : x6k x 9 15 a) x y 5 9

d) x y

3

y8 k

3

5

y

3

b) 72 e) 111

3 5 b) x y

x5n

y 2n

x5

y2

25m

es x 2 . y 20 n)1 / 2 .

Hallar : (m

b) 2 e) 5

c) 3

75. Qué lugar ocupa el término independiente en el desarrollo del C.N. :

Q(x) a) 6 d) 9

x 27

x

9

x3

x

1

b) 7 e) No tiene

c) 8

76. Indicar el lugar que ocupa el término independiente del desarrollo del C.N. :

a) 3 d) 6

x 27

x

x

x3

x

5

b) 4 e) 7

c) 5

77. Calcular "m" para que el término independiente del C.N. :

x 24

m6 x

x4

mx

a) 6 d) 3

6 1

sea 81. b) 5 e) 9

c) 4

x6

c) xy

12 10

e) x y

6

a) 17 d) 22

b) 18 e) 21

x

x 9

x

4 1

c) 19

79. Si el término de lugar 4 contado desde el extremo final del desarrollo del C.N. :

2y)15 y15 3x y Indicar el valor de A(1; -2). (3x

b) -37 e) 128

c) 94

78. Hallar el lugar que ocupa el término independiente en el desarrollo de :

72. Si : A(x; y) es el término central del desarrollo del C.N.:

a) -128 d) 37

a) 24 d) 38

c) 15

70. Dado el cociente notable :

a) 72 d) 56

y75

5

b) 13 e) 16

Sabiendo que el TP

es z q w90 . z w5 Calcular el valor de "n - q".

c) 40

69. Determinar el lugar del término que presenta como grado absoluto a 88 en el desarrollo de :

a) 14 d) 17

wm

2

a) 4 d) 1

Hallar el grado absoluto del sexto término del desarrollo. a) 38 d) 41

zn

74. Si el término central del C.N. :

67. Calcular "mn", si el T24 del C.N. : 325 m

73. El término central del desarrollo del cociente notable :

x5p c) -64

x

5

y 2p y2

tiene grado absoluto 37.

Indicar el número de términos del desarrollo del C.N.

a) 10 d) 15

b) 12 e) 18

c) 14

83. Simplificar :

E 66 7 80. Si : x y C.N. :

5r

es el séptimo término del desarrollo del xp

yq

x11

yr

Indicar el término de lugar 5 contado a partir del extremo final. a) x 55 y 49

b) x 66 y 42

44 56 d) x y

e) x 5 y 66

81. Si el C.N. : Calcular : E a) 210 11

d) 2

x8 1 xm 1

m9

m7 ... m 1 .

m8

1

b) 210

1

1

11

1

e) 2

x 20

x18

10

9

x Hallar : E(-1/3). a) -1/9 d) 3

x

... x 2 1 x

b) -1/3 e) 9

8

... x 1

c) 1

x11 1 x 1

... x 2 1

34

... x 2 1

x

x

x

x 40

c) x 36

c) x 41

e) x 42

84. Reducir : E

x 22

x 20

(x 2

x18

... x 2 1

x 1)(x 2

x 1)

a) x 6

x3 1

b) x12

x6 1

c) x 6

x3 1

d) x10

x5 1

e) x12

x 18

x6 1

85. Si : x 12

F(x ) Hallar : F

E(x)

x74

36

b) 1

c) 29 1

82. Si :

x76

38

a) 0

c) x 55 y 35

tiene 4 términos en su desarrollo..

x78

a) 257 d) 127

x 24

x10

x8

x 20

x6 x16

... 1) ... 1

2 . b) 511 e) 510

c) 25

1

Claves 01.

b

31.

b

61.

e

02.

c

32.

b

62.

a

03.

c

33.

e

63.

-

04.

d

34.

c

64.

-

05.

a

35.

b

65.

b

06.

b

36.

c

66.

e

07.

a

37.

a

67.

d

08.

a

38.

c

68.

d

09.

b

39.

d

69.

d

10.

c

40.

d

70.

e

11.

c

41.

d

71.

a

12.

c

42. 4

a

72.

a

13.

d

43. 4 43 3

b

73.

d

14.

c

44. 4.

e

74.

e

15.

d

45.

e

75.

b

16.

b

46.

d

76.

b

17.

d

47.

e

77.

d

18.

c

48.

d

78.

d

19.

e

49.

b

79.

d

20.

b

50.

e

80.

d

21.

b

51.

c

81.

a

22.

b

52.

d

82.

d

23.

a

53.

a

83.

b

24.

b

54.

d

84.

e

25.

a

55.

c

85.

d

26.

e

56.

b

27.

c

57.

b

28.

c

58.

a

29.

d

59.

c

30.

e

60.

e

Capítulo

5

FACTORIZACIÓN

Factorizar un polinomio es descomponerlo en dos o más polinomios llamados factores, de tal modo que, al multiplicarlos, se obtenga el polinomio original. Ejemplo :

Ejemplo : 1. Factorizar : xy + xz + xw. Solución :

ya factorizado

x2 y2

(x y)(x y)

se extrae "x"

xy+xz+xw "x" factor común

polinomio factorizado

Antes de factorizar factores

Puede notarse que si multiplicamos (x+y)(x-y) se obtiene x 2 y 2 que viene a ser el polinomio original (la factorización y la multiplicación son procesos inversos).

Es aquel polinomio que no se puede descomponer en otros polinomios. Ejemplo : x

2

x

2

y7 z

y 3w

y

menor exponente

3

factor común

se extrae "y3"

y

no es primo (se puede descomponer).

y

2

es primo (no se puede descomponer).

y 3 (xy

y4z

w)

polinomio factorizado

3. Factorizar :

a 2 ab ac bc

El número máximo de factores primos que tiene un polinomio está dado por su grado. Así por ejemplo :

Sol. agrupando :

x 3 6x 2 12x 6 a los más tiene 3 factores primos.

a2

2.

Los polinomios lineales (primer grado) necesariamente son primos.

a (a+b)+c(b+ a)

3.

Sólo se pueden factorizar los polinomios no primos.

MÉTODOS DE FACTORIZACIÓN I.

y3 w

tendremos : 2

Propiedades : 1.

y7 z

Solución :

xy 4

Factor Primo

xy 4

2. Factorizar :

Método del Factor Común Se aplica cuando en todos los términos del polinomio se repite el mismo factor, el que se denomina factor común. Para factorizar, se extrae a cada término del polimonio el factor común, (si éste tuviese diferentes exponentes, se elige el de menor exponente).

x(y+z+w)

ab ac

bc

factor común : a + b

se extrae (a+b)

tendremos : (a+b)(a+c ) polinomio factorizado

II.

Método de las Identidades En este caso, se utilizan las identidades algebraicas (Productos Notales); pero en forma inversa, es decir teniendo el producto se calculan los factores que le dieron origen. Se puede utilizar cualquier Producto Notable estudiado; pero los que se utilizan con más frecuencia los recordamos en el siguiente cuadro :

Producto Notable

b2

(a+b)(a-b)

Trinomio Cuadrado Perfecto : a 2 (TCP)

2ab b 2

(a

b)2

Suma o Diferencia de Cubos : a 3

b3

(a

b)(a 2

Diferencia de Cuadrados :

a2

Polinomio Factorizado

Ejemplo :

ab b 2)

Solución :

1. Factorizar : x 4

y2

Solución : 2 2

(x )

y

2

(x

2

y)(x

2

y)

x 2 10x

9

x

9

x

1

2.

Factorizar : x

10x 25

Ax 2

2 (x )(5) 5 2

3. Factorizar :

(x

5)2

Cy 2

Bxy

Dx Ey F

Regla : * Se descomponen en dos factores :

a

3

Axx 2 ; Cy 2 ; F

27

* Mediante tres aspas, se comprueban: Bxy, Dx, Ey.

Solución : a3

33

(a

3)(a 2

3a 9)

polinomio factorizado

a) Aspa Simple : Se aplica a trinomios, obteniéndose 2 factores binomios. Regla : se descomponen dos de los términos, en dos factores, luego se calcula la suma del producto en aspa, tal que sea igual al término no descompuesto del trinomio. Factorizar :

Ej. 10x 2

Factorizar : xy 3y 2 16x 14y

8

Solución :

Método de las Aspas

Ej.

x

se obtienen dos factores trinomios.

polinomio factorizado

III.

9x

b) Aspa Doble : Se aplica a polinomios de la forma :

Solución : x2

9)(x 1)

10x

polinomio factorizado

2

(x

x 2 10x 9

10x 2

xy 3 y 2 16x 14 y 8

5x

3y 1

2x

2 3

-y

2

-4

Comprobaciones : Aspa 1

-5xy + 6xy = xy

Aspa 2

-12y - 2y = -14y

Aspa 3

4x - 20x = -16x

luego, tendremos : (5x+3y+2)(2x-y-4)

c)

Aspa Doble Especial Generalmente, se aplica a polinomios de cuarto grado

Caso B : coeficiente principal posibles ceros :

de la forma :

Ax 4

divisores T. independiente divisores coeficient e principal

Bx 3 Cx 2 Dx E

Se obtienen dos factores trinomios de segundo grado.

Regla para factorizar :

Regla : *

Ax4 y E, luego se calcula la

Se descomponen :

a)

suma del producto en aspa. *

La suma obtenida se resta de Cx 2 .

*

La diferencia que resulta se descompone en dos

Se calcula los posibles ceros y se comprueba si alguno anula al polinomio, por ejemplo : Si se anula para :

factores para comprobarlos con : Bx 3 y Dx.

x=2 x = -3 x = 4/5

Ejemplo : Factorizar :

x4

5x 3 x4

Solución :

9x 2 14x 6

5x 3

x2 2

x2

9 x 2 14 x 6 4x

3

Ejemplos :

2

1. Factorizar : Solución :

x

3

*

2x 2 3 x 2

Aspa 1

2 que se resta de 9 x 2 , obteniéndose 4 x .

Aspa 2

x 2. 4 x

Aspa 3

4x . 2 + x . 3 = 11x

IV.

4x

3)(x 2

x

5x 3

x 3 6x 2 12x 7

Posibles ceros 1,

5x 2

x 2. x

(x-2) es factor (x+3) es factor (5x-4) es factor

Al polinomio dado, se le divide entre el factor o factores binomios obtenidos en el primer paso, el cociente de esta división es el otro factor del polinomio.

1

b)

Comprobación :

(x 2

1

2,

3,

6

divisores de 6

* Se comprueba que se anula para: x = 1 (x-1) es factor.. * Se divide por Ruffini al polinomio entre (x-1) : x-1 = 0 1

2)

1 1

Método de los Divisores Binomios o Evaluación Binóm ica Se aplica a polinomios de cualquier grado, generalmente con una sola variable, siempre que tengan por lo menos un factor lineal (primer grado). "Ceros" de un Polinomio Son los valores de la variable que anulan el polinomio.

x2

(x 1)(x 2 2. Factorizar : *

12

-7

1

-5

7

-5

7

0

- 5x + 7 factor faltante

5x 7)

6x3 7x2 6x 1

Posibles ceros (coeficiente principal 1,

Caso A : coeficiente principal = 1

(

divisores del término independiente

-6

* Finalmente tenemos :

Para obtener los posibles "ceros" de un polinomio, tendremos :

posibles ceros :

(coeficiente principal= 1).

1 , 2

1 , 3

1 6

Divisores de "1" ) Divisores de "6"

* Se comprueba que se anula para: 1/3. * Se divide por Ruffini entre : 3x - 1. * Finalmente, tenemos :

de 1) :

6

7

-6

1

2

3

-1

6

9

-3

0

2

3

-1

1 3

3

2x 2

Factorizar : x 4

3 x 1 (factor faltante) 2

3x 1) .

En este caso, no existen reglas fijas. Se aplica cuando las reglas anteriores no son fáciles de aplicar ; pero se puede recomendar lo siguiente : a) Si dos o más términos se repiten constantemente, se sugiere hacer cambio de variable. Ejemplo : Factorizar : 2) 2 (a c 1)

b

c 1) 2

Solución :

se elige la letra que se desee menos : a, b, c

reemplazando :

(x

2) 2

4x

4

11x

como :

Artificio

4 x 2b 2c 4

Sumamos y restamos :

2 2 4

4x b c x4

4 b4 c 8

4 x 2b2 c 4

4 x 2 b 2c 4

TCP

(x 2

2b 2c 4 )2 (2xbc 2)2

(x 2

2b 2c 4

2xbc 2)(x 2

2b 2c 4

2xbc 2 )

c) Si aparecen exponentes impares, procuramos formar suma o diferencia de cubos. Ejemplo : Factorizar : x 5

Hacemos : a+b+c = x

2x

para formar TCP, necesitamos :

ya factorizad o

(a b c 5 (a b

2

2 2 2 4 2 Tenemos : (x ) (2b c )

2 (x 2 )(2b 2c 4 )

Método de los Artificios

x2

4b 4 c 8

Solución :

tendremos : (3x 1)(2x IV.

b) Si aparecen exponentes pares trataremos de formar TCP. Ejemplo :

1) 2

(x x2

-

5 (x

2x 1 5 x

x(2x 11)

1) 5

x 1

Solución : * Como hay exponentes impares, buscamos suma o diferencia de cubos. 5 * Si a " x " le factorizan " x 2 " , aparece " x 3 " .

Artificio : sumamos y restamos x 2 . x5

x = a+b+c

x 1 x2

x2

x 2(x 3 1) (x 2

(a+b+c) [ 2(a+b+c)-11 ]

x 2 (x 1)(x 2 (x

2

x 1)(x

x 1)

x 1) (x 2 3

x

2

1)

x 1)

EJERCICIOS PROPUESTOS 01. Indicar el número de factores primos de : P(x; y) a) 2 d) 5

5 3

x y

a) x + 5 d) x + 4

2 7

x y

b) 3 e) 6

b) x + 7 e) x - 1

c) x + 3

09. Factorizar :

c) 4

F(x ; y) x 2 (x y)2 8 xy 2 (x y) 12 y 4 La suma de sus factores primos es :

02. Señalar un factor primo, al factorizar : F(x ; y) a) y d) x - y

x3y

x2y2

xy

a) 2x + y d) 4x + 2y

c) x 2

10. Factorizar :

x2

b) xy - 1 e) xy

a) xy 2 d)

x 2y

x6

x 2y 2

b)

x 3y

e)

y3

y4

xy 3

x 3y 3

a) -1 d) -6

c) y 4

F(x ; y) x 3 y 2x 2 y 2 xy 3 x 2 El factor primo que más se repite es : b) xy - 1 e) x - y

2xyy

y2

a) x + y d) x

2

y

c) (x

y)2

y 2 )2 (y 2 1)2

b) x - y

c) x + 1

e) y - 1

F(x ; y) (1 xy )2 (x Un factor primo es :

y)2

b) x - y e) 1 - x

F(x ) (2x 2 3 x)2 14(2x 2 Un factor primo es : b) 2x - 3 e) 2x + 3

4 xy

c) 2x + y

12. Factorizar :

a) 6x - 4 d) 3x + 7

b) 8x - 4 e) 4x - 3

2

c) 3x + 2

13. Factorizar :

a) x - 4 d) x - 2

b) x - 3 e) x + 1

c) x + 3

14. Factorizar : F(x ) x 2 (x 2 3)2 (3x 2 1)2 La suma de factores primos lineales es :

3 x) 45

c) 2x +5

08. Si el polinomio : F(x ) x 2 (2m 1)x (m 1)2 Es factoriable mediane un aspa simple (en los enteros), además : m Z

c) 5

P(x) x 5 21x 3 16x 2 108 x 144 e indicar el factor primo repetido.

07. Factorizar :

a) 2x - 1 d) 2x + 1

b) 3 e) 9

F(x ) 6 x 3 19 x 2 15 x La suma de sus factores primos es :

06. Factorizar :

a) x + y d) x - 2y

c) 6

F(x ) x 3 2x 2 5 x 6 La suma de coeficientes de uno de sus factores primos es : a) 1 d) 7

05. Factorizar : F(x ; y) (x 2 Un factor primo es :

b) -3 e) -2

11. Factorizar :

04. Factorizar :

a) xy + 1 d) x + y

c) 3x + 3y

F(x ) x 3 2x 2 5 x 6 El término independiente de uno de sus factores primos es :

03. Indicar un término de un factor primo de : R(x ; y)

b) 3x + y e) 2x + 3y

m 1 . Indicar un factor primo..

a) 4x + 1 d) 2x + 3

b) 4x + 3 e) 2x - 1

c) 2x

15. Indicar la suma de factores primos de : 2x 4 a) 5x + 6 d) 4x

7x

b) 4x - 1 e) 5x

3(x 3

x 2 1) c) 3x - 2

16. Dar la suma de los factores primos de : x(x - 4)(2x - 11) + 12x - 48 a) 4x + 7 d) 3x + 7

b) 3x - 7 e) 4x + 11

24. Factorizar :

F(a ; b) 2a4 b3 15a2b3 27b3

c) 4x - 11

Indicar el factor primo de mayor grado.

x 5 ax 3 a) x 2 d) x 2

d) 2a 2

bx 2 abx3 a 2 bx ab2

ab

b) x 3

ax b

ab

e) x 3

ax

c) x 3

ax b

b

a 3 (1 b) b 3 (1 a) ab(a 2 b) a

c) a a b

c) a + ab + b e) a

2

2 2

a b

b

b

(x + 1)(x + 2)(x + 3)(x + 4) - 3 e indicar que la suma de los términos lineales de sus factores primos. c) 8x

a) 5 d) 3

b) 1 e) 4

x)2 2 (x 2 x)

2x

b) 0 c) 1 e) Hay dos correctas

24

bx a 2x ab es : b) ax + b d) abx + 1

27. Uno de los factores de x 6

x 2 8x 16 es:

a) x 3

4

3 b) x

2x 4

c) x 2

2x 4

d) x 3

x 4

e) x

20. Cuántos factores lineales tiene :

x 5 8 x 4 18 x 3 7x 2

F(x) (x 2 x)3 (x 2

a) x - ab c) ab + x e) bx + a

2

b) 10x e) 12x

25. Factorizar :

26. Un factor de : ax 2

19. Factorizar :

a) 6x d) 20x

e) a 2 1

3

a) 4 d) -2

b)

ab b 2 2 2

c) 2a4 1

Indicar el valor numérico de un factor primo, para x = 2.

18. Dar un factor primo de :

a) a + b

b) b3

a) b

17. Dar un factor primo de :

3

x 4

28. Factorizar :

R(x ; y)

c) 2

x4

3y 2 (x 2

y 2) y 4

Indique la suma de factores primos.

21. Sea : a) 2 (x 2

2y 2 )

2 b) 2 (x

y 2)

Indique el número de factores primos :

c) 2 (x 2

y 2)

2 d) 2 (x

2y 2 )

a) 4 d) 1

e) 2 (x 2

xy y 2 )

R(x) 5 (x 2)2(x 7)3(x 2 3)4 (x 6)

b) 3 e) 11

c) 2

22. Indique el número de factores primos lineales de :

P(m) m 6 7 m 3 8 Indicar el término lineal de uno de los factores primos cuadráticos.

P(x ; y) x8y 3x7y 2x6y 6x5y a) 1 d) 4

b) 2 e) 48

c) 3

a) 4m d) 8m

23. Indicar un factor primo de :

F(x ; y) x3 x2 x2y y2 2xy a) x 2

y

d) xy

x2

e) x

2

x

2 b) x

y2 y

29. Factorizar :

y2

y

c) x

b) -m e) -4m

c) 3m

30. Al factorizar un polinomio en el conjunto de los números enteros, mediante el procedimiento del aspa simple, se realiza lo siguiente : y2

8x4

bx 2 (2 d )

2x 2

1

4x 2

d

Entonces un factor primo del polinomio es: a) 2x - 1 d) 2x + 3

b) 2x + 2 e) 2x + 4

a) 3 d) 7

(x 5)(x 7)(x 6)(x uno de los factores lineales es :

4) 504

b) x + 7 e) x - 2

x 1)2 34x (x 1) 179

Indique la suma de todos sus factores primos: a) 2(2x+3) c) 2(2x+1) e) 2(x+1)

b) 3(x+2) d) 3(2x+1)

(12 x 1)(6 x 1)(4 x 1)(3x 1) 5 b) 3x - 1

d) 3x + 1

e) 36x 2 15x 4

c) 2x +1

34. Hallar el producto de los coeficientes del factor primo de mayor término independiente del polinomio. P(x) a) 4 d) 12

8x

3

28 x

b) 5 e) 14

2

c) 8

c) 0

x5

3x 4

5x 3 15 x 2

4 x 12

Indique el binomio que no es factor. a) x - 2 d) x + 4

b) x + 2 c) x - 1 e) Todos son factores

37. Determinar el número de factores primos del siguiente polinomio : P(x )

x5 x4 b) 2 e) 5

2n 4

n3

n2 1 c) 5

b) En 1 c) En 4 e) No varía dicho número

m (m 2

P(m ; n)

mn 1) n (n2

a) m + n

b) m - n

d) mn + 1

d) m2

e) m 2

mn 1)

mn n2

n2 1

42. Un factor de : 2x 2 1 (4 x 3 y 6 x 2 y 2

4 xy 3

y4 )

es :

c) x 2

2 b) x

y2

2xy 1

y2 1

d) 1 2xy

2y 2

2y 2 1

e) 2xy

43. Al factorizar :

K

25a4 109a 2 36

uno de sus factores es :

36. Factorizar : P(x )

n6

40. ¿En cuánto disminuye el número de factores primos de: (3x - 1)(x - 3)(2x - 5) (6x + 1); si le restamos 20?

a) 1 2xy

b) 9 e) 97

c) 1

b) 2 e) 6

2x 7

35. Si se suman algebraicamente, los coeficientes y los términos constantes de los tres factores binomios, en los que puede descomponerse el polinomio : x 4 2x 3 76x 2 8x 320 , se obtiene : a) 14 d) 22

5x 1

41. Señale un factor primo de :

a) 12x + 1

a) 1 d) 4

n7

F(n)

a) En 2 d) En 3

33. Indique un factor primo de :

10 x 3 10 x 2

39. Hallar el número de términos de un factor primo en Q de :

a) 1 d) 4

(x 2

5x 4 b) 11 e) 2

c) x + 6

32. Factorizar :

A(x)

x5

P(x)

c) 2x + 5

31. Al factorizar :

a) x - 5 d) x + 3

38. Indicar la suma de coeficientes de un factor primo de :

2x 3

2x 2 x 1 c) 3

a) a + 3 d) 5a - 1

b) 5a - 3 e) 5a + 2

c) a - 3

44. Descomponer en factores :

x 3y a) b) c) d) e)

x 2y 2

x 2 yz

yz 3

(x-z)(z-y)(x+y)(x+z) (x-z)(x+z)(x+y)(y-z) (x+z)(x+y)(y-x)(z-y) (z-x)(y-z)(x-y)(x+z) N.A.

xyz 2

xz 3

y 2z 2

x3z

45. Descomponer en dos factores :

y)3

(x

51. Indique un divisor de :

3xy (1 x y) 1

R(x)

x10 1 x 2 (2x 4

(x

y 1)(x 2

2xy

y2

x

y 1)

2 a) x

x 1

b) x 3

x2 1

b) (x

y 1)(x 2

2xy

y2

x

y 1)

2 c) x

x 2

d) x 2

x

a)

(x

y 1)(x 2

xy

d) (x

y 1)(x 2

2xy

y2

x

y 3)

e)

y 1)(x 2

2xy

y2

x

y 3)

c)

(x

y2

x

e) x

y 1)

a) a 2

b2

c) a

2

b

e) a 2

c2

d2

x5

2cd(a 2

b2 )

(x 2 1)2 ; para : x = -1.

a) -2 d) 1

a) 71 d) 17

A (x; y ; z) (x 2y)3 (y 3z)3 (3 z y x)3

b) 4 e) 5

c) 1

48. Factorizar : (x 1)2 (x 2

2x

9) 5(x 1)(x 2

2x

2) 1

y 1)(x

y 1) (y 1)2 (1 x)

Indique el número de factores primos. a) 2 d) 4

b) 1 e) 5

c) 3

50. Factorizar el polinomio : P(x ) x 5 x 4 2x 2 1 ; y dar como respuesta la suma de coeficientes del factor de grado 3. a) 0 d) 3

b) 1 e) 4

2x 5

b) x 3

d) x 5

e) x 6

3x 4

3x 2

3x 1

c) x 4

55. Factorice en el campo de los números reales:

32 (a 2 5)5 (a 2 9)5 (a 2 1)5

Señale el número de factores primos : a) 10 d) 6

49. Factorizar :

x7

a) x

P(a)

b) x + 18 c) x + 7 e) Hay 2 correctas

2 (x 2 1) x 4

b) 7 c) 8 e) Más de una es correcta

P(x )

Indicando un factor primo.

x 2 (x 1) y (x

c) 0

54. Señale Ud. el término de mayor grado de un factor primo del polinomio :

Indique el número de factores primos obtenidos.

a) x + 11 d) x + 2

b) -1 e) 2

(x 2 1)7

47. Factorizar :

R(x )

1

b2 c 2 d 2

b c d

a) 2 d) 3

2

53. Dar la suma de los coeficientes del factor primo de menor grado en :

b) a + 2b + c + 2d 2 d) a

c d

x

2

52. Indicar el valor numérico que forma uno de los factores primos de :

46. Factorizar : (a b)2(c d)2 2ab(c d)2 e indicar la suma de los factores :

3

1)

b) 12 e) 7

c) 8

56. Factorizar e indicar el número de factores primos racionales : x10

P(x ) a) 1 d) 4

2x 5

b) 2 e) 5

x2 1 c) 3

57. Señale la suma de coeficientes de un factor primo de :

F(x )

c) 2 a) 8 d) 4

x7

b) 6 e) 3

2x 5

2x 3 c) 5

1

58. El polinomio : P(x) (x 1)(x

60. Proporcione uno de los factores primos de:

2

2)(x

3

3) 2 (2x

3

3x 3)

(a 5b5

Luego de factorizarlo toma la forma :

x n (x c)n (x n

ax a)

Calcular : a + n. a) -4 d) 3

b) -1 e) 5

c) 4

59. Señale un factor de : (x 4

3x 2 1)2

(2x 2

3

b) x 2

x 1

2

4

a) x 4

x2

4

x

2

2

2x 2

c) x

e) x

d) x

2x 2

3)2

2

abc (a 5

M(a ; b ; c)

b 5c 5

b5

c5 )

a 5c 5 ) a 2b 2c 2

a) a 2

bc

b) a3

3 d) a

bc

e) a - bc

bc

a4 b4c 4 c) bc a4

Claves 01.

b

31.

b

02.

b

32.

c

03.

b

33.

c

04.

d

34.

e

05.

c

35.

b

06.

e

36.

d

07.

e

37.

b

08.

d

38.

e

09.

d

39.

c

10.

e

40.

b

11.

b

41.

b

12.

b

42.

a

13.

d

43.

b

14.

c

44.

b

15.

c

45.

c

16.

b

46.

a

17.

a

47.

d

18.

c

48.

d

19.

b

49.

c

20.

d

50.

d

21.

a

51.

b

22.

c

52.

d

23.

e

53.

b

24.

d

54.

b

25.

e

55.

d

26.

b

56.

c

27.

d

57.

e

28.

d

58.

b

29.

b

59.

d

30.

a

60.

c

Capítulo

6

MCD Y MCM DEPOLINOMIOS FRACCIONES ALGEBRAICAS

Regla para calcular el MCM y MCD de Polinomios :

Simplificación de Fracción Algebraica

1. 2.

Para poder simplificar, el numerador y denominador deben estar factorizados para luego cancelar los factores que presenten en común.

3.

Se factorizan los polinomios dados. El MCD estará formado por la multiplicación de todos los factores primos comunes de los polinomios dados, considerados con su menor exponente. El MCM está formado por la multiplicación de factores primos no comunes y comunes, a los polinomios dados, considerados con su mayor exponente.

Ejemplo : Simplificar :

Ejemplo : Hallar el MCD y MCM de los polinomios: x3

P(x)

x2

x 1

Q(x)

x3

x2

Q(x)

x2 x

2x 2x

x (x 2)(x 1)

MCM[P(x);Q(x)] (x 1)2(x 1)x(x 2) Propiedad :

A B

A Es toda expresión de la forma donde por lo menos B "B" debe ser literal.

32 2x 15

x

5

x

3

=

(x 3)(x 3) x 3 = (x 5)(x 3) x 5

Adición y/o Sustracción : En este caso, es necesario dar común denominador (MCM de los denominadores), salvo que las fracciones sean homogéneas (denominadores iguales). Así tenemos :

A x

Son fracciones algebraicas x 1 2 x3 1 , , x 1 x x2 pero : no son fracciones algebraicas

B x

C x

A B C x

B. Fracciones Heterogéneas : Ejemplo :

A m

Ejemplo :

x 2 , 7 5

2

Ejemplo :

FRACCIÓN ALGEBRAICA

*

2x 15

A. Fracciones Homogéneas :

Dados los polinomios A y B.

MCD ( A , B) . MCM (A , B)

9

Operaciones con Fracciones I.

MCD [P(x);Q(x)] x 1

x

2

Resolución :

Factorizando : P(x ) (x 1)2 (x 1) "

x2

B n

C p

Anp Bmp Cmn mnp

C. Regla Práctica (para 2 fracciones): A B

C D

AD BC BD

II.

Multiplica ción : En este caso, se multiplica numeradores entre sí, de igual manera los denominadores.

Ejemplo : III.

A C B D

Tr ans f or m a ci ó n Pa r ci a l e s

A C B D

División de Fracciones : En este caso, se invierte la segunda fracción y luego se ejecuta como una multiplicación. A B

C D

Importante : generalmente es conveniente simplificar las fracciones antes, y después operar fracciones.

A B

D C

A D B C

Ejemplo : *

Efectuar :

1 x 1 *

AD BC

Fr a cci o nes

en

Fr acci one s

Este es un proceso inverso a la adición o sustracción de fracciones. Es decir una fracción se transforma en la adición o sustracción de fracciones que le dieron origen, veamos :

ó A B C D

de

1 x 1

2x x

2

1

Transformar a fracciones parciales :

2x x

2

1

1

1

x 1

x 1

EJERCICIOS PROPUESTOS 01. Hallar el MCD de los polinomios : M(x) N(x)

2

(x

3

6) (x 7) (x 3

4

9)

2

3

(x 10) (x 7) (x

a) a + 1

b) a - 1

d) (a 1)2

e) 1

c) (a 1)2

6)

07. Luego de efectuar : a) (x-7)(x+6)

b) x + 9 2

c) x + 10 d) (x 7) (x e) (x+10)(x+9)(x+6)(x-7)

2

6)

02. Indicar el MCM de los polinomios : 3

P(x)

(x

3) (x

F(x )

(x 1)2 (x

6)(x 1) 3)3

c)

6)2 (x

d) (x 1)4 (x e)

3)2 (x

x2

xy 6 y 2

F(x ; y)

x2

xy

a) x + 2y d) x + y

6)

x3 3

c) x - 2y

x 1

P(x) x 6x 11x 6 para : x = 4, es : b) 1 e) 4

c) 5

05. ¿Cuántos factores cuadráticos presenta el MCM de los polinomios? P(x)

x

3

2x

2

x3

5x 2

R(x)

x3

6 x 2 12x 8

8x

b) 1 e) 4

4

3x 2 x

2

3x 4

equivale a :

, entoces ; m - n es igual a :

a) -1 d) -2

b) 1 e) -3

c) 2

10. Efectuar :

a) 0,25 x 2 d) 0,5x

b) 0,25x e) 0,625x

a) a c) 2

c) 0,125x

11. Efectuar :

a

ellos es (a 2 1)2 y la división entre el MCD y el MCM es (a 1) .

n x

4

06. Calcular el MCD de dos polinomios, si el producto de

2

e) (x 1)3

4x 8

Q(x)

a) 0 d) 3

d) (x 1)3

c) x 3

x 1 2x 2 . x x2 1 Indicar la octava parte del numerador simplificado.

2

a) 25 d) 3

b) 64

m x 1

2y 2 b) x - 3y e) x - y

x2

c) x + 3

a) 64 x 3

09. La fraccción

04. El valor numérico del MCD de los polinomios : F(x )

b) x - 3 e) 2x - 3

3)

03. Hallar el MCD de los polinomios : P(x ; y)

x

x 1 x 1 4 2 x 1 x 1 x 1 Indicar el cubo del denominador.

3)3

(x 1)2 (x

2

08. Efectuar :

3)3 (x 6)

(x 1)2 (x

2x

x 1 x el numerador obtenido, es : a) x 2 3 d) 2x + 3

4

a) (x-1)(x+3)(x+6) b) (x 1)4 (x

1 2

d)

1 a b

b b

2

a b a2

b) b

a b

e)

ab 2

c) ab

b a

12. Al simplificar : 1 a Calcular : m4

1 (ab)2 obtenemos (ma)(nb) b a b

n4 , si : m, n

Z.

a) 17 d) 626

b) 82 e) 257

c) 2

13. Simplificar las fracciones :

x2

4

x2

;

2

b) 2x + 1 e) 2x + 1

a

a

b

a

b

a b

a) a

b) b

a d) b

e) 1

x y

16. Efectuando :

n

x ; tenemos : y

1 x

y

e) 1

x y

y x

c) 1

c) n - 1

6x 8

x

2

4

2

b) -5x e) -3x

2x

y

B

10x 3

a) 3x 2

nab 24 b 2 3ab 4 b 2 m2

c) 120

2

9x 2 17x 6 b) 2 x 2

2x 1

2

x 3 x

d) x

2

x

3

x 1

3

2

P(x)

x3

4x2

Q(x)

x3

cx

;

ax

ay x ax x

2

b

d

b) 8 e) 0

x2

y

2x 3

2x 2 2x x 1 d) 1 a)

2

ax

Tal que, el MCD (P, Q) = (x-1)(x+3), entonces la suma de coeficientes del polinomio MCM (P, Q), es :

c) -8x

4

2xy

2a

2

c) 6

23. Efectuar :

18. Simplificar las fracciones : 3

ma 2

x 3 3x 2 3x 9

a) 9 d) 4

x x xn n x nx señalar un término en el denominador.

x

c) 11

22. Si : P y Q son dos polinomios factorizables definidos por :

1

2

4

2x 4

e) x

17. Simplificar :

a) -7x d) 11x

5c 2 ) .

b) 180 e) 100

A

c) 3x 3x

b) n - 2 e) 1

x

x 1

21. Hallar el M.C.D. de los siguientes polinomios :

1

2

x 1

es independiente de sus variables, entonces n2 equivale a :

1 n 2 obtenemos en el numerador . a) n2 d) n

c

20. Si la fracción : P(a; b)

1

1 n

b

b) 7 e) 2

a) 210 d) 144

b)

obtenemos :

1

1

1

d) 1

ax

a) 3 d) 14

c) ab

1

x

2

1 x 2

c)

x3

Calcular : a (3b 2

15. Simplificando :

a) x - y

e) 2x

c) 2x - 1

; obtenemos :

b

d) x

19. Al descomponer

14. Simplificando : 2

b) 4x

x 2

x 2x x 2 4 x 4 e indicar la suma de los denominadores. a) 2x - 2 d) 2x + 2

a) 3x

2

xy

e indicar la diferencia de los denominadores.

x 1

b) 2 e) 0

x2 2x 2

4 5x

2

c) x

24. Resolver :

30. Efectuar :

x 1 x 1

f(x)

a) x - 1 d) 1

x2 1

x 1 x 1

2x 2

b) x + 1 e) 0

25. La fracción :

2 a) 0

c) x

d)

7x 1 1 5x 6x 2

; se obtuvo sumando las

c) 4

x xy

a) 1 d) 3

c) a

3

2

2b2

d)

y z 1 yz xz xyz

ab a b

c

b) 1

a) ac d) abc

ax 2 bx c x 1

Hallar : (A

d) a 2

b2 2ab

B)C . b) 64 e) 16

c) 27

33. Si : a + b + c +d = 0. Hallar :

cd d

abc abd acd bcd a3

c) -1

ac bd

a) 1 d) 1/3

b3

c3

b) 2 e) 1/2

d3 c) 3

34. La expresión :

b) ab e) 2ac

a

x3

b) 9,5 e) 12,5

1

1 1

1 m

equivale a :

c) bc

bx 2 cx x 2

1

1

(ab)3 (bc )3 2 (ac)3 3abc (a b c)

x3

2ab

2 (2x 2 11x) 13 (x 5)[x (x 5) 1]

Bx C x (x 5) 1

a) 1 d) 9

cx 2 ax x 3

b

se obtiene un polinomio de segundo grado. Indicar la suma de coeficientes de dicho polinomio. a) 8,5 d) 11,5

2

x 5

29. Al realizar : x3

es :

b2

A

28. Si : ab + bc + ac = 0. Calcular : K

2ab 2b 2

32. Si :

a c , la expresión : b d

e)

4 b4

b) a 2

S

ab cd

a b c a b c

b2 ab

resulta : a) 0

c) 1

2ab

2ab 2b

3

c) -3

(a c)(b d) a b c d

e)

(1 ac )(1 bc) (c a)(b c)

31. La expresión simplificada de :

e) a 2

b) -1 e) 2

27. Conociendo que

abc a b c

a) a 2

26. Sabiendo que : x + y + z = 1. Calcular :

M

b) -1

a2

b) -20 e) -4

3

(1 ab)(1 bc) (b a)(c b)

a4

A B fracciones : . ; 1 3 x 1 2x Calcular : (A.B). a) 20 d) -5

(1 ab)(1 ac ) (a b)(c a)

R

c) 10,5

a)

m 2 m 1

b)

m 1 m 2

d)

2m 1 3m 2

e)

3m 2 2m 1

c)

3m 1 m 2

35. Para qué valor de "b" se cumple que : ab (x 2 ab (x

2

y 2 ) xy (a 2 2

y ) xy (a

2

b 2) b2)

1; y

0

a) -a d) a

b) 0 e) 2

c) 1

36. Efectuar : 2

2

4x 2xy y 3 8x y 2y ( 3 )(1 ) 2x y 8x y3

Z

b (a c)2

c (a

b)2

a (b c)2

b (a c)2

c (a b)2

Tomar el valor de 11.

8 xy

2

a (b c)2

a) 6,5 d) 1,33

3

b) 7,2 e)

c) 0,3

41. Si el MCD de los polinomios : a) 2 d) 0

b) 3 e) -1

c) 1

x5 1

1 1

a) x 1

b) x 2

d) x 4

e) x 5

a) 2x + 1 d) 2x + 4

x2 1 x3 1 x4 1 x 1 x x

x3

M

a a

1

b b

1

;N

a

1

b

1

a

2

b

2

1

2 a) n

b) n

2 d) 2n

e) 2n

d)

(a 2

b2) ab

e)

2

a )

(a

2

b2 )

B

1

a3

b3

c3

1 a3

b3

c3

Calcular :

b33

c 33 a) 1 d) -2

c 33

a 33

a 33

c 33

b33

b) -1 e) 3

2

2

a (b c) b (a c) c (a

b)

. Calcular : 1

A . B

a

a)

a b

b)

b a

d)

1 ab

e)

a b ab

c) ab

44. Si : ax + by + cz + abcxyz = 0. Calcular :

(ax 1) (by 1) (cz 1) (ax 1)(by 1)(cz 1)

c) 2

40. A partir de la relación : 2

1

1 b

1

n 2

b 1

b a

1

c)

ba

c)

2

a b b a

b 33

nz 2 1 (z y)(z x )

;

1 a

39. Si :

a 33

1

a b

1 (b

ny 2 1 (y x )(y z)

43. Sabiendo que :

Entonces MN, es igual a :

b)

c) 2x + 3

c) x 3

1

(a b) (a b)

nx 2 1 (x y)(x z)

K

A

a)

b) 2x + 2 e) 2x + 5

42. Efectuar :

38. Si : 2

ax 2 18

Q(x) x 3 bx 12 es de segundo grado, encontrar la suma de los factores no comunes.

37. Simplfiicar :

2

x3

P(x)

Mabc

Determinar el valor de "M" que hace que la fracción :

a) 0

b) 1

d) abc

1 e) abc

c) -1

45. Si se cumple : a

b

c

a 1

b 1

c 1

a

b c

ab a 1 b 1

a) 3 d) 9

1 2

(y

bc

b 1 c 1

ac c 1 a 1

b) 27 e) 81

c) 1

x

z)

y

(z x )

a

x

a2

b

2

b

; y

b2

(a 2

b4 b 2 )2

2

c

2

c

; z

c2

c4

(b2

1

1

y z

z x

x y

;

b) 16 e) 6

c) 2

50. Sabiendo que :

b2

b4

1

a) 8 d) 4

2

a

c2

a b c

2

a2

Calcular :

y además : a4

4

y)2

(x

z.

46. Si : 2

1 2

Calcular : S

obtener E 2 a partir de : E

1

49. Si :

a4

c 2 )2

c4

(a 2

4

c 2 )2

a2 b c

a) 0 d) 2

b c

c a

b2 c a

a

1

b

c2 a b

b) 1 e) -2

c) -1

51. Si : Calcule : x

2

y

a) 3 d) 9

2

a b c

2

z .

b) 5 e) 12

c) 7

a

3

1

3

c3

b

M

47. Calcular el valor de : E

(x

y)3

(y z)3

y3

z 3 (x

x3

(z x)3 z)3

y

a) 1 d) 4

a

1 bc

1 b ac

1 c ab

b) -2 e) -8

c) 3

52. Si :

sabiendo que :

x

216 215 214

y

214

215

216

z

215

214

216

a) 3 d) 3/4

(a 2

b) -3/2 e) 2

P(x) (x a)(x b)(x c)

x x a

c) -3/4

x x

x x c

b

2

a P(a)

b) -1 e) 2

2

b P(b)

P

a) 1 d) 8 x d

calcular :

M

b2) ab

(b 2

c 2)

(a 2

bc

c 2)

2

ac

Calcular :

48. Si : a, b, c, son números diferentes y :

a) -2 d) 1

4

Hallar :

2

3

3

(ab)

(bc)

b6

c6) 3

(ac )

b) 2 e) 16

c) 4

53. Simplificar : M

c P(c) c) 0

(a b c)6 (a 6

a) d)

1

1

(p q)3 p 4

1

q4

p q pq p q p 2q 2

b) e)

1

1

(p q)4 p 3

2

q3

q

p 4 4

p q

p q p 2q 2

2

c)

1

1

(p q)5 p 2

q2

q p p3q3

54. Si :

58. Si : x + y + z = 0.

2 4

2 4

b x 2

2 2

a y 2

a b x Calcular :

y

a b

2

y

a) 1 d) 1/4

2

Reducir :

1

R

b4 x 6

a4 y 6

b2x 4

a 2y 4

b) 1/2 e) 3/4

c) 3/2

a) 1 d) 3

(ay

bx)

bz)4

(az

bx )4

4

(ax

bz)4

(az by)

b) a+b+c e) -abc

2000

a

b

c

b c

c a

a b c

(b c)2

(c a)2

(a b)2

1 x

k 1

0

b

b) 0 e) 2

2k

E

a

Indicando : E a) 1

c) -1

c) abc

1

1 2k 1

1 x2

2000

1. b) -1

d) -x

2 2000

e) x

c) x

2000

60. Reducir : (a b)(z x) (a c)(x y)

(b c)(z x) (a c)(y z)

1

1

a x 1 a x

Reducir : (a b)(y

2

z)

(b c)(x

a) c) 0 d)

57. Si : a + b + c = 0 Señale la suma de coeficientes de los 4 términos obtenidos al reducir : a11 b11 c11 11abc (ab ac bc ) b) 2 e) 5

x 2n 1 (a x)2n x 2n 1 ... (a x)2n ...

y)

(a c)(z x) b) xyz e) N.A.

x2 (a x)3 x2 (a x)3

x (a x)2 x (a x)2

2

2

a) 1 d) 4

(ay

4

59. Reducir :

56. Si :

a) abc d) 1

by)4

a) 1 d) 2abc

55. Sabiendo que :

Hallar :

(ax

c) 3

a 2x a a a x

b) e)

a 2x a x a x

c)

a

a 2x

Claves 01.

d

31.

c

02.

b

32.

e

03.

c

33.

d

04.

d

34.

e

05.

e

35.

b

06.

a

36.

a

07.

e

37.

e

08.

e

38.

e

09.

a

39.

d

10.

b

40.

b

11.

a

41.

e

12.

c

42.

b

13.

d

43.

a

14.

e

44.

c

15.

c

45.

d

16.

d

46.

b

17.

e

47.

c

18.

d

48.

c

19.

e

49.

c

20.

b

50.

a

21.

b

51.

b

22.

e

52.

b

23.

d

53.

b

24.

d

54.

c

25.

b

55.

b

26.

c

56.

b

27.

a

57.

a

28.

a

58.

b

29.

c

59.

c

30.

c

60.

c

Capítulo

7

TEOREMA DEL BINOMIO

Trata del desarrollo o expansión de : (x a)n para "n" entero y positivo. Previamente estudiaremos algunos conceptos básicos necesarios para este capítulo.

S em i f act o ri a l Se representa por : N!! y su definición depende, si "N" es par o impar.

Fact o ri a l El factorial de un número "n" (entero y positivo), es el producto de multiplicar todos los números consecutivos desde la unidad hasta el número "n". Nota ción

2n (par )

(2n)! ! 2 4 6 ... 2n (2n)! ! 2n (n! )

N

n!

2n 1 (impar )

(2n 1)! !

factorial de "n"

n

(2n 1)! !

1 3 5 ... (2n 1) (2 n)! 2n n!

Observación :

Por definición : n! 1 2 3 Ej.

N

........ n

(n

n!! (n!)!

2) Ej.

* 3!

1 2 3

6

* 6!

1 2 3 4 5 6

720

semifactorial de "n". factorial de factorial de "n"

(3!)! = 6! = 72 3!! = 1 3 = 6

Definiciones :

ANÁLISIS COMBINATORIO

Factorial de cero

0!

1

PERMUTACIONES

Factorial de la unidad

1!

1

Permutar "n" elementos es formar grupos de "n" elementos cada uno, tal que un grupo se diferencia del otro por el orden :

Propiedad n!

(n 1)! n

Ej.

Pemutar : a, b, c (3 elementos)

Formando grupos a b c b a c c a b

78!

Ej.

80!

1 2 3 ....... 78 79 80

a c b b c a c b a

# de permutas = 6

79!

80!

79! 80

Número de Permutaciones

80!

78! 79 80

Se representa por : Pn y se obtiene por la siguiente fórmula: Pn

Igualdad de Factorial : I.

Si :

a!

1

a

0óa 1

II.

Si :

a!

b!

a

b (a , b 0 , 1)

Ej.

P3

3! 6

n!

VARIAC IONES

3.

Formar variaciones con "n" elementos tomados de "k" en "k". Es formar grupos de "k" elementos cada uno, de tal manera que un grupo se diferencia del otro en el orden, o en algún elemento. Ej.

C nk 4.

C nk

ab ba

ac ca

bc cb

# de variaciones = 6

El número de Variaciones se representa por :

V23

3! (3 2)! 2!

*

C nk

n n Ck k

*

C nk

n k 1 n Ck k

*

C nk

n C kn n k

1 1

1

1

Esta fórmula atribuida incorrectamente a Newton nos n permite obtener el desarrollo de (x a) , siendo "n" entero y positivo. (El aporte de Newton fue el desarrollo cuando "n" es negativo y/o fraccionario).

6

Fórmula :

COMBINACIONES Formar combinaciones con "n" elementos tomados de "k" en "k". Es formar grupos de "k" elementos cada uno, tal que un grupo se diferencia del otro por lo menos en un elemento.

(x a)n

Ej.

Ej. Formar combinaciones con : a, b, c, d, de 2 en 2. Tendremos :

ab bc

1 1

FÓRMULA DEL TEOREMA DEL BINOMIO

n! (n k)!

Vkn

Fórmula :

n Vk

C nk

1

Degradación de Combinatorios

: Formar variaciones con : a, b, c, de 2 en 2.

Tendremos :

Ej.

Suma de Combinatorios

ac bd

ad cd

(x a)4

(x

a)4

Cn0 xn

C1n xn 1a Cn2 xn

C 04 x 4

x4

2 2

a

... Cnn an

C14 x 3a C 24 x 2a 2 C 34 xa 3

4 x 3 a 6 x 2a 2

4 xa 3

C 44 a 4

a4

n Observaciones ervac er r v ac rv aci c n del desarrollo de (x a)

# de combinaciones = 6

1.

Número Combinatorio

El número de términos del desarrollo, es el exponente del binomio aumentado en uno. Es decir : # términos = n+1

El número de combinaciones formadas se denominan número combinatorio, se representa por : C nk C nk

Fórmula :

Ej.

C 42

(4

n! (n k )! k!

4! 2)! 2!

2 2 2

6

Propiedades del Número Combinatorio 1. 2.

C0n

Cnn

1

1

C n1

Cnn

k

Si el binomio es homogéneo, el desar rollo será homogéneo del mismo grado.

3.

Si los coeficientes del binomio son iguales, los coeficientes de los términos equidistantes de los extremos, son iguales.

4.

Recordando que la suma de coeficientes se obtiene para x = a = 1, tendremos : C0n

C1n

C n2

... C nn

2n

n

Combinatorios Complementarios

Cnk

2.

FÓRMULA DEL TÉRMINO GENERAL Se utiliza para obtener un término cualquiera del desarrollo en función del lugar que ocupa. Se representa por :

Tk

1

II.

n Fórmula : En (x a)

Tk

C kn

1

En donde :

x

(1 x)n

n k k

n k+1 x, a

Fórmula para :

a

exponente del binomio

-1 < x < 1 ; x

(1 x )n

lugar del término términos del binomio III.

Ej.

n : negativo y/o fraccionario 0

(n0 ) (1n ) x (2n ) x 2 (3n ) x 3

...

Número de términos de :

(a1 a 2 a 3 .... a k )n n : entero y positivo..

Halle el término de lugar 40 en el desarrollo de: (x 2

y 3 )60

# de términos

tendremos :

T39

1

T40

60 2 60 C39 (x )

39

( y 3 )39

60 42 117 C 39 x y

OTRAS DEFINICIONES Y FÓRMULAS I.

Coeficiente Binónico : Se representa por ( n ) ; k

n R; k Z siendo su desarrollo : (n) k

n (n 1)(n 2)......[ n (k 1)] k!

Observaciones ; * Si n Z : ( n ) k

n * ( ) 1 0

n

Ck

(n k 1)! n! (k 1)!

IV.

En : (a1 a 2

a 3 .... a k )n

n : entero y positivo..

Coeficiente de a l a 2 a 3 .... a k

n! ! ! ! .... !

EJERCICIOS PROPUESTOS 01. Reducir : 6! 5! 5! 4!

M

a) 1 c)

b) 2

35 6

a) b) c) d) e)

0

c)

35 3

Un Un Un Un Un

número primo. cuadrado perfecto. número impar. número par. múltiplo de 4.

08. Determinar el término de lugar 10 en la expansión de:

1 8

e)

02. Calcular "x", si :

(3x 4)(3x 4)! (3x 6)! (3x 5)! (3x 4)!

72!

12

1 3x

27 x 5

a) 220x 5

b) 220x 7

6 d) 330x

e) 320x 6

c) 220x 6

09. Para qué valor de "n" en el tercer término del desarrollo a) 12 d) 21

b) 30 e) 18

c) 22

x! 2(x 1)! x x! (x 1)!

x! 23

b) 4 e) 7

c) 5

x 8

C3

220 c) 21

17 9

b) 7 e) 6

c) 8

06. Determinar "x" que verifica la ecuación : 1

b) 15 e) 20

desarrollo de (x

c) 10

n

n 1

2

2

3. Al simplificar, se obtiene siempre.

k 2)n es 80 x .

b) 9 e) 7

c) 6

12. Hallar el lugar que ocupa un término del desarrollo de: (x 3

2x

a) 9 d) 7

2 13

)

que tiene como parte literal a x14 . b) 5 e) 2

c) 6

13. Calcular el término independiente del desarrollo de : (x 2

c) 11

07. En la suma combinatoria :

donde : n N, n

0,5x 1)n el onceavo término es de grado 20.

x C7

b) 10 e) 13

S

(x 2

a) 5 d) 10

22 x C x 8 8

c) 7

11. Calcular (n + k), si se sabe que el cuarto término del

b) 18 e) 20

(x! )2 2x 1 C (2x)! x 1

a) 8 d) 12

b) 6 e) 18

a) 5 d) 25

05. Resolver :

a) 5 d) 9

a) 5 d) 9

10. Calcular "n", si en el desarrollo de :

04. Calcular "x" que verifique :

a) 17 d) 23

2 x17 )n el coeficiente es igual al exponente

de x :

03. Resolver :

a) 3 d) 6

de (x 1

a) 297 d) 354

b) 384 e) 374

5

x

3 13

)

c) 286

14. Al desarrollar (5 x17 y15 )n la suma de todos los exponentes de "x" e "y" es "n" veces la suma d coeficientes, hallar "n". a) 3 d) 6

b) 4 e) 7

c) 5

15. El producto de las sumas de coeficientes de los n 1 (4 x 5 y)n 2 desarrollos de : (x 6 y 4) ; es 3n 7 . Halle el número de términos del desarrollo 2 y)n 3 . de: (9x a) 6 d) 9

b) 7 e) 10

c) 8

23. ¿Qué lugar ocupa el término que contiene x 29 en el 2 1 22 desarrollo de (2x 3x ) ?

a) 5to. d) 4to.

B (x)

b) 36 e) 60

a) 8 d) 6

9n

3 . 6 . 9 . .... . (3n 3).(3n) b) 18 e) 36

12

n!

c) 24

18. La suma de "n" y el menor valor de "k", que satisfacen las siguientes condiciones : n! = 720 y

b) 6 e) 7

19. Determinar "a" y "b" en la igualdad :

a) a = 7, b = 3 c) a = 4, b = 3 e) a = 5, b= 6

b) 7 e) 10

(3! )2

b) a = 8, b = 9 d) a = 2, b = 1

25. El término independiente de "x", en : 2 1 9 ( x2 ) es : 5 2x a) 0,018 d) 0,001

a) 10 d) 40

a) 6 d) 4

(n! 1) (n! 5)

b) 3 e) 5

a) 13 380 d) 13 440

c) 2

d)

36 x 2y 33

b)

24 x 3 y 2

e)

12xy 2

c) 24 x 3 y 2

22. Proporcionar el coeficiente del término de grado 7 en el desarrollo de (x 7 a) 21 d) 70

3

2 )5

b) 20 e) 50

c) 30

6 4 y)10 , el coeficiente de x y

b) 13 450 e) 13 455

c) 13 460

4

x y a) 13 d) 21

1

100

xy 4

b) 14 c) 19 e) Es imposible determinarlo.

29. Calcular "n", si al desarrollar :

y 3 )12 .

a) 36 x 2 y11

c) 0,084

28. Indicar el lugar que ocupa el término que sólo depende de "x" :

1 (n! 5)

21. Determinar el penúltimo término en el desarrollo de : (3x 2

b) 0,002 e) 0,025

27. En el desarrollo de (2x es :

20. Calcular "n" en la ecuación :

n! 5 22

c) 9

( 2 c) 11

a! . b! 4

n

26. Deteminar el término racional en el desarrollo de :

n 2 = 56 es : k

a) 8 d) 9

y2 x

3x 3

existe un término cuyos exponentes de "x" é "y" son respectivamente 5 y 8. Halle el número de términos del desarrollo.

c) 30

17. Resolver :

a) 12 d) 8

c) 8vo.

24. Si en el desarrollo de :

16. Si : (x + 1)! - x! = 18. El valor de : (x+1)! + x! es : a) 24 d) 54

b) 6to. e) 12vo.

x

b) 35 e) 14

7 7

) . c) 42

(x 6 1)4 . (x 4 términos. a) 10 d) 20

x 2 1)2n (x 2 1)2n , se obtiene 25

b) 18 e) 12

c) 8

n 30. Dos términos consecutivos del desarrollo de (x n) tienen igual coeficiente; luego estos términos son :

a) b) c) d) e)

Primero y segundo. Segundo y tercero. Tercero y cuarto. Antepenúltimo y penúltimo. Penúltimo y último.

31. ¿Cuántos términos irracionales presenta el desarrollo de :

4

x

3

x

48

a) 44 d) 42

n

? c) 34

3 x

2x 3

b) 21 e) 27

100

c) 24

b) 10 e) 28

34. Calcular : a + b, si :

a) 5 d) 8

1

b) 6 e) 9

35. Determinar el valor de "m" en la expresión :

a) 256 d) 27

b) 3125 e) 7776

1 9

c) 4

1 1

a) 40 d) 50

C nk

n k 2 n Ck n 1

1 1

30 C13

37. Sabiendo que : Cm n m n

1 Cm n 1 m n x

Calcular el valor de "m-n", siendo : x a) 1 d) x

b) 2 e) 3x

n k

a) 2n

1

n2

n 2

b) 2n

1

n2

n 2

c) 2n

1

n2

n 2

d) 2n

1

n2

n 2

n 1

2

n 2

n

39. Calcular "n", si n Z

en :

F(x ; y)

y6

x6

x4

y4

n

b) 2 e) 10

c) 3

independiente del desarrollo. a) 219

23 b) 2

d) 225

e) 221

c) 243

B(x ; y) (x 2 y n )2n si dicho término central es de grado "n".

b) 44 c) 47 e) Hay 2 correctas

C nm 11 m n x

n

41. Hallar el término central del desarrollo de :

36. Calcular "n+k", en : C nk

b)n

40. En el desarrollo de : (2 3x 2 )n , el coeficiente de x 24 es 4 veces el coeficiente de x 22 . Calcular el término

c) 7

(2m)!

(a

k 3

a) 1 d) 5

((b! )! )720

2m. m! . 1 . 3 . 5 ....( 2m 1)

b

para que en el desarrollo de dicha potencia dos términos consecutivos del mismo sean independientes de "x" e "y" respectivamente.

c) 15

(30a!. 24 a! )a

k k

Calcular : 2

e) 2

33. El desarrollo de (a b c d e)n , posee 14 términos n 1 más que el desarrollo de (a b c d) . Calcular : n 1. Cn 1

an

n! k! (n k ) !

n k

32. Cuántos términos fraccionarios hay en el desarrollo de:

a) 6 d) 21

n k

k 0

b) 32 e) 26

a) 18 d) 25

38. Si :

c) 4

0.

a) 10 x

6 9

b) 20 x 6 y 9

6 5

6 4

y

d) 30 x y

e) 10 x

c) 11x

9 6

y

y

42. Los coeficientes de los términos centrales de los n 2 n desarrollos de : (a b) y (a b) ; n Z ; son entre sí como 15 es a 4. Calcular "n". a) 1 d) 14

b) 2 c) 3 e) Hay dos correctas.

43. Dado los términos semejantes uno del desarrollo de

x (x a

b y b )a y otro de y (x

y a )b ambos ocupan la

misma posición en cada polinomio. Determinar el valor de :

(a 2

b 2 )2

1 a 2b2 a) 2 d) 9

b) 4 e) 12

c) 6

a bx b )n , los términos de 44. Si en el desarrollo de (ax lugares a + 3 y b - 1 equidistan de los extremos; además la suma de todos los coeficientes es 27. Hallar la suma de todos los exponentes de variable "x" en su desarrollo.

a) 20 d) 14

b) 18 e) 15

45. Calcular :

(a

2

2 2

b )

(ab)2 1

0.

; ab

2

(1 2xy

b 41 e) 44

c) 42

C 3n x 2

(1 x )3

... (1 x)n

C 4n x 3

C nn x n 1

x

a) 1

b)

x 1 d) x

e) -1

...

1

x 1

c) ( 1)n C 32nn 1

d) ( 1)n C n2n11

c) 19 278

y)5

b) 10(1 y 3 )x 3

c) 5(1 y 2 )x 3

d) 5(y 2

2y)x 3

e) 10(y 1)2 x 3 y x)8 , determinar los

a) 28; 56 d) 1

b) 420 e) 6

c) -420

55. El coeficiente del término x n en el desarrollo de : x 2)

1

; es :

1 ; si : n = 3k; k Z

II. 0 ; si : n = 3k-1; k Z III. -1; si : n = 3k+1; k Z

49. Si : n Z , calcular :

a) n + x d) nx

b) 918 e) 1254

a) (1 20 y)x 3

I.

e) ( 1)n C32nn 11

x(1 x)n

k

3x 4 )7

53. Determinar la suma de todos los términos cuyo grado relativo a "x" sea 3 en el desarrollo de :

(1 x

b) C32nn 11

n

c) 1420

coeficientes de los términos de la forma : x10 y m , donde "m" es par no nulo..

3x 2 4 x 3 ...)n ; (| x | 1)

a) Cn2n11

... k

a) 807 d) 15 362

2 54. En el desarrollo de : (x

c) x

48. Determinar el coeficiente de x n en el desarrollo de :

1

b)105 e) 1480

(1 x

1 (1 x ) (1 x)2

n

a) 1260 d) 120

(1 3x 2

47. Simplificar :

M

3x 2 )7 .

c) 4

(xy 7 x 9 y 63)m tenga al menos 1998 términos es:

(1 2x

c) 148

52. Determínese el coeficiente del término en x10 del desarrollo de :

46. El mínimo entero "m", tal que :

C n2 x

b) 342 e) 510

2

b) 2 e) 8

C1n

1

z)7 es

51. Hallar el coeficiente de x 4 y 2 en el desarrollo de :

F(x, y) (ax a 1 by b 1)ab presentan el mismo grado absoluto.

a) 40 d) 43

y

ax 2y 3 z b . a) 215 d) 212

c) 16

Sabiendo que dos términos cualesquiera del desarrollo de :

a) 1 d) 6

50. Calcular : a+b, si un término de (x

2

x k (1 x)n b) n e) n - x

n 2 k

x 2 (1 x)n ... n

n n

2

xn

c) x

...

a) Sólo I d) II y III

b) Sólo II e) Todas

c) Sólo III

56. Determinar el coeficiente del término del desarrollo de (a 4 b c)n (a 2b c)n en el cual el grado de (a+b+c) excede en 14 unidades al lugar que ocupa y éste es un tercio del valor de "n". b)

c) 210 (3 2 )

d) 230

2[370 C70 0

E a) 370 (370

220 (36 )

a) 200 (13)

58. Hallar el equivalente numérico de :

70

70

c) 3 (2

368 C70 2

366 C70 ... 1] 4

1)

b) 4 70 (270

1)

1)

70

1)

70

d) 2 (2

e) 270 (370 1)

3

e) 110 (3 ) 2 12

57. Dado el binomio : (x 3 y ) , si un término de su desarrollo es contado desde el final. ¿En qué posición se ubica, si en dicho término el G.R.(y) = 2G.R.(x)? a) 6 d) 9

b) 7 e) 10

c) 8

59. Al expandir :

6

y x

6

x y

84

, se obtiene un término

cuya parte literal es (xy)n . Calcular "n". a) 42 d) 49

b) 44 e) 88

c) 78

60. Indicar el grado del producto de los términos centrales obtenidos al efectuar :

(x 39 a) 114 d) 78

39x 38

C 239x 37 ... 39x 1)3

b) 117 e) 123

c) 58

Claves 01.

c

31.

a

02.

c

32.

d

03.

b

33.

b

04.

c

34.

d

05.

c

35.

c

06.

c

36.

e

07.

b

37.

a

08.

c

38.

d

09.

b

39.

d

10.

d

40.

c

11.

e

41.

b

12.

c

42.

d

13.

c

43.

b

14.

a

44.

b

15.

d

45.

c

16.

c

46.

e

17.

c

47.

a

18.

c

48.

e

19.

c

49.

d

20.

d

50.

d

21.

d

51.

a

22.

b

52.

c

23.

b

53.

e

24.

a

54.

b

25.

c

55.

e

26.

d

56.

b

27.

d

57.

b

28.

d

58.

d

29.

a

59.

d

30.

e

60.

b

Capítulo

8

RAD ICAC IÓN

RADICACIÓN

*

División. n

Es la operación que tiene como objetivo calcular una expresión llamada raíz, tal que elevada al índice resulte otra expresión llamada radicando o cantidad subradical.

n

a

n

b

a b

Ra di ca les S eme ja nt es

Veamos : n

Si :

A

bn

b

A

En donde : n

A

radical

Son aquellos que tienen índices y radicandos iguales. Estos radicales son los únicos en los que se puede efectuar la adición o sustracción. Ejemplos :

A

radicando o cantidad subradical

b

raíz

n

índice

*

signo de radical Valor Aritmético de un radical

14 xy ; a 4 xy 2

5 4 xy ;

3 2

Adición :

radicales semejantes.

7 2

Sustracción : 11 3 4

8

3

10 2 33 4

4

Es aquel valor real, positivo y único, que elevado al índice, reproduce al radicando.

Transformación Tr T r ans ans an de radicales dobles en simples

Observación :

I.

Cuando se tiene n A implícitamente nos están pidiendo el valor aritmético.

Radicales d ic ic de la forma :

|x|

Radicales Homogéneos Son aquellos que tienen índices iguales. Es importante tener en cuenta que las operaciones de multiplicación y división, sólo se pueden efectuar entre radicales homogéneos. Ejemplo : *

A

5

xy ;

5

5

a;

zw

2

c

A2 B

Multiplicación. n

a

n

b

n

c

n

abc

A c 2

A c 2

(

debe ser racional exacta)

Ejemplo : Descomponer : *

5 24 calculemos "c" ; donde : A = 5; B = 24.

52

24

1

5

2

5 1 2

3 *

B

Donde :

c

Son radicales homogéneos.

B

Primer Método :

Debemos tener en cuenta la definición : x2

A

2

5 1 2

Segundo Método

Reemplazando en ( ) :

Se forma trinomio cuadrado perfecto, recordemos que: b )2

( a

a

3

10

108

1

3

Observación :

b 2 ab

El mismo método se utiliza para la forma :

Veamos :

3

A

B

a b

2 ab

2

( a

b)

a

1

2

A por "A" y

8

x por "x".

RACIONALIZACIÓN

Ejemplo : *

A B reemplazando en todas las ecuaciones :

b (a b)

60

5 3 2 5 3

5

3

1

Es el proceso que consiste en transformar el denominador irracional de una fracción; en otro que sea racional.

2

Factor racionalizante (F.R.) II.

Radicales de la forma : 3

A

B

3

A

B

+ y (x Q; y Q ).

x

Los valores de "x" e "y" y se obtienen resolviendo las siguientes ecuaciones :

4x3

A 3Cx ...... (1) x2 y

3

A2 B

Propiedad Para racionalizar una fracción bastará con multiplicar sus términos por el factor racionalizante del denominador. Casos de Racionalización C

C ...... (2)

I.

Donde :

C

Es aquella expresión irracional que, al multiplicarla, por una cierta expresión irracional dada la transforma en racional.

racional ( 3

exacta) .

S uge renci a : como "x" es racional entero, es recomendable "tantear" con valores enteros de "x", en la ecuación (1).

Racionalización R Ra a de Expresiones Monomiales En este caso, el factor racionalizante es homogéneo con la expresión para racionalizar, debe cumplirse que luego de la multiplicación los exponentes del radicando deben ser iguales al índice o al menor de sus múltiplos. Ejemplo : Racionalizar el denominador de : N

Ejemplo :

7

Transformar :

3

10

108

10

108

x

y ....... ( )

como : A = 10; B = 108, entonces :

C

3

102 108

x y

tendremos :

tendremos : 3

4 12

2

7

N 7

x 4 y 12

.

7

x3y2 x 3y 2

F. R.

Luego en (1) :

4x3

10 3 ( 2)x

4x3

10 6x

4+3

se verifica para : x = 1

Ahora en (2) : x2

y

2

12

y

2

y

3

igual al índice 12 + 2 = 14 (menor múltiplo de 7)

N . (F.R.) 7

7 14

x y

N . (F .R.) xy 2

deno min ador racional

II .

Racionalización de Suma o Resta de Radicales con índice 2 o sus potencias En este caso, el factor racionalizante se obtiene utilizando la diferencia de cuadrados. Recordemos : ( A

B )( A

B)

A

I V. Racionalización de Radicales de la forma n

b

En este caso, el factor racionalizante se obtiene utilizando cocientes notables, de la siguiente manera : n

n

( a

*

B

n

a

n

...

n

b )( a n

bn

1

a

n

1

n

an 2b

b (n

a n 3b 2

...

par o impar )

Ejemplo : n

x

n

4

x

4

y

n

...

FR1 4

n 1

x

k FR1

y

x

x

y

y

n

( a

*

.

b

n

b )( a n a

y

Tendremos :

k

n

...

k 4

n

( a

*

Racionalizar el denominador de :

b

n

b )( a n

n 1

a

n

1

an

b (n

n

1

2

b

n

an

3 2

b

...

impar)

n

an 2b

b (n

a n 3b 2

...

par)

Ejemplo : Racionalizar el denominador de :

M FR 2 k RF1 x . x y x

7

k FR1 FR 2

y y

x

III. Racional ización de sum a o resta de r adicales con índice 3 o sus potencias En este caso, el factor racionalizante se obtiene utilizando la suma o diferencia de cubos. Recordemos : 3

A

3

B )(

3

A2

3

AB

3

B2 )

A

Ejemplo : Racionalizar el denominador de :

P x

3

y

Tendremos : FR1 P x

3

.

x2

y x2

x3 y

3

y2

x3 y

3

y2

P . FR1 x3

y

denominador racional

7

b

Tendremos :

y2

denominador racional

(

x

B

M 7

x

7

b

.

FR 7

x

6

7

5

x b

7

4 2

x b

...

7

b

6

M.FR x b denominador racional

EJERCICIOS PROPUESTOS 01. Efectuar :

08. Reducir : (2 3

1)(3 3

2)

4

9

x3y2

E a) - 2 3 d) 16

b) 0 e) -1

2

(3 2 a) 4 d) 6

2)

2

(2 2

3)

b) 7 e) 9

10

c)

a) - 7

b) -1

d) 1

e)

7

7

3

a) 6 3 d) 2

c)

3

2

3 1

6

16 2 48 c)

2

1

R

b) d)

2 2

6

6

1 2

b) -2 e) -1

2

1 2

2

c) 1

x 7

E

x 9

2

2 4

; para : x = -7.

2 2

b)

d) 2 2

c)

2

e) 2

11. Sea :

1 2

4

a)

18

30 ) / 12

b) ( 15

18

30 ) / 18

3

c)

( 12

18

30 ) / 12

2 2

d) ( 15

18

30 ) / 18

e)

15

30 ) / 12

8

2 12

2

3

5

( 12

12. Si se cumple : 3 4 3 ( 9 4 5

d)

1

8

( 12 2

2

06. Efectuar :

a) 8

x

09. Efectuar :

E

5 3 6

3

4

xy

Entonces la expresión racionalizada es :

2

e)

c) x - y

7 +1

b) 3 e) 1

3

e)

7

Calcular :

c)

d) xy

a)

E

a)

b) x

10. Hallar el verdadero valor de :

04. Efectuar :

05.

a) 0

a) 2 d) 0

7. 3

x3y2

x y

31

03. Efectuar :

5

3 3 2

c) 6 - 2 3

02. Calcular :

6

5

2)2

b) 5

14 6 5 c) 7 2 5

6

2

a) 1 54

3

3

250

16

a) 0

b) 1

d) 5

3

e)

32

18 50 c) 2

2

2

y

z ; donde :

(x y)(x z)(y 2z)

07. Simplificar : 3

x

x > y > z. Calcule :

e) 3 4 5

5 1

5

8

d)

b) 2 3

e) 2 2

c) 3

13. Indicar el denominador racionalizado de :

19. Reducir :

1

E

35

a) 8 d) 40

6

21

B

10

b) 20 e) 25

a)

c) 10

3

3

b)

6

d)

3 1

e)

6

K

3

3 1

a)

c)

d)

2

2

a) 15 d) 45

1

8

b) 25 e) 55

7

1

c) 35

d)

b) 2 2

3

2

3 1

3

1 2

c)

2

c) 5

5

8 4 3

7 2 10

11 2 30 c) 2

a) 1

b)

5

d) 0

e)

3

6

4 1

6

x

6

y

b) 7/9 e) 3/7

a) 3 d) 6

z

c) 5/3

3b 2

2b b) 4 e) 7

x

2 c) 5

24. Indicar el denominador racionalizado de :

Calcular : V a) 0

2 1 2 1

2 1

; b

18 6 7

e)

a) 1 d) 4

c) 2 2 2

n 1

2 3

1

.

b) 2 e) 7

3

n!

a

a) 5 d) 13

1

3 2

b)

e)

3 2

3+1

Hallar : a + b.

b) 6 e) 8

c) 7

26. Hallar el verdadero valor de :

3

x

E 1 2

b reales que verifican :

b

Además : ab = (n-1)!.

3 1 3

3 2

2 14 c) 3

25. Sabiendo que : n Z ; a y 1

1 2

6 2

a 3 b ab 3 b) 1

24 2

4 7

2 1

18. Efectuar :

1 d) 2

320

1

23. Calcular "x", en :

2 e) 4

a

a)

21

3

a) 7/3 d) 5/9

3 1

17. Si :

d)

80

22. Calcular : x+y+z, si :

2 2

a) 2

2

4

3

3

5

3

9

e)

16. Efectuar :

E

3)

c)

2

b) 3

7

3

Calcular : m + n.

2

21. Efectuar :

1

2 n

3

a) 2

15. Si : m

5

e) 2 5

1

1

15

20. Efectuar :

2

.

2

2

2( 4 b)

5

d) 1

14. Calcular :

E

5

1 c) 2

3 2

8

x 1

a) 1/3

b) 1/6

d) 3

e)

1 3

3

; para x = 8. c) 6

27. Calcular el verdadero valor de :

33. Racionalizar :

2

xy 4 x

M

xy

3y

2 x

2 3

para : x = 3, y = 4.

1 2 3 la expresión resultante es : a)

a) 2 3 d) 3

b) 3 3 e) 4

c) 4 3

3

a) 2/3 d) 1/6

x

x

2 4

2

c) 1/3

29. Hallar el verdadero valor de : F

3

3x 3

x

a) 9

b) 33 3

d) 9 3 9

e)

3

3

; para x = 3. c)

3

9

3

b) 1

2

6

3

1

2

6

3

d)

1

2

6

3

e)

1

2

6

3

34. Si al dividir : 26 2 7 entre 3 7 se obtiene una expresión de la forma a b donde "a" y "b" son enteros positivos, entonces a 2 b es : a) 9 d) 2

b) 15 e) 18

c) 29

4

35. Proporcionar el valor de :

3

30. Hallar el verdadero valor de la fracción : P(x )

3

A partir de :

a) 1 b) -1/6 e) 1

c) 6 d)

4

11 2 12

{ , }

4 x x 5

cuando : x = 5. a) 1/6 d) -6

6

,

; para : x = 8

b) 4/5 e) -1/3

x2

2

c)

28. Calcular el verdadero valor de : E

1

6

3 2 2

4

N

b)

3 2 4

e)

2 2 3

c)

2 3

31. Si se cumple : 36. Racionalizar : 2 2 6x 2 7x 3

ax

de modo que : {a, b, c} Calcular : a + b + c.

N.

5x

a) d) 7

b

cx a

b) 5 e) 8

2 32. Sabiendo que : x

a)

d)

x 2 x 2

b)

e)

25

c)

3

e)

3

3

20

5

3

3

16

b) 9

3

3

2

5

3

2

3

d)

4

0 37. Indicar el denominador racionalizado de :

Reducir :

E

3

a) 1

c) 6

x 1; x

1

A

x

2x 2

x

F

x 1 2

c)

2 2

a) 1 d) 5

1 6

1

6

16

2000

b) 20 e) 8

5 c) 10

38. ¿Cuál es el denominador que se obtiene al racionalizar:

2 x 2

1 1 a) 13 d) 23

b) 17 e) 29

3

2

23 4

? c) 19

39. Racionalizar el denominador de :

44. Hallar : k 3 / 4 , si :

1

F

5

5

8

4

2 2

1

e indicar la suma de cifras de éste. a) 9 d) 12

b) 10 e) 13

c) 11

a) 2 d) 1/4

3

3

2

b) 1/2 e) 4

10

3

10

10 1

3

a) 6 d) 9

10 1 .

es equivalente a :

donde :

2

b) 6 e) 16

41. Efectuar : 1

E

a) 1

d)

3

b) 12 6

3

12

3

18 3 3

e)

3

18

a

3

c)

a para : a 4

a2

a

2

a

1

a

2 b ( 2 1) a

se le puede dar forma a enteros positivos.

b donde : "a" y "b" son

b) 12 c) 11 e) No se puede determinar.

a2 1 a

2

x2 1 2

x

12 3

18 3

a2 1

2b

47. Si : x > 1, reducir :

42. Calcular : E

x

a)

x 1 2

b)

x2 1

d)

x2 1

e)

x

c)

d) 4 3

e) 2 6

c) 3 2

43. Reducir :

5x 6

es equivalente a : a)

x

3

c)

x 3

2

b)

x

3

x

2

x 2

d)

x

3

x

2

x

e) 1 A

x

24 10 x 1 x 15 8 x 1

x x

8 6 x 1

49. Descomponer en radicales :

2 x 1 4

Siendo : 1 < x < 2.

a) 1

b)

d) x - 1

e)

2 5

x 1

c)

8 5

x 1

1

1

2x 5 2 x 2

b) 2 3

x2 1 2

48. La expresión :

6

a) 4 6

2

a b 2

a) 17 d) 19

1

3

c) 8

46. Hallar : a+b, si la expresión :

E

c) 20

3 2

b) 7 e) 10

N . Calcular el valor de : " . " . a) 8 d) 12

k

c) 3

4

10

4

2

45. Dar la suma de las cuartas potencias de los radicales simples que se obtienen al descomponer :

40. Si la expresión :

R

6

4

7

48

a)

3 2

2 2

b)

6 3

6 2

c)

6 2

6 5

d)

6 2

2 2

e)

2 3

3 2

50. Hallar el verdadero valor de : 3

a 3 a2 3

a2

56. Calcular : 3

3

a

a

3

1

H

2

6 ...

para : a = 27. a) 2 d) 0

b) 3 e) 1/3

a) 3 d) 1/3 52. Si :

5 2

3

x 6

2

x

2

2

; para : x = 2.

b) 4 e) 3/4 x

2 6x

c)

3

2

e)

3

2

53. Si :

9

N/a

2 b) 2 2x

a 2

d) 0,9

e) 0,7

...

c) 0,8

57. Calcular : 33

F

1

d)

3

3 3

2

3

T

2b

b . Mostrar un radical simple de :

3

2 1 4

1

1

b)

6

3

e)

3

2

c)

3

3

2 1 { 112 80 2

68 52 2 }

b)

d)

2

e) a ó d

c)

5

b)

d) 4

e) 3 2

c) 2

2

3

3 33

54. Si :

E

a) 1

59. Si : {x ; y ; z} Q proporcionar el valor de "x+y+z", de tal modo que se verifique :

7

1 9

3

2 9

3

4 9

2 1

a) 0,7

b) 0,6

d) 0,7

e) 0,78

3

x

3

y

3

z

c) 0,6

60. Calcular el verdadero valor de :

1)3 b) 3 e) 2

x 2 x 1

x x 1

c) 1

F(x )

x

2

para : x = 2.

55. Calcular : A

(1

3

2

3

3

2 1

4 )3

a) -2 d)

a) 1 d) 2/3

10 4 5

entonces al tranformar a radicales simples se obtiene :

a)

a) 1/3 d) 8

6 4 3

58. Si T es una expresión definida por :

3

b 2 a 6b .

Calcular : (E 3

1

990 100 99 b) 0,3

a)

2x 1 2 4 x 6

d) 2

a 4 b 2

{a ; b} a

3

3 2

a) 1

es : a) 2 2x 3

2 3 1

c) 1/4

3 el equivalente de : 2x

2

1

c) 1

51. Calcular el verdadero valor de : E

1

b) 2 e) 3/2

c) 9

11 4

b) 2 2 e)

3 2

c) 4

Claves 01.

d

31.

c

02.

b

32.

d

03.

d

33.

a

04.

d

34.

e

05.

b

35.

d

06.

a

36.

d

07.

c

37.

b

08.

a

38.

d

09.

a

39.

c

10.

d

40.

c

11.

a

41.

e

12.

b

42.

a

13.

c

43.

c

14.

b

44.

a

15.

c

45.

e

16.

c

46.

c

17.

d

47.

a

18.

a

48.

b

19.

d

49.

d

20.

b

50.

a

21.

d

51.

d

22.

b

52.

e

23.

d

53.

e

24.

e

54.

e

25.

a

55.

c

26.

c

56.

b

27.

a

57.

b

28.

c

58.

c

29.

d

59.

d

30.

b

60.

d

Capítulo

9

NÚMEROS COMPLEJOS

Cantidades Imaginarias

Ejemplo :

Se obtienen al extraer raíz de índice par a un número negativo. Ejemplo :

2;

4

7;

6

4 ; ... etc.

i

1112 910

i

1112 (4 o 1)10

i

4 o 110

1112

i4

o 1

i

Números Complejos Son aquellos números que tienen la forma :

Unidad Imaginaria De fi ni ci ón La unidad imaginaria se obtiene al extraer raíz cuadrada de -1, se representa de la siguiente manera : 1

Z = a + bi = (a ; b); a, b donde :

R

a = Re(Z) se llama, parte real de Z b = Im(Z) se llama parte imaginaria de Z

i CLASIFICACIÓN DE LOS COMPLEJOS

también se define como : i2

Complejos Conjugados (Z) Son aquellos que sólo difieren en el signo de la parte imaginaria.

1

Potencias de la Unidad Imaginaria

Ejemplo : Z = 3 +4 i ; su conjugado es : Z

1

i

i

i2

1

i3

i

4

1

i

Complejos plej Opuestos (Zop) Son aquellos que sólo difieren en los signos de la parte real e imaginaria, respectivamente. Ejemplo : Z = 5 - 2i ; su opuesto es : Zop

Propiedades : i4n

1.

i4 n

1; n Z

k

i 4(120)

i

47

10

2i

Ejemplo : De la igualdad : a + bi = 8 - 11i tenemos : a = 8; b = -11

1

ik ; (n ; k Z)

Ejemplo : i

5

Complejos Iguales Son aquellos que tienen partes reales e imaginarias, respectivamente, iguales.

Ejemplo : i 480 2.

3 4i

4(11) 3

i

3(4) 2

2

i

i

i

3

i

1

Observación : Es conveniente recordar las siguientes propiedades aritméticas.

(a

r)n

a rn

(a

r)n

a r n (n

par)

(a

r)n

a r n (n

impar)

Complejo Nulo Son aquellos que tienen su parte real e imaginaria, respectivamente, iguales a cero. Si : Luego :

a + bi es nulo a = 0; b = 0

a + bi = 0

Complejo Imaginario Puro Es aquel cuya parte real es igual a cero y su parte imaginaria distinta de cero. Si : a + bi

es imaginario puro

a=0

Complejo Real Si un complejo es real, entonces su parte imaginaria igual a cero : Si : a + bi

es real

Graficar : Z = 5(Cos40° + iSen40°) En el sistema de coordenadas polares :

b=0

Z (5; 40º)

Representación de los Complejos I.

=5

Representación Cartesiana o Geométrica En este caso, el complejo está representado de la forma:

40º

O

Z = a + bi

polo

Gráfica del Complejo Cada complejo es un punto en el plano, para ubicarlo se le representa en el llamado plano complejo, Gaussiano o de Argand, el cual está formado por un eje vertical (eje imaginario) y un eje horizontal (eje real).

eje polar

Relación entre la Representación Cartesiana y Po la r Sea el complejo : Z = a+b i (a, b >0) Im Z

b

Ejemplo : Origen

Graficar : Z1 = 3 + 4i

Eje real positivo a

Z2 = 5 - 3i

Polo

Re Eje polar

En el plano Gaussiano : En la figura sombreada :

Im Eje imaginario

Z1 = (3; 4)

4

5

Re

3

Origen

Eje real

Z2 = (5; -3)

-3

Observación : Cada complejo se representa por un punto en el plano al cual se le llama afijo del complejo. II .

Representación Polar o Trigonométrica : En este caso, el complejo adopta la forma : Z Donde :

(Cos

módulo;

i Sen )

a2

* a

Cos

* b

Sen

*

b2

ArcTg

a bi a

b a

Cos

bi

( Sen ) i

(Cos

iSen )

Para transformar de cartesiana a polar se calcula

2

Gráfica del Complejo En este caso, se utiliza el sistema de coordenadas polares el cual está formado por un punto fijo llamado polo y una semirecta que parte del polo, llamado eje polar. El módulo ( ) es la distancia del polo al punto que representa el complejo y el argumento ( ) el ángulo positivo medido en sentido antihorario desde el eje polar hasta el radio vector OZ .

y

. En el caso inverso, se calcula el valor de la función trigonométrica. Aplicación : 1. Transformar : Z = 3 + 4i

>0

argumento; 0

*

*

32

42

5

*

ArcTg

4 3

53

3 4i

5 (Cos 53

i Sen53 )

2. Transformar : Z = 6 (Cos37°+ i Sen37°) Z = 6(Cos37°+ i Sen37°) 4 5

Z

6(

Z

24 5

3 i ) 5 18 i 5

III. Representación de Euler En este caso, se tiene :

(Cos

expresado en radianes

i

i Sen )

e

d) Radicación : En general se asume que la raíz adopta la forma (a+bi) ; luego a y b se hallan por definición de radicación. Ejemplo :

Se cumple :

5 12i Cos

iSen

ei

Asimismo : (Cos

ei

iSen )

OPERACIONES CON COMPLEJOS Operaciones en forma cartesiana a) Adición y multiplicación Se utilizan las mismas reglas algebraicas. Ejemplo : (3+i)(3+2i) - (5-4i)

9 6i 3i 2i2

5 4i

Z

Z

2 3i 3 i . 3 i 3 i

Z

6 7i 3 9 ( 1)

2 3i 3 i

6 2i 9i 3i 2 9 i2 9 7i 10

9 10

7 i 10

c) Potenciación : Se utiliza el teorema del binomio. Ejemplo: (2i 3)2

b 2 ; 12

2ab

a b

3 2

5 12i

a

3

b

2

3 2i

5 12i

3 2i

Observación : i) =

1 i 1 i

2i

i

1 i i 1 i Operaciones O pera en forma polar *

b) Divi sión Se multiplica el numerador y denominador por el complejo conjugado de este último. Ejemplo :

a2

Resolviendo :

*

9 6i 3i 2 5 4 i 2 13i

2abi

Igualando :

* (1

Resolución :

a 2 b2

5 12i

5

a bi

a bi

Elevando al cuadrado

Siendo : e = 2,71828 .... (base de los logaritmos naturales).

I.

5 12i

4 i 2 12i 9 4 12i 9 5 12i

a) M Mult iplicaci ón : En este caso, los módulos se multiplican y los argumentos se suman.

Z1

1(Cos 1

Z2

2 (Cos 2

Z1 Z 2

i Sen

1)

i Sen

1 2 [Cos ( 1

2) 2)

i Sen( 1

2 )]

b) División : En este caso, los módulos se dividen y los argumentos se restan.

Z1

1 (Cos

Z2

2 (Cos

Z1 Z2

1 2

[Cos (

1

2

1

i Sen

1)

i Sen 2 ) 2)

i Sen (

1

2)]

c) Potenciación : En este caso, el exponente eleva al módulo y multiplica al argumento. [ (Cos

i Sen )]n

n

[Cos n

Sea : Z = (Cos + iSen )

Z

n

Cos (

2k ) i Sen( n

2k ) n

k = 0, 1, 2, ..... , (n-1)

Nota : observa que

n

3

1

3

3

1

Cos

i Sen n ]

d) Radicación : En este caso, se aplica la fórmula de De Moivre.

n

Ejemplo : Hallar las raíces cúbicas de la unidad.

z tiene "n" valores.

1 0i 0

3 Cos 0

2k 3

i Sen0

i Sen 0

2k 3

k = 0, 1, 2 k=0

3

1 =1

k=1

3

1 =

1 2

3 i 2

w

k=2

3

1 =

1 2

3 i 2

w2

Raíces cúbicas de la unidad : 1; w; w 2 . donde : 3 * w

1

2 * 1 w w

0

EJERCICIOS PROPUESTOS 01. Calcular :

07. Si : (ni12

2

8

a) 76 d) -44

12

12

3600

b) -76 e) 50

1

Calcular :

(i

i4

i9

2 i5

i16

i10

i

i15

1) b) 2 e) 4i

2 3

b)

d)

1 3

e) 3

a2

(i

i28

Calcular :

i321 i49

i1921 i1932 i1960

i50

i17

i1973

i2003

1) b) -i e) 1 - 1

c) 1

i i2 i 3 i4 ... i 2003

J 1)

b) 2 e) 2i

c) 6

R; además : i 2

m a

3 2

2

2

n

1

b mn

2

b) 2 e) 5

c) 3

A (1 i) (2 i 2) (3 i 3 ) (4 i 4 ) ... (4 n i 4 n ) a) n (2n+1) c) 0 e) 2n(4n-1)

b) 2n (4n+1) d) n(4n+1)

3 7 (a 2ai)

Si : n R a R . 3 8

a)

c) -1

05. Hallar la suma "A" de números complejos :

9 4

b)

9 8

e)

3 4

c) 9

3 (n i) 5(n 3i) 1 2i es un complejo real. Calcular : "n".

10. Si : n R z

a) -3/8 d) 9/4

b) 9/8 e) 3/4

c) 9

11. Hallar "n", si el número siguiente es imaginario puro : 3 2ni 4 3i

06. Calcular :

a) 0 d) 3i

1)

3 (n i) 5(n 3i)

d)

a) 1 d) i

(i

R

m ni

a) 1 d) 4

04. Reducir :

V

bi ; {a ; b ; n}

09. Calcular "n", si se cumple :

a) i d) -1

(i

bi

{a; b; m; n}

c) 3i

03. Simplificar :

Z

a 2 ) ; (i

a)

08. Si :

a) 1 d) 2i

b 2 (n n

a2

c) 44

02. Reducir :

V

i13 ) 2i n

12 1011

i9

16 1415

i13

20 1819

i17

a) -1 d) -4

b) -2 e) -5

c) -3

12. Sabiendo que :

1) b) 1 e) -3i

c) 3

z

a 2i ; es un número real. b 3i

b (a 8) i ; es un número imaginario puro.. a bi Indique : a - b. w

a) -12 d) 8

b) 10 e) -10

c) 24

13. Si : {z1 ; z2}

21. Sean : Z1 ; Z 2 C . Reducir :

C , calcular :

5z z 2z Im ( 1 2 ) Im ( 1 3z1 4 z 2 3 z1 a) -3 d) 3

b) -1 e) 0

| z1 z2 |2 | z1 z2 |2 Re (z1 . z 2 ) Re (z1 . z 2 )

3z 2 ) 4z2 a) 1 d) 3

c) 1

14. Si "i" es la unidad imaginaria, al efectuar la siguiente operación :

2 (1 i)16 (1 i)16 a) 0 d) 512 i

b) 1 e) 256

15. Calcular el valor de : a) 1 + i d) -1 + i

2i .

b) 1 - i e) a ó c

c) -1 - i

16. Determinar el módulo de : (7 3i)( 5

Z

( 5 2i)( 6

a) 1

b) 2

d) 2 7

e) 14

17. Sea : Z1

Z2

Determinar : 58

Z2

a) 3 + i d) 2 - 2i

i)

2

b) n

d)

n (n 1) 6

e)

c)

n (2n 5)(1 n) 6

1

.

a) 2 (7 12i) 1

b) 6 (7 24 i) 1

c) 7 (6 4 i) 1

d)

3(4

c) 4

4 i)((1 i)4

Hallar : K | z

w |2 | z w |2

a) 36 d) 18

b) 26 e) 22

25. Indique el módulo de : 4 i)( 3 i 1)

(2 2i)(1 3i)

W

(1 i)( 7

c) 32

n(1 i) ; n R ; i

b) 4 e)10

1

a) 1

b) 2 3

d) 2 2

e) 2

c) 6

d) 4 13

b) 2 13 e) 5 13

c) 3 13

3i) c)

R.

Además : m ni x yi Hallar el equivalente de :

20. Hallar el módulo del complejo "Z", si al dividirlo entre 5+i y al cociente sumarle 2, se obtuvo 3-i. 13

1

c) 34

26. Sabiendo que : m, n, x, y

a)

3i)

24. Sean : |z|= 2; |w| = 3.

19. Hallar "n".

a) 2 d) 8

n (2n 5) 3

e) 7 (6 28i) 1

2

b) 8 e) 128

8 (1 i)6

n(n 1) 2

Indique : z

18. Determinar el módulo de :

a) 2 d) 64

a)

|z| - z = 3 + i

| Z1 |

((1 i)4

n Z .

3i)

1 i

b) 5 - i e) 4i

Z

(1 i)2 (1 2i)2 (1 3i)2 ... (1 ni)2 ;

23. Si : z C , resolver :

c)

2 5i

c) 2

22. Indique la parte real de : z

c) -256

b) 1/2 e) 1/3

K

a) 6 d) 12

b) 4 e) 10

n2 my 2

y4 c) 8

2

27. Si : 3 a bi

m ni ; {a ; b ; m ; n}

además : i Calcular :

33. Efectuar :

R

m3n 3 b) 1 e) 3

c) -3

z

2| z | 0

b) 9 e) 2

2 (Cos10

z2

8 Cis 20

4Cos5

b) -1/2 e) 1

34. Sea : w1

Sen20

a) 190° d) 340°

29. Efectuar :

i

5

c) 1/4

i Cos20 , hallar :

Arg (w1) .

c) 1

2 i

i Sen10 )

4iSen5

a) 4 i d) i/2

(0,0) . Indique : Re(3z) - Im(z).

a) -3 d) -2

z1

z3

28. Resolver en :

C : z2

z 43

sabiendo que :

(m3 a)(b n3 )

a) 3 i d) -3 i

z15 z 32

K

1.

b) 250° e) 200°

c) 240°

35. Efectuar :

i

1 i a) 1 + i

b) 1 - i

d)

e)

2i

2

c) i

1 i 2

30. Hallar "Z", si cumple : 1 Z

a) 3 - 4i d)

5 3 4i

1

6 25

Z

|Z|

e)

5

c)

3 4i

e) e

a) d)

iCosx )4

Senx iCosx es :

5 Cis 233

b)

10

1 2

Calcular : z 3

c) 2 2 Cis 135°

a) 2 e i

2 Cis 135°

e) 5 Cis 135°

d)

32. Llevar a su forma exponencial :

c)

1

3 i 2

z3 . b) 2 e 2 i

3i

e) e

c)

2 i 3

38. Reducir :

4 4 3i

a) 16 e d) 8 e

4 i 3

4 i 3

b) 4 e

2 i 3

e) 8 e

2 i 3

c) 4 e

L

4 i 3

e4 e4

a) 1 d) -i

2

e)

5

37. Si : z

b) 5 Cis 233° d)

d) e 2

c) e / 2

/2

5

31. Llevar a su forma trigonométrica : z = -3 - 4i a)

b) e

36. Un número real "x", que satisface la ecuación :

5

i

3

a) e

(Senx n

b) 4 - 3i

4i

b) -1 e) e

i i

e

4

e

4

i i

c) i

2 e2

i

45. Una de las raíces "Doceavas" del complejo Cis 12°; presenta el mayor argumento, indíquelo :

39. Proporcionar un equivalente de : ii . a) e

/4

b) e

d) e 3

/2

e) Hay 2 correctas

/2

a) 311° d) 391°

c) e

a)

d)

2 4 2

e (1 i) 4

2 2

b)

e)

2

4

1

c)

4

2

a) b) c) d) e)

w1 Determinar : w1n

3 i ; w2 2

1 2

3 i 2

w 2n ; n Z ; n = par..

n 3

b) 2Cos

2n 3

c) 2Sen

2n 3

d) 2Sen

n 3

51

b) 0

d) 2101

e) 3101

4

b) 2n

d) 3n

e) 16 n 3,

4

5

1)(w 4

a) 1 d) Cos (7°)

b)

/3

1)...( w 6 n 1)

c) 8n

Re

Im

d)

Re

Re

Im w Re

2187 w 49. Calcular "n" en : (1 w)2n Siendo "w" una de las raíces cúbicas de la unidad. a) 1 d) 7

b) 4 e) 8

c) 5

50. Dados los complejos : z1 ; z 2 ; z 3 en el plano Gausseano :

z1

1 , calcular : 1

2 3i

c) 2

w

w

c)

50

son las cuatro raíces imaginarias de:

Im (

?

w

e)

(w 1)(w 2 1)(w 3

2,

/2 /4

Re

w

c) 1

a) 4 n

1,

5

Im

a)

43. Si : "w" es una de las raíces cúbicas de la unidad real, calcular :

44. Si :

b) e)

3 i 2

Im

{(1 w)(1 w )} {(1 w )(1 w )}

E

w

1 2

Im

a) 3100

5

a) /6 d) -

n 6

2

4

48. Del problema anterior, grafique el complejo:

42. Si : "w" es raíz cúbica de la unidad real, calcular : Z

3

47. Indique el argumento del complejo :

a) 2Cos

e) 2Cos

2

Nulo. Real. Imaginario puro. Su módulo es 1. Más de una es correcta.

41. Haciendo : 1 2

c) 361°

46. Si : 0 ; 1 ; 2 ; 3 ; 4 ; 5 ; son las raíces de 0rden 6 de la unidad. ¿Qué clase de número es :

40. Hallar el módulo de "z" que verifica : ez

b) 321° e) 331°

2

b) 0 e) Sen (36°)

3

Im 15º

4)

c) -1

45º

Re

15º z3

z2

Indique verdadero (V) o falso (F) : Im

I.

Arg (z1 . z2 )

340

II.

Re(z1) Im(z2) 0

c)

III. Arg (z3 ) Arg (z1) a) FVV d) FFF

240 c) VVV

z2

d)

2ab a

2

b

b)

2

b

b2

2ab

c)

2

a

2

b2

1 Cos74

Sabiendo que : 1 Cos 2 a) 1,7 d) 1,6

2

b) e

d) 5 / 4

e) 3 / 4

(1 w w 2 )(1 w 2

2 Cos 2

a) 1

b) ( 1)n

2n d) 2

e) 2 2

c) 1,1

e)

5

2 i

a) 1 d) 4

e

c)

3

e

5 2 8

z2

4

4

(

ze

i 3

1

c) 0

e3

i

, donde :

, donde :

4

;

Im

a)

)

, indique el complejo :

;

3 8

.

a) ln 5 d) i ln 3

b)

n

c) ln 2

1 i 1 i

Re

1 ( 3 1) ; i 2

1

si éste es de 4 cifras. a) 1000 d) 1005

b) 1009 e) 1006

c) 1004

60. Reducir : (a bw )2 (b aw)2 (a

a) a + b d) 2b - a

z2 Re

3i 5

b) ln 3 e) i ln 2

Si : b > a ; w

Im

z2

c) 2n

59. Calcular el mínimo valor natural de "n" que verifica la igualdad :

e

b) 2 e) 6

55. Dado el complejo : z

w8 ) ... 2n factores

n

Tg z

2 i

54. Si : y , son las raíces cúbicas imaginarias de la unidad, el equivalente de : K

2

58. Resolver en C :

z

d) 5 e 3

i

c)

w4 )(1 w 4

53. Hallar el módulo de :

b) 1

(1 i)3

/4

a) 1

i Sen n7 74 7

b) 1,5 e) 1,8

3 a) e e

i 3

57. Si "w" es una de las raíces cúbicas imaginarias de la unidad, calcular :

52. Hallar el módulo de :

z

Re

w

a 2 b2 e) 2ab

ab a2

a

Im

56. Hallar el módulo de :

ab 2

Re z2

e)

, son números reales y :

a bi ei a bi entonces el valor de Tg es :

a)

d)

Re z2

b) FVF e) FFV

51. Si : a, b y

Im

3

bw 2 )2 (b aw 2 )2

1.

b) a - b e) 2a - b

c) b - a

2ab

Claves 01.

c

31.

b

02.

b

32.

e

03.

c

33.

d

04.

c

34.

b

05.

b

35.

e

06.

d

36.

c

07.

c

37.

b

08.

c

38.

d

09.

b

39.

e

10.

b

40.

a

11.

b

41.

a

12.

e

42.

e

13.

e

43.

a

14.

e

44.

b

15.

e

45.

e

16.

b

46.

e

17.

d

47.

c

18.

d

48.

d

19.

d

49.

d

20.

b

50.

e

21.

c

51.

c

22.

e

52.

d

23.

d

53.

a

24.

b

54.

c

25.

c

55.

e

26.

b

56.

b

27.

e

57.

d

28.

d

58.

e

29.

a

59.

d

30.

a

60.

c

Capítulo

10

ECUACIONES DE PRIMER Y SEGUNDO GRADO

E cuacio ne s Son igualdades condicionales, en las que al menos debe existir una letra llamada incógnita :

Manera correcta :

(x 1)(x 1) x 1 única solución

Ejemplo : 2x - 1 = 7 + x Es una ecuación de incógnita "x".

3.

Solución de una ecuación Es el valor o valores de la incógnita que reemplazados en la ecuación, verifican la igualdad. Si la ecuación tiene una sola incógnita a la solución también se le llama raíz.

x=3

7

x 7

x 2 14x 49

(no verifica la ecuación dada) solución extraña

Solución :

La ecuación no tiene solución, es incompatible. Ecuaciones E ccu cua ua de Primer Grado Son aquellas ecuaciones que adoptan la forma : ax + b = 0

Solución de la ecuación : En : ax + b = 0

Simplificando :

solución o raíz : x =

(x-1) x +1 = 7 x=6 para no perder una solución : x-1=0 x=1 Si se multiplica ambos miembros de una ecuación por una expresión que contiene a la incógnita, entonces, se pueden introducir soluciones extrañas. (Esto se evita simplificando previamente). Resolver :

En : ax + b = 0

1

no verifica

raíz : x =

b a

Entonces : Si : a = 0 Si : a = 0 0

b=0 b 0

Ec. Indeterminada Ec. Incompatible

Ec. Determinada.

Ejemplo : Hallar, "a" y "b", si la ecuación :

5 (x 1) x

b a

Discusión de la raíz

Si : a

x2 1 5 Ejemplo : x 1 (x-1) pasa a multiplicar :

resolviendo :

x2

x2 7

Si de los dos miembros de una ecuación se simplifican o dividen, factores que contengan a la incógnita, entonces, se perderán soluciones. (Esto se evita, si la expresión simplificada se iguala a cero). Ejemplo : (x+1)(x-1) = 7(x - 1)

(x 2 1)

4

Elevando al cuadrado :

Observaciones :

2.

x

Si ambos miembros de una ecuación se elevan a un mismo exponente, entonces, se pueden introducir soluciones extrañas. Ejemplo :

Ejemplo : x - 3 = 10 Solución o raíz : x = 13.

1.

5

x

4

(a - 3)x + b = 5, es indeterminada.

Solución :

2

x1 ; 2

5 b a 3

x

5-b=0 a-3=0

1

x1 ; 2

si es indeterminada : b=5 a=3

ax 2 ax 2

bx c

{

Discriminante (

Forma General :

2 2 13 6

13 3

CS

Ecuación de Segundo Grado (Cuadrática)

52 6

1 3

13 1 ;

13 3

}

) dada la ecuación cuadrática en "x" :

bx c

0; a

0

se define como :

0

donde :

b2

4 ac

x = incógnita, asume dos valores

a ; b; c R / a

*

0

Para la ecuación : 2x 2 su discriminante es :

Resolución de la Ecuación :

( 5)2

1.

25 8 17

Por Factorización : *

Resolver la ecuación : x 2

x 6

0

factorizando :(x-3)(x+2) = 0

5x 1

0

4(2)(1)

Propiedad del Discriminante : el discriminante de una ecuación cuadrática permite decidir qué clase de raíces presenta; es decir :

ahora : x-3 = 0; x+2 =0 despejando : x = 3; x = -2 luego : C.S. = {3; -2} *

Resolver la ecuación : 4 x 2

9

0

1.

Si :

2.

Si :

3.

Si : < 0, la ecuación tiene raíces imaginarias y conjugadas.

factorizando : (2x+3)(2x-3) = 0

R el a ció n eent n r e l a s R aíce s y l os C oe f i ci e nte s ( pro pi ed a d ess d e l a s r aí ce s ) d e una ecua ci ó n

ahora : 2x+3 =0; 2x-3 = 0 despejando : x = -3/2; x = 3/2

cuadrática : si x 1 ; x 2 son las raíces de la ecuación cuadrática en "x".

luego : CS = {-3/2; 3/2} 2.

> 0, la ecuación tiene raíces reales y diferentes. = 0, la ecuación tiene raíces reales e iguales.

ax 2

Por la Fórmula General : Si :

ax

2

x1 ; x 2 bx c

son las raíces de la ecuación

0; a

0 , estas se obtienen a partir de

la relación :

x1 ; 2

*

b

b 2 4 ac 2a

0

observar que : a = 3, b = -2 ; c = -4

x1 ; 2

( 2)

0; a

0

se cumple : 1.

Suma : s

x1

x2

2.

Producto :

p

x1 . x 2

*

b a c a

Para la ecuación :

2x 2 10x 1 0

Resolver la ecuación :

3x 2 2x 4

bx c

( 2)2 4(3)( 4) 2(3)

x1

x2

10 2

5 ; x1 . x 2

1 2

Observación : para determinar la diferencia de las raíces se recomienda utilizar la identidad de Legendre. (x1

2

x2 )

(x1

ax 2

2

x2 )

4 (x1 . x 2 )

Casos Particulares : dada la ecuación cuadrática en "x",

ax

2

bx c

0; a

0 de raíces x 1 ; x 2 , si éstas son :

1.

Simétricas, se cumple : x1 x 2 = 0

2.

Recíprocas, se cumple : x1 . x 2 = 1

Reconstr ucción de la Ecuación Cuadrática en "x" : siendo "s" y "p", suma y producto de raíces, respectivamente, toda ecuación cuadrática en "x" se determina según la relación: x2

sx

p

Ecuaciones Cuadráticas Equivalentes : siendo :

0

a1 x

bx c

2

0

b1 x c1 0

se cumple : a a1

b b1

c c1

Ecuaciones Cuadráticas con una raíz común : Sean :

ax 2 al x

bx c 2

0

b1 x c1

0

se cumple : (ab1 a1 b)(bc1

b1c)

(ac1 a1 c)2

EJERCICIOS PROPUESTOS 01.

Sea la ecuación de incógnita "x". a) 6

m

x

3

Si la solución es : x = 49. Hallar el valor de "m". a) 4 d) 13 02.

b) 8 e) 2

c) 5

07.

Resolver la ecuación si se reduce al primer grado en "x". 2

ax

2x a 5x

a) -1 d) -1/17 03.

d)

2

3ax 4;

b) -16 e) -1/9

08.

a) 10 d) 32

2

II.

1

09.

10.

x 3 x 3 Rpta. : ....................................................

IV. 2x x 2 3x 4 Rpta. : ....................................................

11.

3 ; e indicar la suma de

x 1

c) 12

1

1 x

b)

1 2

e)

1 5

1

3 x

1 x

c)

1 3

x 4 x 1

b) 2 e) 5

c) 12

En la actualidad, la edad de Pedro es el doble de edad de Juan más 2 años. Hace 3 años la relación de sus edades era como 3 es a 1. Dentro de 5 años, la suma de las edades de Juan y Pedro será : b) 30 años e) 18 años

c) 26 años

Al resolver la ecuación :

a) 3 d) 16 12.

x

b) 4 e) 11

a

c) 9

Si la ecuación : (3a 4 )x 2 2ax 2 ax 2 2x 18 Se reduce a una de primer grado en x". Indicar el valor de "x".

Hallar "x" en : b 1 ;a x b

b) 14 e) 10

44 x 3 se obtuvo como una de sus soluciones el valor 5, hallar el valor de "a".

1 x 1

c) 1

a b a x

De un juego de 32 cartas, se sacan primero "x" cartas y tres más; luego se saca la mitad de lo que resta. Si todavía quedan 10 cartas. ¿Cuántas cartas sacó la primera vez?

x2

indicando, luego : x 2 1 .

a 1 x b

1 4

a) 36 años d) 20 años

Resolver :

a) 0 d) 3

b 2

Resolver la ecuación :

a) 9 d) 8

1

5x

2x 3 x 1

a

c) 20

x 2 (8 x)(x 9) 16(x 9)(x 8) Rpta. : .................................................... 6

a b ab

c)

b) 11 e) 15

a) 1

(3x - 1)(x - 8) = (2x + 7) (x - 8) Rpta. : ....................................................

III. x 2

e)

x 2 cifras de : 3x + 8.

d)

b) 24 e) 44

a b a x

Resolver :

1

Resolver las ecuaciones mostradas : I.

06.

b

Si la ecuación :

Tiene infinitas soluciones. Hallar : ab.

05.

a

b)

1

36x - 8 + 4ax + b = 13ax - b + 2

04.

b b

a) 10 d) 13

(a R)

c) -15/17

a x

b a)

5 2

b)

4 3

d)

2 5

e)

3 4

c)

8 3

13.

Calcular : "m.n", si la ecuación :

20.

n (x 1) 2 es compatible indeterminada. mx

a) 12 d) 54 14.

3

b) 18 e) 45

a) 124 d) 123

c) 72 21.

(2a 1)x 2

2x (x 3)(x e indicar lo correcto : a) b) c) d) e)

4)

(x

2

9)(x

4)

Tiene dos soluciones enteras. Tiene tres soluciones negativas. La mayor solución es 4. Tiene una solución fraccionaria. Tiene tres soluciones.

3x 2 x 3x 1

a) 3/4 d) 1/2 22.

4 , se obtiene : 23.

a) x = 0 c) E. Incompatible e) x = -2 16.

17.

m m

x

n n

a) m + n

b) m

d) n

e)

2

a) 3 1 d) 1

4 4

m 2 n2 mn

2

24.

5x 1

1

b) 2

x

2 1

c) 4

26.

x a) a + b d)

a

1 a

x b) a - b

b

e)

1 b

x

a

x

b

b) 5 e) -2

ab

b) 8 e) 12

c) -3

Si la ecuación :

c) 32

b) 3/2 e) 5/2

2x 3

c) 1/2

Resolver la ecuación de primer grado en "x" : 4 x ) ax(3x 2

a) 5 2

b) 6 2

d) 2 2

e)

4)

2(6x 3

5)

c) 3 2

22

¿Para qué valor de "m" la ecuación : 1

b) 3 e) -2 ó -3

3m

c) 2 ó 3

Hallar el valor de "n" para que la ecuación :

a) 8 d) 7

c) ab

El jardinero A planta rosas más rápidamente que el jardinero B, en la proporción de 4 a 3. Cuando B planta "x" rosas en una hora, A planta "x+2" rosas. ¿Cuántas rosas planta B en 4 horas? a) 6 d) 24

a) 4 d) -1

(n 2 10)x sea incompatible.

1

27. 19.

c) 5/6

0 tiene una raíz que es igual a -7?

a) 2 d) -2

Calcular "x", en :

1

a 3b b 1

( m 2 5 m 6 )x m m es compatible indeterminada?

e) 5 1

1

x) es 2.

ax 2 15 x 7

2(a

25.

x

5) 7 b(a

b) 2/3 e) 7/8

a) 2 d) -1

(n m) 2

3

b)(x

¿Qué valor admite "a", si la ecuación :

c) n - m

3

a(x

ax 3 3x 2 ax 2a ab bx bx 2 es de primer grado, el valor de "x" es :

Resolver :

x

18.

b) x = 2 d) x = 1

Hallar "x", en : x

c) 132

Dar el equivalente de : E

Al resolver la ecuación : 2x 4 x 2

b) 142 e) 134

Una de las soluciones de la ecuación mostrada :

Resolver : 2

15.

Los 3/4 de un barril más 7 litros son de petróleo y 1/3 menos 20 litros son de agua. ¿Cuántos litros son de petróleo?

nn

2

7nx

n 1

b) 5 c) 2 e) Dos anteriores son correctos.

Indicar la suma de soluciones de : x 2 (x 5) a) 5 d) 1

2 x x 4 b) 9 e) -4

16 (x 5)

2 x x 4 c) -1

28.

Indicar el cociente entre la mayor y menor de las soluciones de : 1 x

2

(x 6)(x

3x 10

a) 5 d) 1

x 2(x 2)(x 6)

2)

b) 9 e) -6

35.

x

3x 10

c) -1 36.

29.

La ecuación :

x 1 x 3

2x

b) {1} e) { }

x

x

2y 2

2

1

y

y 1 2

31.

a) 7/13 d) 5/13 32.

2 3

2x 8 x 5

b) 11/3 e) 6/13

Resolver :

a) a + b d) b

b (1 a

b ) 1 x

b) a - b e) ab

39.

a 1 b

d)

a

a

b b

b) e)

40. a

c)

ab 1 b

b

Resolver la ecuación :

1 a) 7 d)

b)

1 7

x

7

e) 49

3

x

c) 15

Resolver : 2

2x

b) 3 e) 5

2 2

5

c) 4

Tres niños se han repartido una bolsa de caramelos, el primero la mitad de los caramelos y uno más, el segundo la tercera parte de lo que quedó y el tercero el resto. ¿Cuántos caramelos hubo en la bolsa? c) 38

Habiendo perdido un jugador la mitad de su dinero, volvió al juego y perdió 1/2 de lo que le quedaba; repitió lo mismo por tercera y cuarta vez, después de lo cual le quedaron 6 soles. ¿Cuánto dinero tenía al comenzar el juego? b) 84 e) 86

c) 72

Los ahorros de un niño constan de : (P + 1), (3P - 5) y (P + 3) monedas de 5, 10 y 20 centavos, respectivamente. ¿A cuánto ascienden sus ahorros, si al cambiarlo en monedas de 25 centavos, el número de monedas obtenidas es el doble del número de monedas de 5 centavos? b) 455 e) 360

c) 345

Se compran cajones de naranjas a 100 soles cada uno; cada cajón contiene 20 kilos, primero se vende la mitad a 20 soles el kg, después la cuarta parte a 15 soles el kg, y por último el resto se remata a 10 soles el kg, ganando 11,250 en total. ¿Cuántos cajones de naranjas se habían comprado? a) 65 d) 50

a 1

9x

3

b x

a)

3

a) 800 d) 400

Hallar "x" de la ecuación :

b

e)

a) 94 d) 96

c) a

a2

34.

0

c) 3/11

a ) x

4

a) 25 b) 32 d) 14 e) No puede ser determinado. 38.

x 3 x 4

a (1 b

33.

37.

c) 0 e) 2

Hallar el valor de "x", en : x x

d) 20

x

5 2

1

a) 1 b) 0,1 d) Indeterminado.

b) 21

a) 2 d) 1 c) {2}

2

3x

5x 6

En la siguiente ecuación, determinar el valor de "y", si: x = 1. x2

3

a) 41

x 11

x2 tiene como conjunto solución a : a) {3} d) {-3}

30.

x 5 x 2

2

x

Dar como respuesta : 2x + 1.

1 2

Resolver :

3 x

1 7 1 c) 49

41.

b) 70 e) 60

c) 55

Si : " " es una raíz de la ecuación : x 2 5

Calcular :

8 1

a) 5 d) -3

b) -5 e) 1

c) 3

x

1

42.

Dada la ecuación indeterminada en "x": 1 b(2x 3 Calcular el valor numérico de: a(x

2)

a3

R

43.

a)

5 8

d)

3 4

3

b)

1 3

e)

2 3

2

4 3x 2

5 2

c)

2

a) VVV d) FFF

2

48.

7 2 e) 5/2

d) -5/2

Si la ecuación : nx 15 5

49.

c) -2

6n 5 2n

x 12 2

3 2

45.

c) R

1/3; 3/2

e) R

0 ; 5/2

b) R

0;1/3

d) R

1/3

46.

9 4

b)

2 5

d)

7 5

e)

49 20

Resolver la ecuación : x

mab nbc pac

x

pac nbc mab

a) b) c) d) e)

6

a)

1) c)

c) -2

e) 51.

qx mab nbc

q 3

pac

b) mab + nbc + pac d) 1

b)

b

b)

1 a3 a 3b

d)

(1 a)2

5x 17

2x 2

17 x 15

2

17 x 15

2x 2

5x 17

2

3

x

xb 2

3

x 2b

b3 (1 a)2 a 3b (1 a)3

a b (a b)3

Luego de resolver : x a x a

2x 2

Hay 2 valores para x. x es par. x es negativo. x es positivo. Hay 2 correctas.

mab pac nbc

Hallar el valor de "x".

Al resolver la ecuación: 2x

x

a (x

Calcular la suma de posibles valores que adopta "x". a)

b) (a+b) (a-b-c) d) (a+b) (a-b+c)

a) 2 c) mnp e) a + b + c 50.

n(x n

c) VFV

Determinar el denominador positivo de dicha raíz.

De la ecuación de primer grado mostrada:

(n 1 5x n 5 ) x

5x 6 2x 3x 2

Sabiendo que: b c b a c Resolver : x x x a c b 3(a c) a b a b a b b c b a a c

5

Presenta solución única en "x". Calcular los valores que adopta "n" a) R

4 x

2x

b) FFV e) VVF

a) (a+b) (a+b-c) c) (a-b) (a+b-c) e) (a+b) (-a-b-c)

n2 .

b)

2 3x 2

Se afirma : I. El conjunto solución = {2/3}. II. La ecuación es compatible indeterminada. III. La ecuación es inconsistente.

b3 c 3 abc

Calcular el valor de "n" a partir de la ecuación incompatible en "x": 1 n(x 1) 7 (4 x 10) n

a) 9/2

Luego de resolver :

5) c

2

Dar como respuesta : 1 n

44.

47.

Señale : x

2

ax

a

x a x a

4x a 2a

2

a)

25 a 2 16

b)

61 a 2 16

d)

9 a2 16

e)

61 a 2 25

c)

5 a2 4

52.

57.

Resolver en "x" : a2

(ax

b)

a

ax

b

a2

a)

b

a

c)

b

a2

d)

a

3a) b ; a

b

b

a) 1480 d) 2380

b

e) 53.

58.

Si las soluciones de : (nx 1) (x m) (mx 1) (n x )

son

54.

0

a b2 a

b)

a

a( x

tales que :

y

Hallar : 3 - 2

2

a) -5 d) -3

b) 2 e) 1

(nx 1) (m x) (mx 1) (n x )

<

.

c) -1

59.

Resolver : (a b)x

2

(a

b 2 )x

2

2

b )x

2abx

ab(a

b)

2abx ab(a b)

a 2 a b ab a) - a a b

d) 55.

b) - b

c) ab

e) a + b

Al resolver la ecuación : 45

45

8 x 8

8 x x

45

x 2

2a

se obtiene :

b

a

c

1

Indicar el valor de : a + b - c. a) 1 d) 4 56.

b) 2 e) 5

c) 3

Resolver, para "x" : n 1 K

1 ( )K 1 x

1

1 n n

x n 1(x 1)

a)

n n 1

b)

2n n 1

d)

1 1 n

e)

n n 1

c)

2n n 1

60.

c) 1400

b) S/. 490 e) S/. 800

c) S/. 575

Una librería tiene, para la venta, un cierto número de libros. Vende primero las 3/5 partes y después le hacen un pedido de los 7/8 de lo que le queda, pero antes de servir este pedido se le inutilizan 240 libros y por lo tanto, enviando todos los libros útiles que le quedan, sólo cubre los 4/5 de la cantidad pedida. ¿Qué cantidad de libros se vendieron? a) 2000 d) 3520

a 2 ab a b

b) 1500 e) 2000

Se reparten S/. 3000 entre cuatro personas, de tal manera, que a la primera le corresponda S/. 400 más que a la segunda; a ésta, 3/5 de lo que le corresponde a la tercera, y a ésta S/. 600 más que a la cuarta persona. ¿Cuánto recibió la segunda persona? a) S/. 500 d) S/. 600

.

(a b)x 2 (a 2

Un comerciante tenía una determinada suma de dinero. El primer año gastó 100 pesos y aumentó a lo que quedaba un tercio de este resto. Al año siguiente, volvió a gastar 100 pesos y aumentó a la cantidad restante un tercio de ella. El tercer año gastó de nuevo 100 pesos y agregó la tercera parte de lo que quedaba. Si el capital resultante es el doble del inicial. ¿Cuál fue el capital inicial?

b) 3000 e) 2240

c) 1760

¡Caminante! Aquí fueron sepultados los restos de Diofanto. Y los números pueden mostrar, ¡Oh milagro! cuán larga fue su vida, cuya sexta parte constituyó su hermosa infancia. Había transcurrido además una duocécima parte de su vida, cuando de velo cubrióse su barbilla. Y la séptima parte de su existencia transcur rió en un matrimonio estéril. Pasó un quinquenio más y le hizo dichoso el nacimiento de su precioso primogénito, que entregó su cuerpo, su hermosa existencia, a la tierra, que duró tan sólo la mitad de la de su padre. Y con profunda pena descendió a la sepultura, habiendo sobrevivido cuatro años el deceso de su hijo. Dime cuántos años había vivido Diofanto cuando le llegó la muerte? a) 99 d) 86

b) 95 e) 90

c) 84

Claves 01.

e

31.

b

02.

d

32.

a

03.

c

33.

a

04.

-

34.

c

05.

d

35.

c

06.

d

36.

c

07.

b

37.

b

08.

d

38.

d

09.

a

39.

d

10.

a

40.

d

11.

e

41.

a

12.

c

42.

e

13.

b

43.

b

14.

d

44.

e

15.

d

45.

e

16.

c

46.

c

17.

e

47.

d

18.

c

48.

d

19.

d

49.

d

20.

a

50.

d

21.

b

51.

b

22.

e

52.

e

23.

a

53.

a

24.

d

54.

c

25.

b

55.

c

26.

b

56.

d

27.

e

57.

a

28.

e

58.

d

29.

e

59.

c

30.

c

60.

c

Capítulo

ECUACIONES DE GRADOSUPERIOR

11

Teorema Fundamental del Álgebra Toda ecuación polinomial P(x) = 0, donde P(x) es un polinomio de cualesquiera coeficiente numérico de grado mayor que la unidad, tiene por lo menos una raíz generalmente compleja. Corolario : Toda ecuación polinomial de grado "n" tiene exactamente "n" raíces. * *

x2 x 5 x

7

x

x1x 3

a1x n 1a 2 x n

2

a 3x n

3

... a n

1 2

1 2

1.

Paridad de raíces imaginarias : Sea P(x) = 0 una ecuación polinomial, donde P(x) es un polinomio de coeficientes reales, si una raíz de la ecuación es el número imaginario a+bi, otra raíz será a-bi.

2.

Paridad de raíces irracionales : Sea P(x) = 0 una ecuación polinomial, donde P(x) es un polinomio de coeficientes racionales, si una raíz de la ecuación es el número irracional : a b /a Q b Q' , entonces, otra raíz será : a b.

0

si sus raíces son : x1 ; x 2 ; x 3 ; ..... x n

5

Teoremas Adicionales :

tiene 7 raíces

Teorema de Cardano - Viette : Dada la ecuación polinomial de grado "n", cuya estructura es :

10 2

x 2x 3 x1x 2x 3

0 tiene 2 raíces

1

ao x n

x1x 2

se cumple :

Ecuación E Ecu uac ua a ccii n d ac de Tercer Grado : (cúbica)

1.

Forma general :

Suma de raíces : x1

2.

x2

ax 3

a1

... x n

ao

x1x 3

x 2x 3

... x n 1x n

a2 ao

Suma de productos ternarios : x1 x 2 x 3

x1 x 2 x 4

... x n

x

x

2 n 1 n

a3

( 1)k .

ak

2x x1 x 2

x3

5x 5 2

2

cx d

0 ... (1)

0

Si en la forma general se sustituye "x" por x

b , se obtiene 3a

la siguiente ecuación : x3

px q

0 .... (2)

cuyo discriminante se denota por D y se define según la relación :

ao

Veamos un ejemplo para la ecuación : 3

a, b, c d R / a

ao

En general, si " s k " representa la suma de los productos de las raíces tomadas de "k" en "k", se cumple :

sk

bx 2

Donde : x = incógnita, asume tres valores :

Suma de productos binarios : x1 x 2

3.

x3

3

p 3

D

q 2

2

Con lo cual las raíces de (2) se obtienen según :

10x 1 0 x1

3

q 2

D

3

q 2

D

x2

3

q 2

D .W

x3

3

q 2

D .W 2

3

Observación : Para resolver una ecuación binomia, se podrá aplicar algún producto notable, cierto criterio de factorización o la radicación de los números complejos.

D .W 2

q 2

Ecuación Trinomia

D .W

Forma general :

3 i/i 2

1 2

siendo : W

q 2

3

1

Observación : Es recomendable utilizar el proceso anterior siempre y cuando la ecuación dada no pueda resolverse por factorización. Ecuación Bicuadrada : Es aquella ecuación polinomial de cuarto grado que presenta la siguiente forma : ax4

bx 2

c

ax 2n

a, b c R / a 0

Donde : x = incógnita, asume "2n" valores.

n N/n 2 a b c R/a

0, b

0

Toda ecuación bicuadrada :

ax 4 bx 2 c 0 , donde "m" y "n" son dos raíces no simétricas presenta por conjunto solución.

Propiedad de las Raíces : Siendo "m" y "n" las raíces no simétricas de la ecuación bicuadrada ax 4 bx 2 c 0 , se cumple :

II.

b a

Ecuación Recíproca : P(x) = 0, será una ecuación recíproca, si P(x) es un polinomio cuyos coeficientes de sus términos equidistantes son iguales.

* *

x

3

5x 5x 4x 4x

4

5x

*

4x5

1.

Siendo "m" y "n" las raíces no simétricas, tenemos: (m2

n2 ) x2

m2 n2

Ecuación Binomia

2

2

2x

0

4x 1 3

3x 4

7x

2

2x 3

0 2x

2x 2

ax n

b

Donde : x = incógnita, asume "n" valores.

b

5

0

3x 4

0

0

En toda ecuación recíproca, se cumple que si : r una raíz, entonces, otra raíz será

0

0

0 es

1 . r

2.

Toda ecuación recíproca de grado impar acepta como raíz a 1 ó -1.

3.

Si : P(x) = 0 es una ecuación recíproca de grado "n", se verifica lo siguiente : P(x)

Forma general :

0

2xx 2 2

*

Reconstrucción de la ecuación bicuadrada en "x":

a b R/a

y

Propiedades :

c a

x4

0

Ejemplos :

CS = {m, -m, n, -n}

m2 . n 2

c

xn

Teorema del Conjunto Solución

n2

0

ay 2 by c 0 Donde los valores de "y" se podrían obtener, según los criterios vistos en la resolución de una ecuación cuadrática, para finalmente resolver la siguiente ecuación binomia :

x = incógnita, asume cuatro valores

m2

c

Observación : Para resolver una ecuación trinomia se recomienda que, en la forma general, se realice el siguiente cambio : x n por "y", con lo cual la ecuación sería :

0

Donde :

I.

bx n

xn . P ( 1 ) x

EJERCICIOS PROPUESTOS 01.

Indicar la suma de la mayor raíz positiva con la mayor raíz negativa que se obtiene al resolver : 36x 4 148x 2 16 a) 0 d) -11/6

02.

b) 11/6 e) -5/6

07.

ax 4

0

a) |b|+ 2a 0 c) |b| 2a 0 e) a + b = 0

Formar una ecuación bicuadrada que tenga por dos de sus raíces : a 2 3 y 5.

280

0

b) x 4

40 x 2

390

0

x

4

37 x

2

300

0

d) x

4

42x

2

280

0

x

4

37 x

2

280

0

c) e) 03.

08.

42x 2

09.

x1 ; x 2 ; x 3 y x 4 .

a) 84 d) 95 04.

"k"

x1

a) 4 d) 2

1

(x 1 x 3 )

b) 4 e) 10

c)

1

2

a) 2 d) 1

x 3 11x 2

b) 0 e) 2

x 12

x5

3x 4

c)

1

6x 3 10x 2

0

b) x

10x

2

31x 30

0

x

3

31x

2

30x 60

0

12x

2

0

15x 30 0

Construir la ecuación con coeficientes racionales de grado mínimo que tenga como raíces los números : 1; 1 + 2 ; 3i. Dar como respuesta el coeficiente de su término lineal. b) 5 e) 12

c) 7

Si : 1-i, es raíz de :

a) 4 i d) 1 + 2i 12.

5 raíces diferentes. 2 raíces de multiplicidad 2. 1 raíz de multiplicidad 2 y otra de multiplicidad 3. 1 raíz de multiplicidad 4. 1 raíz de multiplicidad 5.

3

62x 20x 2 60

4 x 3 11x 2 14 x 10

0

entonces, la suma de las otras raíces es :

21x 9

presenta : a) b) c) d) e)

30x 60

3

x4

El siguiente polinomio : P(x)

10x 2

a) 1 d) 9 11.

0 c) 10

a) x 3

e) x

12

Determinar la suma de las raíces racionales del polinomio : x4

b) 13 e) 2

d) 2x 3

x 3 ; (x 2 x 4 )

2x 2 7 x 2

Formar la ecuación de menor grado posible con raíces: 2, 5 y 3.

c)

10.

P(x )

06.

c) 45

en la ecuación bicuadrada. 0 , si las 4 raíces de la ecuación

ax4 48x 2 k cumplen con :

05.

32 .

b) 85 e) 44

Calcular

Indicar la suma de los cuadrados de los ceros no racionales de la ecuación :

a) 14 d) 5

Si : x1 = 2 y, x 2 . x 3 . x 4

0

b) |b|>|2a| d) |b| + 2a < 0

x3

Indicar la suma de coeficientes de una ecuación bicuadrada de raíces :

bx 3 bx a

tiene dos raíces reales. ¿Qué relación existe entre a y b, sabiendo que : a < 0?

c) 5/3

a) x 4

Si la ecuación :

b) 3 + i e) 4

c) 1

4i

Hallar los valores reales a y b, de modo que : 1-i sea una raíz de la ecuación x 5 ax 3 b 0 . Indicar su suma. a) 8 d) 10

b) 6 e) 7

c) 9

13.

Sean : P(x) : el polinomio de menor grado con coeficientes racionales que tiene a : 3 y 1+i, como raíces simples. Q(x) : el polinomio de menor grado con coeficientes reales que tiene a : 3 y 1+i, como raíces simples. Luego, podemos decir : a) b) c) d) e)

14.

15.

16.

Grado (P) = Grado(Q) Grado (P) < Grado (Q) Grado (P) > Grado (Q) Grado (P) = 3 Grado (Q) = 4

18.

x3

19.

b) a 2

d) a 1 / 3

e) a 2

a)

6x

d)

5x 2

b) 8 x

20.

2

c) 2x

5 x1

a) 0 d) 21

b) 7 e) 10

Dada la ecuación :

2005

17.

2005

b) 2 e) 32

0

(

6

c) 3

4 x 3 24 x 2 mx 18

0

x1 x2 x3 b) 21 e) 27

a) 3

22.

c) 23

k2 1

0

b) 3k 2

c) 0

e) 6

En la ecuación :

x 3 , entonces el

5

c) VVF

ax 2

una raíz es el doble del negativo de la otra, luego, se cumple :

0,

es 1. 2x o 1 III. Si P es un polinomio de quinto grado con coeficientes reales que tiene como raíces a "2i" y a "i", entonces, la gráfica de P corta al eje "x" en un punto. b) FVV e) VFV

2x o xo 1

x 3 ax 2 ax m

II. Si : x o es una raíz de x 3

a) VVV d) FVF

2 x x

)

c) 8

x 3 (m 1)x 2 (3m 1)x 19 entonces, m = 4.

valor de : T

14

(a b)3 (b c)3 (c a)3 6abc abc

Si : x = 1, es una raíz de

2x o3

c)

2x 3 x 1

x3 x Calcular el valor de :

Señale el valor de verdad de las proposiciones : I.

5 x3

x 23

Si, a, b y c; son las raíces de la ecuación :

d) k a) 4 d) 16

5 x2

Calcular el valor de "m", sabiendo que las raíces de :

tiene como conjunto solución { ; } . Calcular :

xo4

a) 18 d) 25

2

Si la ecuación :

2005

x 22

b) 2 e) 5

son :

21.

2x

0.

Calcular : x12

a) 1 d) 4

c) a 1

e) 4 x 2

x2

5

Hallar : E

Formar la ecuación de menor grado posible con coeficientes racionales enteros y de menor valor absoluto, tal que admita como dos de sus "ceros" : 3 - 2 ; i. Indicar el término cuadrático de dicha ecuación. 2

7x

Donde " x o " es una solución :

Si : a1 / 3 a 1 / 3 es una de las raíces de la ecuación x 3 3x c a 0 entonces, el número "c" es igual a: a) 2a

Si : x1 ; x 2 y x 3 son las raíces de la ecuación :

a) 2a = -3m

b) 3a = -2m

c) 4 a 3

d) 27a 3

27 m 2

4 m2

e) 3a = 2m 23.

Si dos raíces de la ecuación : 2x 3 4 x 2 (m2 1)x m 2 Indicar el valor de : m a) 2 d) 1

b) 2 e) 0

0 , suman 3.

5 m c)

1

24.

Si una de las raíces de la ecuación: 3

a) 1 d) 4

2

3x 18x ax 60 0 (a R) es la media aritmética de las otras 2. Calcular la suma de las inversas de estas 2 raíces.

a) 1/5 d) 4/5

b) 2/5 e) 1

31.

b) 2 e) 5

c) 3

Sean : x1 ; x 2 y x 3 ; raíces de la ecuación :

x3 Calcular el valor de :

c) 3/5

x15

E 25.

Sean x1 ; x 2 y x 3 las raíces de la ecuación:

x 3 4ax b 2004 0 ; a 0 Además : x 2 x1 x 3 x 2 . Dar como respuesta una de sus raíces.

26.

b)

d) 2a

e) 2 a

1

c)

32. 2

3px q

3 a) q

2p

c) p 2

q3

e) q

2

4p

0 ; pq 0

3 b) q

0

d) 4 p 3

3

4p2

0

p2

0

Dado : F(x) ax 5

(b ac )x 4 bcx 3 (a bc )x bx bx 2 ac Además : F(c) = 0. Señalar la relación correcta para que las otras raíces sean reales. c) 2b = a + c 34.

Si : a, b y c; son raíces de la ecuación :

6 x 2 7x 1

Calcular : a 2

b2

a) 14 d) 5

b) 30 e) 2

35.

K

x 23

30.

b) 5 e) 4

a) 1 d) 9

b) 3 e) 12

c)

36.

En la ecuación polinomial :

2

x 23

x3 5

0

c) 1/3

(ab 1)x 2 (a

b) 1 e) 4

b)x

ab

0

c) 2

5 1 es una raíz de la ecuación : 2

0. b) 4 e) 7

c) 5

Si dos raíces de la ecuación :

Ax 4

Bx 3 Cx 2 Dx 30

de coeficientes racionales son : 1 i

Calcular el valor de :

x 3 (m 2) x 2 (m 2 3)x m3 2 0 de raíces : x1 , x 2 , x 3 . Calcular el valor de "m", de tal manera que la 2

b)x 3

Calcular, b a, si :

35;

expresión: A x1 x 2 tenga el máximo valor.

(a

3x 5

5

x2 3

n

¿Cuántas raíces no numéricas presenta la ecuación en "x" ?

a) 3 d) 6

adopte su mínimo valor. Siendo :

x1 , x 2 , x 3 , raíces de la ecuación. a) 2 d) 3

1 (x 3 1)2

x7 ax b

2

x (n 3)x (n 1)x n 1 0 Calcular el valor de "n", tal que la expresión: x 22

92x

c) 136

Siendo : f(x )

a) Ninguna. d) 3

A partir de la ecuación polinomial :

x12

kx 2

b) 240 e) 102

x4

c) 43

2

175

Las raíces de la ecuación : x 3 3 x 1 son, a, b, c. Calcular : S = f(a) + f(b) + f(c)

0

c2 .

3

c)

Hallar el valor de : k + n.

33.

2x 3

x 53

Las raíces de :

a) 138 d) 156

0

0

165 200

x1 1

d) | b | 2| a | e) | a | 2b

29.

b) e)

x 52

P(x) x 3 están en la relación :

a

0 , tenga una raíz doble.

a) 2c = a + b b) 2a = b + c

28.

155 180

Hallar la relación entre "p" y "q", para que la ecuación:

x3

27.

a

a) 2004

a) d)

5x 7

BC AD 3

E a)

2

d) 2 2

b) e) 6

3

2 c) 2 3

0

37.

Si "P" es un polinomio completo definido por : x5

P(x )

43.

... 15 y la ecuación : 3

P(x) = 0 tiene a

5 como raíces, entonces, 2 i

3 ya

ab(a 2b 2 1) bc(b 2c 2 1) ca(c 2a 2 1) 5a 3 abc

la sexta parte del resto de dividir. P(x) entre (x 3

3x 2

b) x 2

4x 6

c) x 2

4x 1

d) x 2

4x 5

2

4x 5

e) x 38.

3) es :

a) x 2

a) d) 44.

2x 3

0

e) 3x 2

2x 4

0

d) 4 x

10 13

b) e)

11 14

Hallar los valores de "

x4

3

24 x 1

0

17 x 10

0

c)

45.

41.

b) 3 e) 2

12

f(x ) a) 5 d) 6 42.

x5

b) 6 e) 4

Calcular el valor de : (a b)2 (b c)2 (c a)2 2 3 (m) (n) 3 2 a) 4 d) 12

47.

b) 27 e) 108

c) 54

0

c) 2

2

e) 5

Indicar una raíz de la ecuación : x3

3ax 2

3(a 2

bc )x

a3

b3

c3

3abc

0

siendo : a, b, c, R . a) a+b+c d) a+b-c 48.

a 3 ma n 0 b3 mb n 0 c 3 mc n 0

c) 1/3

2x 2 bx 4

b)

3

d) 3

5

Sabiendo que :

b) 5/6 e) 17/6

En la ecuación polinomial :

a)

ax 1 c)

x 3 ax 2 bx 2

el cuadrado de la única raíz positiva es igual a la diferencia de los cuadrados de las otras dos. Señalar dicha raíz.

c) 8

ax 2

6x 4

x3

0

Determinar el coeficiente "a", de tal modo que el número (-1) sea una raíz múltiple de orden no inferior a 2 del polinomio.

c) 7,5

Si la ecuación :

a) 1 d) 1

tenga sólo 2 raíces reales, dar como respuesta el mayor valor entero negativo que asume " ". a) 4 d) 1

b) 9,5 e) 13,5

Calcular la suma de las raíces racionales.

", para que la ecuación :

6)

0

0 ; admite dos raíces imaginarias conjugadas : m+ni; m-ni; tal que la suma es 3 y dos raíces racionales.

46.

) x 2 (2

(1

c) 1

Calcular la suma de los valores que admite "a", para que la ecuación :

a) 2 d) 2

¿Qué valor debe asumir "n" para que las raíces de la ecuación : x 4 nx 2 9 0 ; se diferencien en una constante "K" ? a) d)

40.

b) 48 x 3

0

3

c) 5x

39.

4

b) 2 e) 0

3a 2

admita dos soluciones.

2x 3 3x 2 7 x 1 0 en otra cúbica que carezca de término cuadrático. 2x

2 1

x 3 (1 a) x 2 (7 2a) x 18

Transformar la ecuación cúbica :

a) 3x 3

Si el conjunto solución de la ecuación : 5x 3 3x 2 5 0 ; es {a; b; c}. entonces, el valor de :

b) a b+c e) a b c

c) a+b c

¿Cuál es la relación que deberá existir entre a, b y c; para que las ecuaciones : ax 5 bx c 0 ; cx5 bx a 0 ; a c , tengan sólo una raíz común? a) (a b) (a c) (a d) (a e) (a

b)5 c 5 b)5 c 5 b5 0 b)5 c 5 0 c)5 b5 0 c)5 b5 0

49.

Si las ecuaciones :

ax

Siendo, además : m + n = m . n = 10.

3

3bx

3

3cx d

bx

2

c

0

a) 0 d) 1

0

tienen una sola raíz común, calcular : a) 9 d) 32 50.

b) 12 e) 36

(ad 4 bc)3 (ac 2 b2d)2

56.

ax3

a) 19/4 d) 13/4

b) 17/4 e) 15/4

0; a

0

Se concluye :

0

sabiendo que son reales, positivas y que: r1 r2 r3 r4 1 2 4 5 8 Señalar la suma de dichas raíces :

Dada la ecuación :

2x a Se afirma : I. Tiene dos raíces reales. II. Tiene raíz negativa. III. No tiene raíces reales.

c) 27

bx 2 cx 5

c) 8

x15

Hallar las raíces r1 , r2 , r3 , y r4 de la ecuación :

4x4

b) 3 e) 2

a) FFF d) VVF 57.

Si :

1

b) VVV e) FFV ;

2

;

3

; .....

n

c) FVF

son las raíces de la ecuación

polinomial.

c) 21/4

xn

2x 2 3x 1 0

Proporcionar un valor de : 51.

Si las raíces : x1 , x 2 , x 3 , x 4 , de la siguiente ecuación: 4

3

2

2x 2

3x 3

x mx nx px 86 864 6 son reales y positivas; además : x1

4x 4

t

0

48 8

Dar como respuesta la suma de raíces. a) 3 d) 25

b) 9 e) 36

c) 17

58.

n k 1

(

a) - 1

b) 2n - 3

d) 2n - 1

e) 2n + 1

K

2) c) 2n + 3

Sea la ecuación polinomial :

3x125 2x100 4x75 2 0 Determinar el valor de :

52.

53.

Sean : a y b dos números reales para los cuales, la ecuación : x 4 ax 3 bx 2 ax 1 0 tiene al menos una solución real. Para todos los pares 2 b2) . (a; b), encontrar el máximo valor de : (a

Si : S es la suma de todas las multiplicaciones de las m

a) 3/4 d) 1/2

a) 1/3 d) 4/3

b) 4/5 e) 2/3

x 6 18 2 x 3 2

d) 4 2 54.

b) 2 2

0 c) 3 2

¿Cuántas raíces reales tiene la ecuación?

a) 1 d) 4

59.

2x 1 0

b) 2 e) Ninguna.

c) 3

Dada la ecuación : x5 x4 x3 x 2 x 0 Calcular la suma de sus raíces, si dos de ellas son "m" y "n", m n.

S125

60.

b) 0 e) 7/3

c) 8/3

Si el polinomio : P(x) x 3 ax 2 x 2 es divisible por (x+2). Entonces, el producto de las raíces racionales de la ecuación : P(2x 3 2 P(x)) 0 es : a) 0 d) 3

e) 5 2

x4

55.

64

S 50

raíces tomadas de "m" en "m".

c) 7/8

Indicar la suma de las raíces no imaginarias, de la siguiente ecuación :

a)

S 25

b) 5 e) 2

c)

2

Sea "P" una función polinomial definida por : P(x) 2x 7 ax 5 bx 3 1 , donde "a" entero positivo y "b" entero negativo. Entonces, indicar el valor de verdad de las siguientes proposiciones : I. Una raíz de ecuación : P(x) = 0 es 1/3. II. La ecuación P(x) = 0 tiene una sola raíz real. III. Si : x1 a) VFV d) FVF

x 2 , entonces : P(x ) 1 b) VVV e) FFV

P(x 2 ) c) VVF

Claves 01.

a

31.

c

02.

c

32.

a

03.

c

33.

d

04.

a

34.

c

05.

d

35.

c

06.

c

36.

d

07.

d

37.

e

08.

a

38.

d

09.

b

39.

a

10.

d

40.

d

11.

b

41.

c

12.

d

42.

e

13.

c

43.

b

14.

c

44.

a

15.

b

45.

e

16.

d

46.

b

17.

b

47.

e

18.

d

48.

e

19.

b

49.

c

20.

c

50.

a

21.

a

51.

d

22.

c

52.

b

23.

c

53.

c

24.

b

54.

b

25.

c

55.

c

26.

e

56.

c

27.

d

57.

d

28.

e

58.

c

29.

e

59.

c

30.

b

60.

d

Capítulo

12

MATRICES - DETERMINANTES

MATRICES

Ma tr i ces E s pe ci al e s

Definición : Una matriz es un arreglo rectangular de elementos dispuestos en filas y columnas. Para representar a una matriz, se utiliza letras mayúsculas.

1.

M. Fila : Es aquella matriz que tiene una sola fila. *

2. Ejemplos :

[1

5

7

10]

M.Columna : Es aquella matriz que tiene una sola columna. 2

*

*

A

2 0

3 1 1 2

1 5 4

B

Fila

c o l u m n a

*

3.

0 3 1 1 2 0 4.

3 3

a 21 A

a 31

a12

a 22

a m1 a m 2

a13

a 23

A

A

a 2n a 3n a mn

1

2

3

4

2

1

2 4 1 7

M. Nula : Es aquella matriz, donde todos sus elementos son iguales a cero. *

a1n

A

M. C M Cuadrada : Es aquella matriz, donde el número de filas y el número de columnas son iguales. *

5.

Fo r m a Ge ne ra l de una Mat r i z d e " m " f i l as y " n" Columnas : a11

M. Rectangular : Es aquella matriz, donde el número de filas y el número de columnas son diferentes. *

A es una matriz de orden 2 B es una matriz de orden 3

5 7

Orden de una Matriz Viene dada por la representación m n , donde "m" es el número de filas y "n" el número de columnas de la matriz. Para los ejemplos citados anteriormente, tenemos : * *

A

4

*

A

0 0 0 0 0 0

0 0 0 0 0 0 0 0 0

m n

Igualdad de Matrices : Donde : a ij es el elemento genérico, ubicado en la fila "i", columna "j". En forma abreviada se tendrá :

A

[a ij ] m

i = 1, 2, 3, ......, m = 1; m n

j = 1, 2, 3, ......, n = 1; n

Dadas las Matrices :

A

[a ij ]m

n

B

[bij ]m

n

si estas son iguales, es decir : A = B, se verifican simultáneamente las condiciones :

I. II.

A y B son de igual orden : m n . Los elementos correspondientes son iguales :

a ij

bij ;

A .B

i; j *

Oper aciones con Matr ices I.

se define : [ A11.b11 a12.b 21 a13

b 31 .. a1n.b n1]

Multipliquemos A por B, donde :

Adición : Dadas las matrices de igual orden

A

[a ij ]m

B

n

[bij ]m

n

bij ]m

n

A

[2 1 3]

B

6

se define : A B

[aij ]m

[bij ]m

n

[aij

n

A.B *

A

2

1 3

0

1 2

B

6

A

A . B = [(2).(2)+(1).(4)+(3).(6)]

1 2 5

B

1 4

2

1 3

1 2 5

0

1 2

1 4 3

(2 1)

A B

II .

2 [2 1 3] . 4

Hallar la matriz A + B, a partir de : A

(1 2)

3

A . B = [4+4+18]

Dadas las matrices A y B, existe el producto matricial de A por B denotado por A.B, si se verifica lo siguiente :

(3 5)

# de columnas de A = # de filas de B

3 3 8 1 3 5

luego : Am

Multiplica ción : II.1 Multiplicación de un escalar por una matriz.

Sean : A

[a ij ]m K. A

n

A

K .[a ij]m

n

[K .a ij]m

n

2 1 4 1 3 2

2.A

2A

2.

4

2 1 4 1 3 2

p

.Bp

n

Cm

n

* Veamos un ejemplo :

k R , se define:

* Multipliquemos por 2 a la matriz. A

A . B = [26]

III.3 Multiplicación de las Matrices

(0 1) ( 1 4) (2 3) B

2 4

2

1

3

1

B

2

2

5

1

2

3

¿Existe A . B?, veamos : A tiene orden 2 2 B tiene orden 2 3

# col = 2 # fil = 2

como : # col de A = # fil de B se afirma que si existe A . B, cuyo orden es de 2 3.

2 8

A .B

2 6 4

2

1

3

1

.

2 1

2 5 2

3

Ahora se multiplica de forma similar que el caso (II.2). II.2 Multiplicación de una matriz fila por una matriz

columna. A.B

Sean : A

[a11 a12 a13 .... a1n ] b11 b 21

B

b 31 b n1

A.B

A.B

(2).(2) ( 1).(1) (2).( 2) ( 1).(2) (2)(5) ( 1)( 3) (3).(2) (1)(1) (3)( 2) (1).(2) (3)(5) (1)(3)

4 1 6 1

3 7

4 6

2 10 3 2 15 3

6 13 4 12

¿Existe B.A?, veamos : # col de B = 3 y # fil de A = 2 como # col de B fil de A, se podrá afirmar que B.A no existe.

#

En General : El producto matricial no es conmutativo.

Propiedades :

Teoremas : Sean A, B y C matrices para las cuales se define la adición y/o multiplicación, además al escalar "k".

Siendo A y B matrices, y el escalar "K". 1.

(A

1. 2. 3. 4. 5. 6.

2.

(K.A)T

K.A T

3.

(A T )T

A

4.

(A.B)T

BT . A T

K . (A+B) = K . A + K . B A+B=B+A A . B . C = (A.B).C = A.(B.C) A.(B+C) = A.B + A.C A.B = 0 no implica A = 0 B=0 A.B = A.C no implica B = C

Propiedades : Sean las matrices A y B, de modo que existen A.B y B.A.

A

Potenciación : Siendo A una matriz cuadrada y "n" un entero positivo, se define : ; n=1 A A.A.A. ..... . A ; n 2

An

Hallar A 2 , si : A

A2

A2

A

2

A .A

2 3

2

1

3

1

1 2 . 1 3

4 3

2 1

1.

1 1

2.

6 3

3 1

A

1

3

9

2

Transpuesta de una Matriz Dada una matriz A, existe su matriz transpuesta denotada por A T y definida como aquella matriz que se obtiene al transformar todas las filas de A en columnas. A *

[a ij ]m

A

n

3.

(3).( 1) (1).(1) 2

T

a11 a12 a13

a1 n

a 21 a 22 a 23

a 2n

a 31 a 32 a 33

a3n

a n1 a n 2

a nn

DP

[a ji ]n

m

Toda matriz cuadrada de "n" filas y "n" columnas es de orden "n". La diagonal trazada de izquierda a derecha recibe el nombre de Diagonal Principal (D.P.). La diagonal trazada de derecha a izquierda recibe el nombre de Diagonal Secundaria (D.S.).

Traza de A (Traz(A)) Se denomina así, a la suma de todos los elementos de la diagonal principal. Traz (A) *

a 11

a 22

a 33 ... a nn

Para la matriz

A

2

5

1

8

7

1

0

4

Veamos un ejemplo :

A

2 0

1 4 1 5

AT

n n

Observaciones Ob O bs bs :

(2).(2) ( 1).(3) (2).( 1) ( 1).(1) (3).(2) (1).(3)

BT

DS

"n" veces

*

AT

Estudio de las Matrices Cuadradas

1. Si : A.B = B.A, se dice que A y B son matrices conmutables. 2. Si : A.B = -B.A se dice a A y B son matrices anticonmutables. III.

B)T

4

D.P. 2 1

0 1

4

5

Traz(A) = (2)+(8) + (-4) Traz(A) = 6

Propiedades :

C ar a cte r í st i cas Cuadradas :

N ot a b le s

de

a lgunas

Mat r i ce s

Siendo A y B matrices y el escalar "K". 1. 1. 2. 3.

Traz (A+B) = Traz(A) + Traz(B) Traz (K . A) = K . Traz (A) Traz (A . B) = Traz (B . A)

Matriz Simétrica : Si A es una matriz simétrica, verifica : AT

A

Matr ices Cuadradas Especi ales 2. 1.

M. Diagonal : Es aquella matriz no nula, donde todos los elementos fuera de la diagonal principal son ceros. Ejemplos :

*

A

2 0 0 0 1 0

Mat r i z Anti s i mé t r ica : Si A es una matriz antisimétrica, verifica : AT

3.

-A

Mat r i z I de m pot e nte : Si A es una matriz idempotente, verifica :

0 0 3

* 2.

4.

A

I

M. triangular Superior : Es aquella matriz donde solamente todos los elementos ubicados debajo de la diagonal principal son ceros. Ejemplo : 5 4 *

A

0

2

*

0 ; (matriz nula)

p : índice de nilpotencia. DETERMINANTES ERM R M I NAN RM Definición : Un determinante es la relación funcional que aplicada a una matriz cuadrada la transforma en un escalar (número real). Si A es una matriz cuadrada, su determinante se denota así : det(A) o |A|. Determinante de Orden Uno A

a

b

c

d

|A|

a b c

|A| = a . d - b . c

M. Triangular Inferior : Es aquella matriz donde solamente todos los elementos ubicados encima de la diagonal principal son ceros. Ejemplo :

A

Matriz Nilpotente : Si A es una matriz nilpotente, verifica :

1 7

0 0 4 5.

I ; (matriz identidad )

Ap

1 0 0 0 1 0 0 0 1

A

Matriz Involutiva : Si A es una matriz involutiva, verifica : A2

5.

4 0 0 0 4 0 0 0 4

M. Identidad (I) : Es aquella matriz escalar donde todos los elementos de la diagonal principal son iguales a la unidad. Ejemplo :

*

4.

3 0 0 0 5 0 0 0 0

M. Escalar : Es aquella matriz diagonal donde todos los elementos de la diagonal principal son iguales. Ejemplo :

*

3.

B

A2

3 1

0 4

0 0

2

1

8

Deter minante de Orden Tres :

A

a b c d e f g h i

d

Según, la Regla de Sarrus :

c.e.g +

f.h .a i.b.d

a

b

e

d

e

f

g

h

i

a

b

c

d

e

f

Teorema : El determinante de una matriz será igual a la suma de los productos obtenidos al multiplicar todos los elementos de una fila (o columna) por sus respectivos cofactores. a.e.i d.h.c g.b.f

+=N

Para : +

A

+=M

2 1

1 5

3 2

3

2

1

con los elementos de la primera fila :

|A| = M - N

| A | 2.

Menor Complementario de una Componente El menor complementario de la componente (elemento) a ij denotado por M es el determinante de la ij matriz que resulta al eliminar la fila "i" y la columna "j" de la matriz dada.

5

2

2

1

1.

1

2

3

1

( 3) .

1 5 3 2

|A| = (2)(9) - (1)(7) + (-3)(-13) |A| = 18 - 7 + 39

| A | 50

Para : 2 A

-1

5

4 3

1

-2

3

2

el menor complementario de a12 M12

5 2 1 3

M12

15 2

M12

13

Observación : Para aplicar el teorema anterior, se recomienda escoger la fila (o columnas) que presente más ceros. Propiedades : Dadas las matrices cuadradas A y B, y el escalar "K".

4 es :

(5).(3) (1).(2)

1.

|A . B| = |A| . |B|

2.

| AT | | A |

3.

| K . A | K n .| A | ; "n" orden de A.

4.

Si dos filas (o columnas) son proporcionales, el determinante será igual a cero. Si todos los elementos de una fila (o columna) son ceros, el determinante será igual a cero. Si se permutan dos filas (o columnas) consecutivas, el determinante cambia de signo. El determinante no varía si a todos los elementos de una fila (o columna) se les aumenta un múltiplo de otra. El determinante de una matriz triangular superior, triangular inferior y diagonal se obtiene multiplicando todos los elementos de la diagonal principal.

5.

Cofactor de una Componente El cofactor de la componente (elemento) a ij denotado por A ij , se define de la manera siguiente :

A ij

( 1)i j . M ij

6. 7. 8.

Para : 2 A

-3

5

1 -1 2 3

2

Deter minante de Vander monde

4

1.

1 1

el cofactor de la componente a13 es :

A13

( 1)1

3

A13

(1) .[(1).(3) (2).( 1)]

C13

3 2

C13

5

M13

( 1)4 .

De orden dos :

a b

1

1

2

3

2.

b a

De orden tres :

1 a a2

1 b b2

1 c c2

(c b)(c a)(b a)

3.

De orden cuatro : 1

1

1

1

a a2

b b2

c c2

d d2

a3

b3

c3

d3

(d c)(d b)(d a)(c b)(c a)( b a)

Cálculo de Matrices Inversas 1.

De orden uno

A

Definición : Una matriz cuadrada A es no singular, si : |A| 0, asimismo, si : |A| = 0, la matriz A será singular..

2.

[a]

A

[1] ; a a

1

0

De orden dos A

a

b

c d

A

1

1 . |A|

d

b

c

a

MATRIZ INVERSA Dada una matriz cuadrada no singular A, si existe una única matriz B cuadrada del mismo orden, tal que : A . B = B . A = I (matriz identidad), entonces, definimos B como matriz inversa de A y lo denotamos por A 1 . Teorema : Una matriz cuadrada tiene inversa, si y sólo si, es una matriz no singular; en tal caso se dice que la matriz es inversible. Propiedades : Sean A y B matrices cuadradas no singulares y el escalar "K". 1

A 1. A

1.

A. A

2.

(A . B)

1

B 1. A

3.

(A 1 )

1

A

4.

(K . A)

5.

|A 1| |A| 1

1

K 1. A

I 1

1

1 |A|

Observación : Para matrices de orden mayores o iguales a tres se recomienda utilizar el método de Gauss-Jordan, el cual consiste en construir una matriz ampliada (A I) donde por operaciones elementales debemos encontrar otra matriz ampliada (I B), con lo cual se podrá afirmar que B es la inversa de A, es decir : B A 1 .

EJERCICIOS PROPUESTOS 01.

Escribir explícitamente la matriz "A". A

[ A ij ]2

3

/

a ij

ij ; i

a ij

i

j

a)

j;i

j

7 0 1 1 7 3

d)

1 4 3 5 1 3

a)

1 4 3

d)

02.

1 3 4 3 4 5

b)

3 4

2

06.

3 4 6

4 x 9y 5x 18 x 2y

donde se cumple :

a 22

0

3 3

2

7

0

1

9 1 0 2

2

1

3

3

2

4

1 1 B

2 3 1 2

Hallar : A×B.

A

2 a 21

e)

c)

Dados :

A

Dada la matriz :

a12

7 9

1 3 4

1 3 4

e)

5 1 2

c)

9 2

b)

a)

0 1 2 4

b)

1 3

1 6

d)

1 5

e)

2 1

2 5

1 6

c)

1 3

1 5

c)

3 4 1 0

Calcular : x + y. 07. a) 5 d) 7 03.

b) 9 e) 6

m n 2p q m n p q

3 5 1 4

Hallar : (m - p) + (2n - q).

04.

b) -3 e) -2

c) 2 08.

Calcular : 3B T

a)

d)

05.

Calcular : A 2

A.

a)

6 0 4 2

b)

0 12 0 5

d)

1 12 0 5 2

e)

5 0 0 1

Hallar la suma de los elementos de "x", tal que :

Dada la matriz :

B

13 0 4

16 5 9

9

a) -2 d) 3

6

13 15 c) 3 7

09.

9

3 1

1 3

9 2 0 1

10.

c) 1

8 2 7 2

P(x;y) = 3x - 2y + 2

b) 1 e) 9

c) 2

Luego de resolver la siguiente ecuación :

Si :

Hallar : P(A; B).

b) 0 e) 5

Hallar la matriz inversa de :

a) 5 d) 10 y B

2 5 4 0

Señalar la traza de dicha matriz inversa.

6

Dados :

A

2 1 2 1

A

18 15

e)

2

5 2

I.

b)

7

x

4 1

15 13

4 0 1 2

A

Si :

a) 4 d) 3

Dada la matriz :

c) 8

5

1

2

x

3

8

1

1 x

28

indicar su solución : a) 1 d) 4

b) 2 e) 5

c) 3

11.

16.

Se define la siguiente regla : a P(a, b, c )

Hallar el valor de :

b c

2 0 1

a b a 1 1 1

E

3 4 1

b 0 a

A partir de ella, calcular : P(-2, 0, 1). a) a + b a) 16 d) 21 12.

b) 19 e) 22

c) 20

1

2

x

17.

x

Si :

18.

a) -2 d) 1

2

b) -1 e) 2

Luego de resolver la siguiente ecuación : x

0

0

5

x

1

8

2

1

a) 5 d) 3

c) 0

19.

a) 1 d) 4

2 x

c) 6

2

1

x

1 2

y

1

0

0

2

z

0

y 3 =0 0 z

b) 4 e) 11

3 1 1 1

0

x z 1

Además : a + b + c = 18. Calcular :

a c b 3 1

. a) 6 d) 12

c) 3 20. y B

b c

4 5 6

x

4 2 0 3

Si :

b) 13 e) 18

; y

c) -6

son las raíces de la ecuación :

x 3 5x 3

0

Calcular el determinante de :

| A.B | |B| |A| b) 2 e) 5

0

Si se sabe : a

Dadas las matrices :

Hallar :

3

1 2 3

3

A

3

b) -5 e) -7

Dada la ecuación :

a) 2 d) 5

c) 16

Indicar el producto de soluciones.

1 d 1 b

se pide calcular el valor numérico de :

15.

b) 9 e) 36

2

Hallar el valor de :

14.

" y " " son las raíces de la ecuación :

a) 4 d) 25

c) 3

2 a b 2 c d

b

1

b) 2 e) 5

a b c d

c) ab 2

Calcular el determinante de :

0

Indicar la suma de cuadrados de las soluciones.

13.

"

2

x 2 4 x 31 0

3

a) 1 d) 4

e) a

d) ab - 1

Luego de resolver la siguiente ecuación : x

b) a - b

c) 3

a) 0 d) 4

b) 1 e) 7

c) -1

21.

Construir la matriz : A

[ a ij ] 3

i

3j

a)

3 4

4 7

4 7

a) 2 1 6 7

b) 5 1

c) 5 8 6 9

6 7

3 4

8 9

0 8

14 24

e)

26.

2y x

3y x

3y

x

9

y

2x 2x

8

a 21 a 31

TRAZ(A) = 16 Calcular : "x.y". a) 6 d) 3

b) 4 e) 7

7

a 22 1 .

27.

2x 3 y 4 x 2x 12 y 6

9 10

2 1 0 1

x 2 5x 2I .

b) -6 e) -8

c) -4

Sean las matrices :

a)

a12 y TRAZ(A) = 6. c)

x.y . x y

2 3 1 2

B

1 4

2 3 1 2

14 9

1 12 0 7

14

1 10

9

28.

1

c) 3 A

1 12

9

0

1

2

2

1

2

1

1

1

0

Hallar la traza de A 2 .

Sean las matrices :

x 2y

x

3

x y

; B

a) 7 d) 4

2 y 4 3

4 29.

Hallar : "x.y", si : A = B. b) 10 e) 14

b) 2 e) 5

c) 3

Hallar la matriz "x", que cumpla:

2 5 X 1 3

c) 8

Sean las matrices :

Hallar : 3A - 4B.

14

d)

Dada la matriz :

1 e) 2

A

0

11 0 12 9 0 7

b)

e) N.A.

4 b) 3

a) 6 d) 12

5

6

Hallar : A.B.

Se cumple : a 21

A

22

Dada la matriz : A

A

c) 5

A

d) 2

0

a) 8 d) 6

Si en la matriz :

a) 4

4

d)

13

16 12

Dar la suma de elementos de P(A) :

y

donde se cumple :

25.

12

4 5

b)

Además : P(x) x

24.

8

c)

Sea la matriz :

Calcular :

32

6 7

e)

A

23.

24

4 5

d) 4 5 5 6 22.

/ a ij

2

4 2

6 1

Indicar : TRAZ(X).

4 2 3 1

yB

3 7 1 2

a) 2 d) 10

b) 5 e) -2

c) -17

8

30.

Hallar la matriz inversa de :

35.

2 : n2 : n3 : n4 : .. cuya razón es k 2 ; se cumple en

6 4 3 3

A

Sea la progresión geométrica : ella que la suma de los cuatro primeros es igual a 80.

(k R ) . Además se tiene el siguiente resultado: 1 2

a)

2 1 2

d)

31.

2

1

b)

1 3

1

3

2

c)

4 1 2 1 2

e)

1 3

1 2

2

1 2

1 2 1 3

a 1 b 2

2 3

a) 1 d) 4

x 0 0

x

0 0

x 5 7

0

4

y 0

0 y 5

y 0

0 0

z

k2 2k 2

ak 3 bk 3

k3 2k 3

Hallar : 2a - b.

1

Se define la siguiente función : F(x,y,z)

ak 2 bk 2

ak k bk 2k

8 9

z

0 0

36.

z

b) 2 e) 6

Si :

; y

x3

4x 3

c) 3

son las raíces de la ecuación :

0 . Calcular el determinante de :

A partir de ella, calcular : Q(1, 2, 4). a) 16 d) 15

32.

b) 18 e) 23

c) 24

a) 0 d) 4

Sabiendo que :

a 2

b 3

a 6

33.

34.

37.

b) 16 e)128

c) 32

x 1

x

x

x

x 1

x

x

x

x

3

a) 17 d) 20 39.

c) 0

k

3

1

1

k 1

1

k 2 k

b) 29 e) 4

c) 6

8 3 .A 2 1 a) 3 d) 25 40.

0 2

Calcular el |A|, si :

1 2

Donde "X" es una matriz cuadrada de orden 2. Hallar : Det(X). b) 7 e) 19

2

c) -4

4 0

b) 1 e) 2ab

Si : k N , obtener "3k + 5", sabiendo que :

0

b) -5 e) -3

1 2 .X 3 7

a) ab d) 4ab 38.

A partir de la ecuación matricial :

a) 6 d) 8

Calcular :

a b a b a b a b

Resolver la ecuación :

a) -6 d) 3

c) -1

32

4a b 4 1

Calcular el valor de :

a) 8 d) 64

b 5

b) 1 e) 7

4 2 2 5

b) 9 e) 36

c) 8

Hallar "x" en :

c) 11

a

a

x

m m m b

x

0

b

e indicar uno de sus valores. a) a d) 1/b

b ab e) m

c) 1

120

41.

Escribir explícitamente la matriz : [c ij ] k 2

C

a)

d) 42.

1 1 1 2 1 2 1 2 3 1 1 3

3

/ c ij

b)

1 2 3 2 2 3

e)

1 1 1 2 2 2

46.

1 2 3 1 2 3

A

donde se cumple que : A 2 Hallar : "xy".

47.

B

3 2

2

3

9

1

a)

7

0

4 7 d) 3 0 43.

b)

2

5 2 1 2

9 4 3 4

e)

y

A

1 4

c)

3 4 3

a) 6 d) 4

9 2

48.

7 2

1

x

8

b) 5 e) 2

Si : A

1 1

2 0 1

y

4

1 1 0

z

6

b) 13 e) -4

49.

c) 6

1

2

2

1

2

2

1

1

0

d)

1 2

e) 4

c) 16

3 0 1 2

50.

a) 3. 2n

b) 5. 2n

d) 2n

e) 5. 3n

c) 2. 3n

Sea : 1 0 a 0 b 0 0 c

; con : a, b y c, enteros positivos, se

0

sabe que la segunda columna de : 1,

3 1 a) 1

2.

n Calcular la suma de elementos de " A ".

c) 3

2 1 1 b) 2

3x

Dada la matriz :

A

b) 2 e) 5

Hallar la traza de A

x2

b) 14 e) 11

A

Hallar la traza de A 2 .

45.

x

c) 3

2 y F(x ) 2

a) 2 d) 18

Dada la matriz :

a) 1 d) 4

3 2y

Hallar la suma de elementos de la diagonal principal de F(A).

0 1 2

A

y 5

a 21 . Calcular "x".

Hallar : (x + y + z).

44.

2x

9

7

Si :

a) 11 d) 7

6

donde se cumple : Traz(A) = a13

3 2

c) 180

Sea la matriz :

,y

donde "x" es : 7 3

B.

b) 140 e) 160

x

x 2y A x y B

4 3

21x 48 16 y 1

B

a) 200 d) 130

Sean :

0 1

12 3 1 4

A

Máx (i , j)

c)

Sean las matrices :

si :

.A

1 2 1 1 c) 0

B

A2

3 A T es : 2 . 6

Calcular : a + b + c. a) 3 d) 6

b) 4 e) 7

c) 5

51.

Sea la matriz :

H

x2

3

x

1

a) 2; -2; 4 d) 1; 0; -1 , tal que x > 0 y Det(H) = 4.

56.

Luego H 2 es :

a)

1

3

1

16 4

3 1

x3 1 0

2 3 4 4

c)

2 2

6 2

2 4

e)

2

1 2

a) d)

2 0 1 52.

Sea : B

A 4 , donde : A

6 4

8

8

e) 2 .10

d) 2 .10 53.

57.

Hallar "x", a partir de :

54.

55.

3 2 5

a) 1 d) 7 x2

1 x

x 1 x2

1

x

1

1 x

1

x

1 1

1 1

1

b) -1 e) 2

58.

x2

Si : F(x )

b

2c

a

2b

c

2a

b

c

a)

abc (a b)

b)

abc (a b)

d)

5abc (a b)

e)

3abc (a b)

x

ab

c

a b

c)

59.

2

1

1

Además : I

matriz identidad.

1 4

x

2

1

3

x 10

1

1

22

b)

c)

4

22

d)

e)

0

4 2

11

11

Sea "A" una matriz definida por : A

Además :

0

x

2

Hallar los valores propios de la matriz "A", si :

1

, donde :

c) 1

a)

la ecuación : |A-xI| =0.

1

I)

Resolver : 3

60.

3 3

b) 2 e) 0

5abc (a b)

propios de la matriz A", a los números "x" que satisfacen

c) 5

B

Dada una matriz cuadrada "A", se denomina "valores

A

3 3

3.

5x

c) 0

a

2

4

9

b) 3 e) 9

a) 4 d) 5

2

3 x

Encontrar el determinante de F(B

Calcular "x" en :

2

7

4

Resolver :

a) -2 d) 1

c) 3

e) 0

c) 24.102

b) 2 .10 16

1

b) 4 2

Entonces, el determinante de "B" es : a) 2

1

3

0 3 5

2

2

1 2

d)

c) 4; 5; 1

Si : " " es raíz de la ecuación : Hallar el valor de :

b)

1

b) 3; 2; 1 e) 3; 2; -2

a b c

2a

2a

2b

b c a

2b

2c

2c

c a b

Si : a+b+c = 3, entonces el valor del det(A) es : a) 27 d) -9

b) 9 e) -27

c) 0

Claves 01.

b

31.

c

02.

e

32.

b

03.

c

33.

e

04.

c

34.

d

05.

d

35.

c

06.

c

36.

a

07.

d

37.

d

08.

e

38.

a

09.

a

39.

c

10.

a

40.

a

11.

a

41.

b

12.

e

42.

a

13.

e

43.

c

14.

c

44.

a

15.

d

45.

e

16.

b

46.

b

17.

c

47.

d

18.

c

48.

c

19.

c

49.

c

20.

a

50.

d

21.

c

51.

d

22.

c

52.

e

23.

c

53.

d

24.

d

54.

c

25.

e

55.

e

26.

e

56.

e

27.

a

57.

b

28.

a

58.

e

29.

d

59.

c

30.

e

60.

e

Capítulo

13

SISTEMA DE ECUACIONES

SISTEMAS LINEALES Forma General : Consideremos un sistema lineal de "m" ecuaciones con "n" incógnitas.

a11x1 a12x 2

a13x 3 .........

a1n x n

a 21x1

a 23x 3 .........

a 2nx n

b2

x 3 ... a mn x n

bn

a 22x 2

a m1x1 a m 2x 2

a m3

Resolución de un Sistema lineal según el Método de Cramer : Dado un sistema lineal de "n" ecuaciones con "n" incógnitas :

b1

Donde :

a12x 2

a13 x3

a 21x1

a 22x 2

a 23x3 ....

a n 1 x1 a n 2 x n

x n son las incógnitas, siendo el s

{(x1 ; x 2 ; x 3 ; ..... x n )}

Observación : Para resolver un sistema de ecuaciones lineales, existen diversos métodos como por ejemplo : * * * * *

Método Método Método Método Método

de Sustitución. de Reducción. de Igualación. Matricial. de Cramer (Determinantes).

Sistema Lineal Homogéneo : Es aquel donde los términos independientes son nulos (ceros). Ejemplo :

x 2y z 0 ....... (1) 2x y z 0 ....... (2) x 3y 2z 0 ..... (3) Un sistema lineal homogéneo siempre es compatible donde una de sus soluciones es la solución trivial (cada incógnita es igual a cero). Para el ejemplo : Solución trivial = (0; 0; 0). Asimismo, el sistema lineal homogéneo puede tener otras soluciones, las llamadas no triviales.

b1

a 2nx n

b2

... a nn x n

bn

Determinante del Sistema (

conjunto solución de la forma : CS

a n 3 x3

a1n x n

....

Consideremos : 1.

x1 , x 2 , x 3 , .........

a11x1

2.

a11 a12 a 21 a 22

a13 a 23

a1n a 2n

a n1

an 3

a nn

an 2

s

)

Determinante de una Incógnita (

i

)

Se obtiene a par tir del determinante anterior, reemplazando los elementos de la columna de coeficientes de la incógnita en referencia por los términos independientes.

i

a11 a 21

a12 a 22

b1 b2

a1n a 2n

a n1 a n 2

bn

a nn

cada incógnita del sistema se obtendrá, según la relación.

xi

i s

;

i 1; n

Ejemplo :

Análisis de las Soluciones de un Sistema Lineal Dado el sistema :

Resolver :

a11x1

a12x 2

a13 x3

3 ...... (2)

a 21x1

a 22x 2

a 23x3 ....

2

5

a n 1 x1 a n 2 x 2

3

2

2x 5 y

7 ...... (1)

3x

2y

observar que : s

(2)( 2) (3)(5)

7

5

3

2

xi

(7)( 2) (3)(5) = -14 - 15 = -29

2 7 y

x

x

s y

y

y

bn

2.

El sistema tiene infinitas soluciones, si y sólo si:

3.

15 19

0. 0

s

0.

El sistema no tiene solución si siendo i

s

0 , existe

0.

Propiedad Un caso particular de lo visto anteriormente se presenta en el sistema lineal de dos ecuaciones con dos incógnitas :

Teorema : Dado el sistema lineal homogéneo. a11x 1

a12 x 2

a13 x 3

....

a1nx n

0

a 21x1

a 22x 2

a 23 x 3 ....

a 2n x n

0

a n 1 x1

a n2 x 2

... a nn x n

0

an3 x3

si este admite soluciones aparte de la trivial, el determinante del sistema deberá ser nulo, es decir:

a12 a 22

... a nn x n

El sistema tiene solución única, si y sólo si:

i

29 19

( 29 ; 15 ) 19 19

a11 a 21

b2

, luego :

algún

s

CS

i s

s

= 6 - 21 = -15 x

a 2nx n

1.

(2)(3) (3)(7)

3 3

b1

donde la solución se obtiene a partir de :

= -4 - 15 = -19

x

a n 3 x3

a1n x n

....

a13 a 23

a1n a 2n

a n1 a n 2 a n3

a nn

a ax by a1x b1y 1.

c .... (1) c1.... (2)

El sistema será compatible determinado, es decir, tendrá solución única, si se verifica:

a a1 2.

El sistema será compatible indeterminado, es decir, tendrá infinitas soluciones, si se verifica : a a1

0 3.

b b1

b b1

c c1

El sistema será incompatible, es decir no tendrá solución si se verifica : a a1

b b1

c c1

SISTEMAS NO LINEALES

Como : x = Ky

en (1) con x = 3y : 9 y

Criterios de Resolución : 1.

2

21y

Si el sistema está conformado por ecuaciones de diferentes grados se deberá encontrar una nueva ecuación en función de una sola incógnita, para a partir de ésta determinar las soluciones del sistema. Ejemplo : Resolver : x

y

7 ...... (1)

xy

10 ..... (2)

x=3

y 2 7 y 10 0 (y-5)(y-2) = 0

21

x = -3

y=2

Sol : (2; 5)

CS = {(2; 5), (5; 2)} Si el sistema está formado por ecuaciones, cuya parte literal es homogéneo y de igual grado se recomienda realizar la siguiente sustitución : y = Kx, donde el parámetro "K" se determinará por eliminación de las incógnitas x y.. Una vez encontrado el valor de "K", fácilmente se obtendrá el valor de cada incógnita del sistema. Ejemplo : Resolver :

21 ........ (1)

2

15 ........ (2)

Hagamos : x = Ky Reemplazando en (1) : y 2 (K 2 3K Reemplazando en (2) : y 2 (K 2 Dividiendo m.a-m :

y2

3

3

y=- 3

x=

3

x=- 3

CS

Sol : (5; 2)

3xy 3y 2

21

Soluciones : ( 3 ;

Si : y = 2 en (2) : x = 5

k

3)

21

3) 15

K 2 3K 3 K2 K 3

De donde, obtenemos : K 2

7 5

4K 3

K=3

0

K=1

3y 2

7y 2

y=

Si : y = 5 en (2) : x = 2

xy 3y

3y 2

y 1 y=1 y = -1

en (1) con x = y : y 2

De donde, obtenemos : y = 5

x

9y

Soluciones (3; 1) y (-3; -1)

Efectuando, tenemos :

2

x=y 2

21

Reemplazando en (2) : (7-y)y = 10

x2

2

2

De la ecuación (1) : x = 7 - y

2.

x = 3y

3) y (

3y 2

3;

{(3;1), ( 3 ; 1), ( 3 ; 3 ), (

21

3) 3;

3 )}

EJERCICIOS PROPUESTOS 01.

Dar el valor de "a", si para : (x; y) = (5; y0) el sistema verifica :

(2a 1)x (a 3)y (2a 1)x (a 2)y a) 8 d) 7 02.

x

2a 2b

b) R 3 2

d) R

x Hallar : x

08. a . b

2 3

c) R

x

y

14 ; x

05.

c) 3

x , es : y b) -1 e) 4

Calcular : x 3

y 3 , si :

a) 63 d) 65

y2

c) 0

3xy x y

b) 28 e) 0

y

x y

2

4

9 b) -18 e) 24

c) 18

Respecto al conjunto :

Tiene 6 elementos. Tiene 4 elementos. Tiene 1 elemento. Es el conjunto vacío. Tiene un número ilimitado de elementos.

Hallar el producto de los valores de "x+y", que resuelve el sistema :

b) -156 e) -171

Al resolver el sistema :

4

1 x

3 y 1

5 4

c) 26

4 x

7 y 1

15 4

se obtiene :

13

b) 1 e) 4

y

a) 112 d) 171 12.

x 2 | y | 11 a) 0 d) 3

c) 34

x 2 y 2 113 xy x + y = 43 - xy

¿Cuántas soluciones tiene? x2

2

a) b) c) d) e) 11.

a) 1 d) 8

c) 335

A={(x, y)/2x+3y - 6=0; 4x - 3y - 6 = 0; x - 1 = 1; 3y = 2}

10

5xy x y

06.

10.

20

Entonces :

b) -12 e) 16

a) 12 d) 20

Si : y

b) 334 e) 925

Indicar un valor de "xy", al resolver :

x

y , del sistema : y

b) 2 e) 5

x

a 1

Si el sistema : 3x + 5y = 1 2ax - by = 8 tiene infinitas soluciones. Hallar el valor de "a-b".

x

3x 2y 9 ... (1) x y 15 11(x y) 135 x ... (2)

04.

y 3z

a) 52 d) -28

2 3 09.

e) R {0}

a) 1 d) 4

35

a) 333 d) 331

tiene solución única, hallar : 3 2

2

Además : x, y, z; son proporcionales a los números 4, 2, 5; respectivamente. Hallar el valor de "a".

c) 10

Si el sistema :

a) R

El sistema :

x y z

1... (1) 1... (2)

b) 9 e) 6

(a 3)x (a 3)y (b 2)x (b 2)y

03.

07.

c) 2

a) b) c) d) e)

x= x= x= x= x=

1, y = 2 2, y = 1 1, y = 3 3, y = 3 2, y = 3

c) 121

13.

¿Para qué valores de "m" el sistema de ecuaciones : 2x + 7y = m 3x + 5y = 13 tiene soluciones positivas ?

a)

26 3

m

91 5

26 91 m 3 5 e) 9 < m < 11 c)

14.

m

91 5

d)

26 3

m

91 5

b) -112 e) 96

y2

a) 4 d) 10 20.

nx

z3

a) b) c) d) e) 21.

a) 3 d) 7 c) 7/12

22.

2y

25

x si : 2y > x, entonces el valor de es : y b) 3/2 e) 3

c) 2

23.

x a) b) c) d) e)

xy 2y 7 (x = 1, y = 8) y (x = 3, y = 9/2) (x = 2, y = 3) y (x = 8, y = 9/2) (x = 2, y = 9/2) y (x = 3, y = 1) (x = 3, y = 5) y (x = 2, y = 8/3) (x = 3, y = 2) y (x = 8, y = 19/2)

Hallar "n", para que el sistema sea incompatible :

y

12 y2

z

a) 9 d) 72 24.

b) 18 e 144

c) 0

c) 36

Si : a

b c a

b c

2 0

3a 5 b c

(n + 3)x + 2ny = 5n - 9 (n + 4)x + (3n - 2)y = 2n + 1 b) -2 e) 2

c) 5

El mínimo valor de "z" que satisface el sistema de ecuaciones : x2 es :

5

2

a) -1 d) 1

b) 2 e) 4

Conjunto unitario. Un conjunto de dos elementos. Un conjunto de tres elementos. Un conjunto de cuatro elementos.

x

Resolver :

3x 2 y

2x y

El conjunto de soluciones del siguiente sistema :

a) b) c) d) e)

7

a) 1 d) 8/3

33

x2 y2 r2 y = r ; para : r > 0 es :

Dado el sistema :

x

y 2 12x

x + y = 23

y ( ). x

4y2

z)

Resolver el sistema : x 2 12 y

.... (2)

b) -12/5 e) 3/5

3xyz

Existen cuatro soluciones. Existen tres soluciones. Existen sólo dos soluciones. No existen soluciones enteras. Existe más de cuatro soluciones.

Calcular :

x2

18.

y3

x 2 (y se concluye que :

c) 1

c) 8

Si : x, y, z son enteros y no negativos, entonces con respecto a las soluciones del sistema :

Si : n > 0; proporcionando el valor de :

a) -7/6 d) 5/7

17.

b) 6 e) 12

2

144 .... (1)

y 13

10

2x (b 1)y 5 posea infinitas soluciones.

x3

Determinar la única solución del sistema: x2

16.

26 3

Hallar "a+b", de modo que el sistema :

(a 1)x 4 y

Sea la terna (a; b; c) solución del sistema de ecuaciones: 7x + 4y - 4z = 7 7y + 5z = 12 11y + 8z = 10 Entonces, la suma (b + c), es igual a : a) -100 d) 80

15.

b)

19.

Entonces : 2a a) 13 d) 10

0 5 2c es igual a : b b) 12 c) 11 e) 9

25.

Sea "m" un entero, tal que el sistema de ecuaciones : 2x + 3y = 8 mx - y = 37 3x + 8y = m

31.

(x + y) (x - y) = 11 ......... ( ) (y + 3) (y - 3) = x

sea compatible. Si : ( x0 , y0 ) es la solución de dicho sistema. Hallar el valor de :

E a) 0 d) 3 26.

m (x 0

c) 2

Hallar el valor de "a" para que el sistema tenga solución única :

x 2 y2 x y z

32.

y x

b) - 2

d) - 5

e) - 10

b) 2 e) 6

b) 3 e) -3

x x

x

y

1

a) Únicamente b) Sólo c)

29.

2

34.

= -1

= -1;

= 1,

e) Sólo cuando

=1

x

y

30.

c) 2 (a b)2 e) a - b

d) 2 (a

b)2

Si las ecuaciones :

1 ; cx 2 dy 2 a2 c

a) 1 d) 5/4

16 ......... (1)

y 5 mx ......... (2) para un cierto valor de "m" admite solución única. Obtener dicho valor de "m". a) 3/4 d) 1/2

2 (x-y) es igual a : b) (a b)2

= -1

El sistema de segundo grado : 2

b 2)

1

tienen solamente una solución, calcular :

=0

d) Únicamente

2

b b

a) (a b)2

ax by

=0

a a

ab (a 2

Entonces, el valor de

, el sistema en x é y :

x y es compatible indeterminado?

y y

xy

c) 4

¿Para qué valor del parámetro

c) 3

Se tiene el siguiente sistema de ecuaciones:

x xy y 9 .... (2) Indicando el menor valor que toma "x".

28.

3

Resolver en R el sistema : x+y-z=1

a) 1 d) 4 33.

3 ..... (1) 2

a) 2 d) -2

c)

Indicando el número de elementos del conjunto solución real del sistema.

b) a = 2/3 d) a = -2/3

Resolver en R 2 el sistema de ecuaciones:

x y

a) - 6

x 2 y2 z2 1 x 3 y 3 z3 1

z a

a) a = 1 c) a = 4/3 e) a = -1/2 27.

......... ( )

Indicando uno de los valores obtenidos para "x" ó "y".

y0)

b) 1 e) 4

Resolver :

b) 1/4 e) 1/5

35.

b) 3/2 e) 4/5

b2 d c) 2/3

Indicar "z" al resolver :

x

2y w

5

2x 3z 7 x y z 2

c) 7/4

2x

y w

2

¿Cuántas soluciones no nulas tiene el sistema : 3xy + 2z = xz + 6y = 2yz + 3x = 0 ? a) 2 d) 5

b) 3 e) 6

c) 4

a) -1 d) 0

b) 2 e) 8

c) -3

36.

El valor positivo de "x+y+z", del sistema : 2x + y + z = xy + yz 2y + x + z = xz + xy 2z + x + y = xz + yz x2

37.

y2

z2

42.

y2

b) 2 + 5

d) 2 + 3

e) 2 + 2

c) 2 + 7

43.

manera que el sistema lineal homogéneo : (1 - k) x + y - z = 0 2x - ky - 2z = 0 44.

admita también soluciones no triviales. a) 12 d) -9 38.

b) -2 e) 0

2

x

ax

x

3

bx 12 0

18

c) 4

b) 6 e) 16

z

a)

a

b)

b

d)

a b

e)

a b

c)

(10 x y)2

b) 6 e) 10

c) 16

Resolver :

4y2 1

2x 3x 2y

2 ..... ( )

3y (x 1) .......... ...... ( )

1 1 1 1 ...... (2) x y z 12 xy+yz+xz = -2 ..... (3)

a) 1 d) 1/8 46.

b) 3 e) -3

c) 4

x

a) 2 d) 5

y 3

x

2

3

b) 3 e) 6

x

x3

y3

b (x

47.

c) 4

0, para que :

105 b) 4 e) 7

c) 5

2xy xz

xy 2xy 2 zx

, donde : x R .

2

yz

4

2

6

2yz z

z

y) ....... (2)

0

b) 4a + b < 0

2

0

d) b 4 a 2

0

70

En el sistema, hallar : x y

x2

a) 4 b a 2 e) a 4 b

c) 1/4

?

admita soluciones de componentes reales.

c) a 4 b

y

2

a) 3 d) 6

y3

Establecer la condición para : a, b, x + y = a > 0 ........ (1)

y xy

x xy

7 3

b) 1/2 e) 1/16

Resolver el sistema, y hallar : y - x. x2

¿Cuántas soluciones tiene el sistema : y

ab

Dado el sistema :

Indicando el menor valor para "y".

2

1 ?

c) 4

Luego de resolver el sistema : x + y + z = 5 ........ (1)

a) 2 d) -2

41.

y2

Del sistema :

3 x 2y 2x

c) 3

Señale el menor valor que toma "x".

40.

x2

y

b) 3 e) 6

a) 5 d) 14 45.

tienen 2 raíces comunes.

39.

x2

Entonces, el valor de : x + y, es :

0

a) 4 d) 5

z2

(x 1)2 (y 1)2 xy x y 11

Hallar : (a+b), para los cuales las ecuaciones : 3

x

x 2 y 2 xy ab (x+y)(ax+by)= 2ab (a + b) un valor que toma "x" es :

Determinar la suma de valores que adopta "k", de tal

x - y - (1 + k) z = 0

z2

a) 2 d) 5

2

a) 2 + 6

¿Cuántas soluciones tiene el sistema :

0

a) 1 d) 4

b) 2 e) 5

c) 3

48.

Resolver :

2 x y 8

54. 4

y)4 (x

(x

x

2y

4 ...... ( )

2y)2

a) 6 d) 40 49.

x

2 ...... ( )

Indicando (xy), si : x, y

c) 30

x 2y 6

85 ..... ( )

55.

Indicando la suma del mayor valor de "x" con el menor valor de "y". a) 8 d) 3/2 50.

b) 16/3 e) 7/2

a) 1 d) 4

y2

y x2

¿Para qué valor real de "K", el sistema :

x2

y2

z2

2

x 3 y 3 z3 K tiene solución real?

a a2

b) 2 e) 5

a) 2 d) 56.

c) 3

(x 1) (y 1) (10 x xy - x - y = 11

2x (2a

b)y

b

6a 4

(xo, yo) es solución del sistema. c) 0 e) 4

z2

58.

(i =

1)

yz

zx . c) 30

b) 3 e) 5

c) 2

Resolver el sistema y dar "xy".

5x 8 y 44

5 ... (1)

1 10x 88 ... (2) 2

b) -3 e) 99

c) -90

Encontrar el intervalo de "m" para que el sistema : 2x - 5y = 1 ; mx + 10y = 4

a) m > -4 d) m > 8

41

b) 39 e) 40

y = bi

se satisfaga

2(1 xy)2 x y

Indicar como respuesta : x y a) 29 d) 49

1 ... (2)

3xy y

a) 11 d) 86

c) 19

Resolver el sistema :

y2

2

5 16y 2x 3y 17

2a + b = 1.

Determinar : a+ b + 2xo + yo, donde :

x2

0 ... (1)

2

8 2x 3y 17

3.b a b

x+y+z=9; z

xy 2y 2

a) 4 d) 1

c) 7 57.

Donde : a - 2b = 3

a) 11 d) 3

3x 2

Hallar : (a b)a , si : a , b , Z .

Dado el sistema : 4a

7 4

e) Más de una es correcta

5

x = ai

y)

b) 5 e) 11

(a 2b)x y

c)

3

se obtiene una solución de la forma :

2

a) 3 d) 9

b)

Al resolver el sistema :

2x

Resolver y dar el valor de : "x+y". 2

53.

c) 6

x+y+z=2

c) 17/2

Se obtienen para "x" é "y", 2 valores de la forma : 1 [1 (na 1)(na 3)] 2 Hallar "n".

52.

b) 5 e) 10

Resolviendo el sistema : x

51.

9 ....... ( )

a) 1 d) 8

15 .......... . ( )

x 2 x 2y 2 x 2 y 4

y3

indicando la suma de todos los valores de "x" con todos los valores de "y" obtenidos.

Resolver el sistema :

xy 3

3 xy ....... ( ) 2

y

x3

R.

b) 9 e) 16

x xy xy 2

Resolver en los reales :

x R ;

y R .

b) m < -4 e) m < 8

c) -4 < m < 8

59.

Dado el siguiente sistema de ecuaciones: x+y+z=0 (b+c)x + (a+c)y + (a+b)z = 0 bcx+ acy + abz = 1 Entonces, la solución del sistema para : x, y, z, en ese orden con a b, b c, a c, es : a) b)

(a

1 ; b)(a c) (a

1 b)(c

b)

;

(a

1 c)( b c)

1 1 1 ; ; (a b)(a c) (a b)(c b) (a c)(b c)

60.

Resolver el sistema adjunto y proporcionar el valor real de "x"; a, b, c R . x3

a

y3

b

z3

c

xyz

siendo : a 2 b2

a)

3

b)

3

2a 2 a

b 2c 2

a 2c 2

ab ac b c

bc

2a 2 ab ac bc 2 (a b c)

c)

a b c ; ; (a b)(a c) (a b)(c b) (a c)(b c)

d)

a b c ; ; (a b)(a c) (a b)(c b) (a c)(b c)

c)

3

e)

a ; b ; c (b c) (c a) (a b)

d)

3

(a b c)2 ab bc ad

e)

3

a 2 b2 ab bc

2a 2 ab ac bc 2 (a b c)

c2 ac

2abc(a b c)

Claves 01.

c

31.

d

02.

d

32.

b

03.

a

33.

a

04.

e

34.

a

05.

d

35.

c

06.

e

36.

a

07.

b

37.

e

08.

a

38.

c

09.

d

39.

d

10.

c

40.

e

11.

b

41.

a

12.

e

42.

d

13.

d

43.

c

14.

b

44.

a

15.

b

45.

b

16.

b

46.

b

17.

e

47.

a

18.

b

48.

c

19.

b

49.

c

20.

a

50.

b

21.

e

51.

b

22.

b

52.

a

23.

d

53.

b

24.

c

54.

c

25.

a

55.

e

26.

e

56.

d

27.

c

57.

e

28.

e

58.

c

29.

a

59.

c

30.

c

60.

c

Capítulo

14

DESIGUALDADES EINECUACIONES VALOR ABSOLUTO

DESIGUALDADES

Teoremas de la Desigualdad

De fi ni ci ón

1.

Se denomina desigualdad a la comparación que se establece entre dos expresiones reales, mediante los signos de relación >, b ab c>d a.c > b.d

5.

Axiomas de la desigualdad

a b R :a

0

a>b c>d a+c > b+d

mayor que b menor que b mayor o igual que b menor o igual que b

Observación : A los signos de relación > o < se les da el nombre de signos simples mientras que a o se les denomina signos dobles.

1.

a R : a2

a

0,c

a

b

2.

a

0:a

1 a

3.

a

0:a

1 a

c2

0 c

0

b2 2 2

a2 c2

c 2n

1

Propiedad adicional :

1.3.Intervalo mixto (semi abierto o semi cerrado) :

Considera sólo a uno de sus extremos para : Para números reales positivos, tenemos : MP MA MG MH

= = = =

MP

k

ak

bk

a

2

bk 3

ck

a

MG

b

x

b

a ; b] x

MH

a

a

b

ab

2

1 a

2

2. 1 b

b c 3

x

b

b

x [a ; b

Intervalos no acotados : Son todos aquellos donde al menos uno de los extremos no es un número real. 2.1. Intervalo acotado inferiormente :

c; k Z 3 abc

3 1 b

1 a

x a

1 c

Donde : a x a;

INTERVALOS De fi ni ci ón Se denomina intervalo al conjunto cuyos elementos son números reales, dichos elementos se encuentran contenidos entre dos números fijos denominados extremos, a veces los extremos forman parte del intervalo. 1.

x

para :

b; k Z

para tres números : a, b ak

a

a

MA

Para dos números : a

k

x

Media potencial Media aritmética Media geométrica Media Armónica

x

x

a

x a

Donde : a

x

x

a

x [a ;

Intervalos acotados : Son todos aquellos intervalos cuyos extremos son reales, estos pueden ser :

2.2. Intervalo acotado superiormente : 2 x

1.1.Intervalo abierto :

a

No considera a los extremos, se presenta por existencia de algún signo de relación simple. En la recta, se tendrá :

Donde : x ;a

x

x a

Donde : a

x

b

x

a

Donde :

a;b

x

x

b

También : x (a ; b)

x

a

x

a

; a]

Observaciones :

Se considera a los extremos, se presenta por existencia de algún signo de relación doble. En la recta real, se tendrá :

x

a

x

1.2.Intervalo cerrado :

Donde : a

x

b

También : x ] a ; b [

a

a

1.

Un conjunto se dice que es acotado si y solo si es acotado superiormente e inferiormente a la vez.

2.

Para el conjunto de los números reales R, se tiene :

R ]

b

; [

Es evidente que

x [a ; b] 3.

; y

no son números reales.

Como los intervalos son conjuntos, con ellos se podrán efectuar todas las operaciones existentes para conjuntos, tales como la unión, intersección, diferencia simétrica, etc.

Clases de desigualdad 1.

Desigualdad absoluta : Es aquella que mantiene el sentido de su signo de relación para todo valor de su variable. Vemos un ejemplo : *

2.

x2

2x 10

0;

Resolución de la inecuación : Se recomienda utilizar el método de los puntos de corte cuya aplicación consiste en los siguientes pasos : 1.

Se trasladan todos los términos al primer miembro, obteniendo siempre una expresión de coeficiente principal positivo.

2.

Se factoriza totalmente a la expresión obtenida.

3.

Se calculan los puntos de corte. Son los valores reales de "x" obtenidos al igualar cada factor primo a cero.

4.

Se ubican, ordenadamente, todos los puntos en la recta real, dichos puntos originan en la recta dos o más zonas.

5.

Se marcan las zonas obtenidas a partir de la derecha alternando los signos "+" y "-".

6.

Si el signo de relación es > o , el conjunto solución estará formado por todas las zonas positivas, pero si el signo de relación es < o el conjunto solución lo formarán todas las zonas negativas.

x R

Desigualdad relativa : Es aquella que tiene el sentido de su signo de relación para determinados valores de su variable. Veamos un ejemplo : * 2x 1 x 3 x 2

INECUACIONES De fi ni ci ón Se denomina inecuación a cualquier desigualdad relativa. Los valores de la variable que verifican la inecuación forman el conjunto solución, el cual se presenta en función de intervalos. 1.

Inecuaciones racionales : 1.1. Inecuaciones de primer grado do o (lineal))

ax b 0

a b R/a

x2 x

0

1.2. Inecuaciones de segundo grado (cuadrática) i a))

ax 2

a,b c R / a

bx

x2 x 6

0

Hallando puntos : x = -3; x = 2

bx c

En la recta :

0; x R ,

0 b2

entonces : a

4ac

0

bx c

0;

4ac

0

1

a o , a1 , a 2 , ....

a2x n

2

... a n

an R / aº

0

2

0 ; [H]º 1

+ -3

x

1.4. Inecuaciones fraccionarias :

+

+

0

3

F(x) H(x)

-3

como el signo de relación es > la solución viene dada por todas las zonas positivas.

1.3.Inecuaciones de grado superior :

a o x n a1x n

2

+

x R,

0 b2

entonces : a

-3

marcando zonas :

II . Trinomio siempre negativo

n N/n

0

Factorizando : (x+3)(x-2) > 0

I . Trinomio siempre positivo

2 Si : ax

6

Resolución : De acuerdo con el método de los puntos de corte, procedemos así :

c 0

Propiedades

Si : ax 2

Ejemplo : Resolver la inecuación :

; 3

2;

2

Ejemplo :

2.2. Forma : 2n A

Resolver : 9 x 10 x 2

CS

Resolución : Procedemos de un modo similar que en el ejemplo anterior : 9x 10 2 0 x 2 7x 6 0 x 2 Puntos : 6 7 x = -2 x

7x + 6 = 0 x+2=0 +

0 B

0

+

< 2n B ; m n Z 2.3. Forma : 2 m A >

CS

A

0 B

A 2n

0

x 1

x 1

0

x 1

0

x 1

x 1

0

x 1

0

x2

3x

0

2

3x

0

0

1

0

x

1

0

x

x 1

0

x 1

0

x(x

3)

+

+

+

-1

x

5 x 12

(x 1)2

S1 : x 1

Ejemplo : x2

B 2m

Ejemplo :

x

Observación : En una inecuación fraccionaria, si el signo de relación es doble, sólo cerraremos los extremos que provienen del numerador.

2

B2n

A

Resolución : De acuerdo con la forma (2.1), se plantea :

2; 6 7

Resolver :

A

Resolver :

- 6 7

-2

x

B ;n Z

2

1

+

0

3

Intersectando :

1

Resolución : x2

x

2

x2

5 1 x 12

x 7 x 12

-1

0

Observar que : S1 S2 : x 1

0

Observar que: x 2 x 12 x 7 0 (x 4 )(x 3) Puntos : { 7 , 4

(x

4)(x

0

1

3

[1 ; 3

x 1

0

+

3)

+

-1

1

Intersectando :

3} +

+

-7

-3

x [ 7; 3 2.

0

4

4;

-1

Observar que : S 2 Finalmente : CS

[ 1;1 S1

Ejemplo :

B;n Z

Resolver :

se resuelve : S1

(A

0

B

0

S2

(A

0

B

0)

CS

S1

A

S2

CS [ 1; 3

Inecuaciones Irr acionales 2.1. Forma : 2n A

1

B 2n )

x 2

5 x

Resolución : De acuerdo con la forma (2.3) se plantea:

x 2 x 2

S2

0 5 x 0 x 5

0 0

x 2 5 x 2x 7 0

+ 2

+

+ 5

7

Intersectando :

Ejemplo : Resolver : |5x - 1| = 2 - x 7 2

2

Resolución : Se plantea lo siguiente :

5

[2 ; 7 2

CS

2 x

0 (5 x 1

2 5x 1

x

0 (6x

4x

2

1 x 2

x 2 (x

VALOR ABSOLUTO (V.A.) De fi ni ci ón Dado el número real "x", la relación funcional denotada por |x| es el valor absoluto de "x", definido de la manera siguiente : x;x 0

|x|

0;x 0 x;x 0 Según la definición : * |5|= 5 5>0 * |-7| = -(-7) -7 < 0 |-7| = 7

|x| 0;

1) 4 1 verifica x < 2. 2 1 x verifica x < 2. 4 CS { 1 ; 1 } 2 4

Observar que :

x

1.

|x| b

x

b

x

b

2.

|x| b

b

0

( b

x

3.

|x | |y|

(x

y)(x y)

b) 0

Ejemplo :

x R

2.

|x| | x|;

3.

| x . y | | x |. | y | ; |x | ;x |y|

1)

Resolver : |3x + 4| < 5

1.

x y

2)

Inecuaciones con Valor Absoluto

Teoremas :

4.

3

x

Resolución : De acuerdo con la forma (2), se plantea :

x R

5

x y R

y R/y

5.

|x 2 | | x | 2 x 2 ;

6.

| x | x | x |;

7.

| x y | | x | | y |;

-5 - 4 < 3x < 5 - 4 -9 < 3x < 1

x y R

-3 < x <

Propiedades :

x

2.

entonces : xy 0 Si : |x - y| = |x|+|y|, entonces : xy

1 3 3; 1 3

Ejemplo : Resolver : x 2

0

3| x | 4

Resolución : Se sabe que x 2 | x |2 . Luego, se tendrá :

Ecuaciones con valor absoluto : |x | b; b

0

x

| x |2 b x

b

Ejemplo :

3| x | 4

| x |2 3 | x | 4

0

(| x | 4) (| x | 1) 0 Observa que : | x | 1

Resolver : |2x-1| = 7 Resolución : Observar que : b = 7 > 0. Luego, tenemos :

2x 1 7 2x x

8 4

CS {4 ;

2x 1 2x x

3}

5)

-5 < 3x + 4 < 5

x R

Si : |x+y| = |x|+|y|,

3x 4

¿ ? porque es una verdad

Luego, sólo se resuelve :

0

x R

1.

0 ( 5 R

x R

|x| 4

En consecuencia :

0

|x| 4

7 6

3

0;

Según la forma (1) : x

x

4

x ;

4 4] [4 ;

EJERCICIOS PROPUESTOS 01.

Resolver las siguientes inecuaciones : x

I.

3

x

2

2

II. (x

2)

III. 3(x 5) IV.

2 4

V.

3

Rpta. ...........

5(x

Rpta. ...........

5

x 1

3x

VI. 0,3 4 x 02.

x(x 1)

x 1

1

2

x 3

Rpta. ........... 2

Rpta. ...........

x 3

2

x 4

2

a) x < 0

b) x > 0

d) x > 4

e) x > 2 5

5

x 1

08.

5

x 2

c) x

0

09. c) 0

................. x2 ..............

II. -9 < x < -4

................. x2 ..............

III. -4 < x < 7

................. x2 ..............

IV. -8 < x < 3

................. x2 ..............

V. 3 < x < 11

................. x-1 .............

VI. -9 < x < -5

................. x-1 .............

6

b) K

1 3

d) K

4 3

e) K

3

10.

c) 1

-1; 2

c)

;

2a b

d)

b) -10 y 1 e) 1 y 10

12

1;

2a b

Un vehículo, marchando a 25 km/h recorre un camino que mide un número entero de km. Cuando llevaba recorrida la mitad del camino, le faltaba menos de 3h 31min, y cuando llevaba recorridos 60 km le faltaban más de 4h 35min de marcha. ¿Cuál es la longitud del camino?

c

b) 225 km e) F.D.

Si : x [ 4 ; 2 de : f(x )

II. Si : x

11.

, indicar el intervalo de variación

6(x 8)

1

3 ; 5] , indicar el intervalo de variación de: 2x 6 x 1 5 ; 4] , indicar el intervalo de variación de : x2

6 x 15

Resolver el sistema :

ab 5, entonces, c

c) 2 y 10

c) 175 km

Resolver :

está comprendido entre : a) -10 y -1 d) 2 y 20

; 1

b)

f(x )

b

1

2a b

III. x

-5; -2

a

bx a a

1;

f(x )

b) 7 e) 0 a

b

a)

I.

5x 3y 2 2x y 11 y 3

Si : -10

c) K

Resolver :

a) 130 km d) 170 km

Hallar el valor de : P = |x - y|. Donde : x, y son números enteros positivos que satisfacen las siguientes desigualdades :

a) -1 d) 8

m 2 p2 mp

2 ;1]

b) 2 e) 6

3 0 con relación a :

c

1

a b , b x

a b

b) b < c < a

a c) -4 x - 2y < -7 2x + 3y < 6

a) -2 d) 6 22.

b)

Sean p, q, r, tres números positivos diferentes, que cumplen : pqr = 1. Entonces, la suma : s = p+q+r satisface.

{x; y}

I.

b) VVV e) FFF

c)

a) 2 d) 8

De las siguientes proposiciones :

a) VFV d) FVF

ab

E

b) 18 c) 16 e) No es posible

Un closet tiene capacidad para 60 trajes, pero, sólo hay cierto número de trajes guardados en él. Si el número de trajes se redujera a la sexta parte se ocuparía menos de la décima parte de su capacidad; pero si se duplicara el número de trajes; más de ocho trajes no podrán ser guardados por falta de espacio. ¿Cuántos trajes hay en dicho closet? a) 20 d) 35

19.

d) a< c 1 < 1

Se sabe que el cuádruplo del número de objetos que hay dentro de un depósito, es tal, que disminuido en 5, no puede exceder de 35 y que el quíntuplo del mismo número de objetos, aumentado en 2 no es menor que 50. Hallar este número. a) 20 d) 10

a)

a) s > 3 c) 0 < s < 3 e) 1 < s < 2

podemos afirmar que : a) 1 < c <

Para : a > 0 y b > 0. ¿Cuál de las siguientes expresiones es verdadera?

Z. Indicar "xy". b) -6 e) 10

c) 3

La suma de los dos números enteros positivos es mayor que 76; su diferencia menor que 10, y si al mayor se le suma el duplo del menor, el resultado no llega a 112. ¿Cuál es el mayor? a) 34 d) 43

b) 38 e) 83

c) 42

23.

Si : x, y, z

R , hallar el máximo valor de "a" en : x4

y 4 z4 xyzw

w4

29.

Si : 0 < b < a, Además :

a2 b(a b)

K

a

b2 ; a(a b)

luego, podemos afirmar que : a) 1 d) 24.

b) 2 e) 8

2

Cuando nací, papá tenía más de 20 años; hace 10 años el doble de mi edad era mayor que la de él; si tengo menos de 33 años, ¿qué edad tiene él? a) 32 d) 54

25.

c) 4

b) 53 e) 45

30.

S S a

2

b) K

1

d) K

8

e) K

8

Si : a > 0 y P a) P > 1

S S b

S S c

...

1 2

d) P> 31.

b) P >

c) K

0

1

1 4

(1 a)

c) 52

Si : "S" es la suma de "n" cantidades positivas a, b, c, ......, entonces :

E

a) K

a , luego :

1 2

c) P > 0

e) P > 20

Resolver cada ecuación cuadrática :

resulta : a) E c) E e) E

2

b) E

n

d) E

n

n 1

n

2

2

n n 1 n2 n 1

Si : b2 b 1

a)

1 2

b)

1 d) 6 27.

N,

c)

1 3

33.

a) 14 669 d) 14 999

1)

x 2 6x 5

0 2)2

(x

29

Resolver cada inecuación de segundo grado : I.

x 2 3x 1 0

II.

2x 2 9x 3

0

III. x

2

5x 8

0

IV. x

2

2x 5

0

Determinar "m+n", si la inecuación :

0

presenta como conjunto solución :

x

A qué número entero se aproxima :

1

5x

2 (x

x 2 mx n

1 e) 4

S

28.

2 3

II.

IV. (x 1)

Sean : a, b R , tal que : a + b = 1. a2 a 1 entonces, MN resulta :

35

2

2

1

M

x 2 12x

III.

32. 26.

I.

1 2

3

1 3

3

...

b) 14 999 e) 14 899

a) -13 d) -2 1

3

10 6

34.

Determinar el menor valor de "E", si se cumple :

a) 1 d) 4 35.

8

c) -15

b) 2 e) 5

c) 3

Resolver cada desigualdad :

Indicar la suma de las cifras de 10 P .

I.

(x + 1)(x - 3)(x + 4) > 0

a) 12 d) 18

II.

(x 1)2 (x

b) 13 e) 20

c) 14

E

se verifica para todo x R .

c) 14 866

a5 b3c 2

b) -17 e) 2

x 2 2x 5

Sean : a, b, c; números no negativos, tales que : a+b+c = 1, hallar el máximo del producto :

P

5;3

III (x 6)(x

2)3 (x 5)5

0

4)2 (x 1)(x

3)2

IV. (x 1)(2 x)(x

3)

0

0

36.

Resolver :

x4

a)

x

;2

3;

b) x

;1

2;

c)

3x 3

5x 2

9x 6

43.

0

x 1 x 2 x x 3 se obtuvo como solución :

;a

x R

37.

x

a) -1 d) -7

1; 2

Después de resolver : x 3 4 x 2 2x 8 0 Señalar el mayor entero que verifica la desigualdad. a) 0 d) -1

38.

b) 2 e) 1

II.

x2

4x 3

40.

45.

1 x

I.

x 3

2

II.

x

3

III.

x 8

IV.

x

3

2

; 2

b)

; 2] [3 ; 4

c)

; 2

d)

; 2

1; 0 [ 1 ; 0]

Sean las funciones : x2

5x

3 0

46.

2m

3

3

7 x

II.

9x

4

18

x

2 III. (x 5)(x

a) 4 d) 6

c) [5 ; 7]

7x 2

2 x

a)

3; 3

c)

3 ;4

e)

;0

b) d)

3

2 x

b) 2 e) 10

n

c) 3

El conjunto :

a) [ 2 ; 1

4(x 5)

2 2 Resolver la inecuación : x (x e indicar un intervalo solución.

e) Menor que 12.

se verifique para todo "x" real.

e) [5 ;

3)

b) Menor que -12.

Indicar el menor número "n" entero que permita :

A

x3

0

0 ;1]

Resolver las inecuaciones : I.

x

g(x) 2x 13 x m 4 ¿Qué raro?, se observa que al darle cualquier valor a "x" se obtiene que f(x) b > 1. Hallar el conjunto solución. a)

1 ;1] b) [ b

; b

e)

; b]

[ 1; 1 b

[1;

3 c) FFV

x2

ax

2a 2

a

2a

b) [ a ; c) d) e) 57.

2a ; a 2a ; a ; 3a

a; 2a ; a;

Determinar, por extensión, el conjunto :

{x R / x 2 4 x

2

a) { 1 ; 0 ; 1}

b)

c) [ 2 ; 3]

d) { }

e)

c) [ 1; 1] d)

3a ;

A

b ; 1]

0

Resolver :

a)

De la inecuación :

ax 1 bx 1

1

Si : a < 0.

c) 1

b) 2 e) 0

x

x

b) FVF e) VVV

x

2,

indicar la suma de valores enteros de "x". a) 1 d) 2

x

a) FVV d) FFF 56.

Luego de resolver :

x 1 x 3

0; 1

16 x x 2

II. Si : x [0 ; 4

coincide con : a)

0

27)(4 x 16)

Determinar el valor de verdad de las proposiciones : I.

Entonces, el conjunto : {x R /

[2; 4]

S

b)

55.

A

c) x

S

b) [3 ;

5;0

0

entonces, es verdad que :

Determinar en qué conjunto de números negativos debe estar contenido "x", para que :

x 4 17x 2 60 x (x 2 8x 5)

5 x)

Si : "S" es el conjunto solucion de la desigualdad :

3; 6

a)

6 x 8 (x 2 x 1

b) x R e) x

{2; 4}

15 ; 10

e) a

50.

x2

2 ?

1; 3

d) a

49.

2

0 ;1

2x 10} 1;0

58.

Al resolver :

63. 2

2

(2x 1)(x 5 x 1) se obtiene como solución :

0

x

x R [m ; n]

a) [3 ; 7]

Calcular : mn.

59.

Sea :

b) -3 e) 0

c) -4

x 7 (x 5)

64.

x 1 ? x

¿Entre qué valores está :

60.

3 7 ; ] 5 5

d)

2 1; ] 5

b)

6 0; ] 5

a)

e)

6 1; ] 5

ax 2

Dado : f(x )

bx

c)

c)

6 ; ] 5

c , tal que :

65.

S

S

[ 5;0

d) S

3; 1

e) S

2; 2

d) 61.

b)

¿Cuántos valores enteros verifican la inecuación :

7 2

c) 3

66.

Si :

x2

II. Si :

x

1 , entonces, x 2

III. Si : x < 1, entonces, x 2 V. Si : x 2

2

b) 2 e) 5

Resolver las inecuaciones : I.

x 3

2

II.

x

3

III.

x 8

IV.

x

5

3

1

67.

1

3 0

c) 5

Hallar el intervalo formado por los valores de "x" que satisfacen la siguiente desigualdad :

a)

4;

b)

d)

0;

e)

2; 4

1

c)

2;

2; 4

Resolver :

3x 2 2 x 3 e indicar el número de valores enteros que no la verifican.

1

1 , entonces, x < 1

a) 1 d) 4

3 ?

2x x 2 4 x 2 x 2 (x 4)

1 , entonces, x > 1

IV. Si : x > 1, entonces, x

x 13

b) 7 e) 3

e) 4

¿Cuántas de las proposiciones siguientes son verdaderas? I.

62.

a) 6 d) 4

5 2

0; 1 3

3 x x 3

a b c b a

3 x

0; 1 2

3; 1

Hallar el mínimo valor positivo de :

a) 2

3 x

[ 4 ;1

x R ; f(x) 0 . A

c) [5 ; 7]

Sea "S" el conjunto solución de :

b) S a)

7 x

e) [5 ;

1 x 1 3x entonces :

0

3

b) [3 ; 5]

; 5]

d)

a) 1 d) -1 6

Indicar el intervalo solución de :

c) 3

a) 12 d) 15 68.

b) 13 e) 16

c) 14

El conjunto solución de la desigualdad : x2 4x está contenido en : a) [1; 4]

b) [4 ; 8

d) [8 ;

e)

x

4

x 6

c)

; 4]

4; 6

69.

El conjunto solución obtenido al resolver :

x2 x 1 es :

a;b

b) 6 e) -5

c) 8

a) [ 1; 1] d) 71.

a) 5 d) 8

Hallar el intervalo solución de la inecuación : 3 x

1 x

1 ; 1] 4

b)

1 ;2 2

4

e)

0

c)

76.

15 ;1

2 x x 2 x 3 Indicar la suma de los extremos finitos del intervalo solución. c) 1

Resolver :

x 2 8x a)

20

13 x

3

77.

b)

2 1 ; 2

c)

2 1 1 ; 2

d)

4 2 1 1 ; 2

e)

2 ; 2

Considerar los 4 pasos para resolver la desigualdad :

1 x 9

; 10] [2 ;13]

1 81 x 2

b) [13 ; Paso 1 :

; 2] [10 ;13]

c)

d) [ 2 ; 10]

x

Indicar el intervalo solución al resolver :

a) [0 ; 1] b)

c) [0 ; 2]

a) b) c) d) e)

[4 ;

; 0] [4 ;

e)

;

3

73

]

8

78.

3

x 4 6x x 1 Indicar el conjunto no solución.

d)

b) [0 ; 64

; 64

315 0 4

Todos los pasos son correctos. El primer error se comete en el paso 1. El primer error se comete en el paso 2. El primer error se comete en el paso 3. El único error se comete en el paso 4.

Al resolver :

| x

2 |x

6| 2

x

9|

2

0 , se obtiene un

conjunto solución de la forma:

Resolver :

a) R

2

Entonces, se puede decir que :

4;

d)

9 2

Paso 4 : x R , por lo tanto, la solución es todo R.

x 2 6x 8

[8 ;

0; 2

x 9

81 x (x 9)2 Paso 2 : 8 Paso 3 : simplificando

[13 ;

3x

74.

81 x 2 2

e) 73.

x

2 1 ; 2

a)

b) 2 e) -2

c) 7

Resolver :

0 ;1]

4

72.

b) 6 e)12

x2 1 x2 x

Luego de resolver :

a) 0 d) -1

Resolver :

32 2x x x 2 Indicar cuántos valores enteros la verifican.

4 x

. Indicar : a.b

a) 4 d) -3 70.

75.

e) R

8;

[a ; b

c ; d] .

0 Dar como respuesta : b)

0 ; 64

a) 11/5 d) 13/6

(a d) . ( b c)

b) 9/7 e) 12/7

c) 10/3

79.

Al resolver la ecuación : x

2

24 x 144

x

2

12x

86. 36

x

2

6x

9

,

x2 x

se obtiene un conjunto solución de la forma : [a; b]. Hallar : a + b. a) 2 d) 5 80.

b) 3 e) 6

c) 4 87.

2 | x |. (1 x ) (| x

3 | x 1)(| x | 2)

2; 2

a)

e)

0 es:

d)

88.

a) 1 d) 4 89.

d) x [2 ; 9

d)

b) 10 e) 5

90.

4

x | | x3

b) 8 e) 1

b) -

4

6

e)

1

c) 0

64

1

Resolver :

x 2 16 x 4

Indicar el conjunto solución :

2 3

c) 4

3 2

91.

a)

b) {2}

4 d) { } 3

4 e) { } 5

20 |

0

b) 8 e) 2

c) 9

a) [-4; 4] c) [-3; 3] e) [-4; -3] 92.

[2; 4]

Resolver :

a) R d) [2 ;

c) 5

6

[3; 4]

x

Indicar la suma de estos.

8 x

b) [-2; 2] d) [-4; -2]

Hallar los valores de "x" en :

| 5 | x 3 || | 4 | x 3 ||

1 c) { } 4

Resolver : x

x | 3x 15 | | 4 x

b) 0 e) 4

x 3 | ... | x n

c) -3

Indicar la suma de soluciones obtenidas.

a) -2 d) 6

1

c) 3

x2 | | x4

Luego de resolver :

a) 7 d) 10

x 7

x2 x 1

Una solución de : |2x+3| = |x - 1| es :

5x 2

c) 35

b) -1

Resolver : |2x + 3| = 6, e indicar la suma de soluciones.

x3

5

b) 2 e) Infinitos

| x 1| | x 2

c) x [2 ; 4

e)

x 4

Hallar el único valor entero que verifica la ecuación :

a) 2

1 4

3

Luego de resolver :

b) x [2 ; 3

d)

2

Indicar el producto de soluciones de la ecuación :

a) x [2 ;1

2 3

6x

c) { 2 ; 3 }

x 8

Resolver :

a)

85.

e) { }

4

¿Para cuántos valores se verifica la ecuación mostrada?

e) x [2 ; 7

84.

b) { 2 ; 3}

d) R

2 ; 1]

| x 3| | 4 x | x 1 x 2 x 2 x 1 |x 4 | |3 x |

83.

a) {1 ; 2 }

a) 7 d) 14

2 ; 1] [1 ; 2

a) 0 d) 4

4

x 5

b) [1 ; 2

c) [1 ;1]

82.

x 2 5x

3

El conjunto solución de la inecuación : 2

81.

Hallar el conjunto solución de la ecuación mostrada :

b) R

{5}

1 x

2 c) R - {0} e) R

xn | 0

93.

Resolver :

100. Si : 5 2x 1

1 x

|y| 3 6

2

;1

a) d) [3 ; 94.

b) e)

1 11 ; ] 2 7

3;

c)

101. Al resolver :

| xy 1| y 1006 hallar la variación de "x", si "y" toma su mínimo valor entero.

Si : a, b, m R. Resolver para "x".

a) R

b) R

d) R o

e) R o

c) R

b) 0 < x < 1 d) -1 < x < 0

(x

2)2 | x 2| 6

Resolver : a)

e indicar el número de valores enteros de "x". a) 4010 d) 2006

b) 4009 e) 2001

c) 4011

d)

2;4

b)

1; 4

0;4

1; 5

c)

2;5

e)

103. Resolver :

|| x 1| x |

x

b) [0 ; 1

c)

Resolver :

x 0 |x| 6 e indicar un intervalo solución. a) [ 6 ; 0 d) 97.

a) 5 < x < 10 c) 1 < x < 2 e) -1 < x < 1 102. Resolver :

| 2x | | x 2006| | x 2006|

96.

b) |y| < x d) |y| x

a) x + |y| < 0 c) |x| |y| > 0 e) |y| |x| < |

| mx ab| | x || m b | | b || x a |

95.

27

entonces :

e indicar un intervalo solución. 1 ;5 2

4 2, y

|x| 3 3

6;

b)

2; 5

e)

0;

; 6

c)

a)

1 ; 0]

d)

; 0]

x3 1

0

a) [ 1 ;1

b) R o

c) [-1; 1]

d) [ 1;

{1}

e) R - {1}

e)

104. Resolver : |3x + 8 | < 9x + 1. a)

;

3 4

b)

;

3 4

Resolver :

x2 |x | 1

5 ; 0]

d)

1 7 ; 9 6

7 ; 6

c)

1 7 ; 9 6

e)

3 7 ; 4 6

105. Resolver : 98.

Hallar el máximo de :

| 2x 3 | | x 8 | | 2x 1| | 7x 8 | Indicando su intervalo solución.

|x| - |x - 2006| a) -2006 d) 2005 99.

b) 2006 e) 2004

c) -2005

Resolver : |3x - 1|< |2x - 3| a)

; 2

c)

4;

e)

4;

4 5 4 5

4 ; 5

b)

4 ;4 5

d)

2;

4 5

a) x [ 11 ;1

7 ; 5] 5 3

b) x [ 11; 0

7 ; 5] 4 2

x [ 11;1

7 ; 5] 5 3

d) x [ 11; 0

7 ; 5] 5 3

c)

7 e) x [1 ; 5

5; 3

]

0

106. Dados los conjuntos :

113. Dados los conjuntos de números reales :

A

{x R / | x 2| | x 1|}

S {p R / 2p 3

B

{x R / | x 4 | | x 2| | x 3 |}

T

entonces : A

B es igual a :

a)

1; 9

b)

1;

d)

1;

e)

1; 2 2

1; 2

c)

x2

Entonces : S

T , es :

a) R

b)

0; 1

e)

1 ;1 2

;1

0 , podemos afirmar :

a) x = {-1} c) x > 0

b) x = {0; 1} d) x < 0

e) x 108. Resolver : |x| x 2006

; 2006

{0}

c) R - {2006} e) R

0

b)

; 20 ;20 2006 006

d) R

{2006 6} 6}

|x| y x

|x

4

2 2

10 | | x |

;0

b)

; 1

1

Hallar el conjunto de valores de "y", cuando "x" toma su mayor valor entero. a)

;0

b)

1; 1

c)

;1

d)

;1

1; 2

e)

;2

2; 4

1 a 6

8x

2

c)

0;

d) R o

b) [0; 1]

b) a <

1 2

d) a

e) a

1 4

111. Resolver e indicar un intervalo solución de : ||2 - x|-3| < 1 a)

2;0

b) [4 ; 6

d)

3;0

e)

c) [ 2 ; 0

4 ;7

112. Resolver :

|| x 1| x 2 | x 2 b) [-6; 1] e) [-2; 4]

1

116. Resolver : | x 2

3| x 2

c) R

e) R

a) [0; 5] d) [-8; 2]

a) a < 1

e) [1 ;

| 7x 1| | 3x 1| | 2 4 x | 0 ;1

1 4 x

a

Luego, de "a", se puede afirmar :

110. Resolver :

a)

0}

1 2

115. Si : | x | 3 , entonces :

e indicar un intervalo solución.

d)

0;

a

1

y

109. Resolver :

a) [2 ;

c)

2b

114. Dadas las inecuaciones :

x |x| 1

a)

{q R /| aq b | | a b aq |;

d)

107. Al resolver :

6 p}

3 c) [-8; 4]

a)

; 3

3 ; 2

b)

; 3

3 ;1 2

c)

;3

d)

3;

e)

3;

1;

3 2

1;

c) a

x

1

117. Resolver la desigualdad :

119. Dadas las desigualdades :

| x 4 | | 2x 6 | 10 3

Dar como respuesta la suma entre el mayor valor entero negativo y el menor entero positivo que verifica la desigualdad. a) 5 d) 2

b) 4 e) 3

{x R / x 2 1

2 (x 2)

(y 3)|1 | axy || 0 ; a

d)

| x 1| 0} ,

b) [ 2 ; 2]

2 ;1

0

a) Menor que 2. c) Menor que 2. e) Menor que 1.

e) [ 2 ;1]

c)

b) Menor que 0. d) Menor que 1.

120. Resolver :

1

entonces, el conjunto R-A está dado por : a)

0

Luego, podemos afirmar que "x y" es :

c) 1

118. Si el conjunto :

A

x 2y 2

2; 2

1 |x | 1

2

e indicar un intervalo solución.

a)

1; 2]

d)

;

b) [ 2 ; 1 3 3 ] e) [ , 2] 2 2

3 c) [ ; 2

Claves 01.

-

31.

-

61.

c

91.

d

02.

a

32.

-

62.

-

92.

c

03.

b

33.

b

63.

b

93.

d

04.

-

34.

d

64.

d

94.

c

05.

c

35.

-

65.

a

95.

c

06.

e

36.

e

66.

a

96.

d

07.

a

37.

e

67.

d

97.

d

08.

a

38.

-

68.

b

98.

b

09.

c

39.

-

69.

d

99.

d

10.

-

40.

a

70.

c

100.

b

11.

a

41.

-

71.

a

101.

b

12.

e

42.

b

72.

c

102.

b

13.

a

43.

d

73.

e

103.

a

14.

d

44.

d

74,

d

104.

d

15.

d

45.

d

75.

a

105.

a

16.

b

46.

a

76, 7

d

106.

a

17.

e

47.

c

77. 77

b

107.

e

18.

a

48.

a

78.

d

108.

a

19.

c

49.

a

79.

c

109.

e

20.

e

50.

b

80.

b

110.

e

21.

b

51.

a

81.

c

111

a

22.

d

52.

d

82.

c

112.

e

23.

c

53.

d

83.

b

113.

e

24.

b

54.

c

84.

b

114.

b

25.

b

55.

e

85.

d

115.

d

26.

d

56.

b

86.

d

116.

b

27.

b

57.

d

87.

d

117.

b

28.

d

58.

a

88.

e

118.

d

29.

e

59.

e

89.

e

119.

d

30.

b

60.

c

90.

e

120.

e

Capítulo

15

RELACIONES

FUNCIONES Propiedades :

1 . Definiciones Previas

I. El producto cartesiano no es conmutativo :

1 . 1 . Par ordenado : Es un conjunto de dos elementos considerados en un determinado orden. Si los elementos del par ordenado son "a" y "b", al conjunto se le denota por (a; b) y se define de la manera siguiente :

A B

II. El número de elementos A B es igual al número de elementos de B A y se obtiene según la fórmula : n ( A B)

(a; b) = {{a}; {a; b}}

Donde : a = primera componente del par b = segunda componente del par Propiedades : I.

(a; b)

II.

(a; b) = (c; d)

(b; a);

a b a=c

b=d

1 . 2 . Producto Cartesiano : Dados los conjuntos no vacíos A y B, el producto cartesiano de A por B (en ese orden), se denota así A B y se define de la siguiente manera : A B

{(a; b) / a A

b B}

Donde : A = conjunto de partida B = conjunto de llegada Ejemplo : Dados los conjuntos : A = {1; 2; 3} B = {-1; 2} Determinar : A B B A Resolución : Para , A B , tenemos : A B {1; 2; 3} { 1; 2}

A B = {(1; -1), (1; 2), (2; -1), (2; 2), (3; -1), (3; 2)}

B A

n (B A)

n(A ). n(B)

2 . Relación Binaria 2 . 1 . Definición : Dados dos conjuntos no vacíos A y B, se dice que R es una relación de A en B (en ese orden), si y sólo si, R es un subconjunto de A B , es decir : R A B R

{(a ; b) / a B

b B

a R b}

Donde : a R b, indica la relación que existe entre los componentes "a" y "b". Ejemplo : Dados los conjuntos : A = {1; 2; 4}

B = {2; 3}

Determinar la relación de R de A en B definida de la manera siguiente :

R

{(a ; b) / a A

b B a

b}

Resolución : Hallar el producto cartesiano de A por B. A B = {1; 2; 4} {2; 3}

A B = {(1; 2), (1; 3), (2; 2), (2; 3), (4; 2), (4; 3)} observar que los elementos de R son todos los pares (a; b) A B / a

b . Luego, tenemos :

R = {(1; 2), (1; 3), (2; 3)} Para B A , tenemos : B A { 1; 2} {1; 2; 3}

B A = {(-1; 2), (-1; 2), (-1; 3), (2; 1), (2; 2), (2; 3)}

2 . 2 . Relación en A : Dado el conjunto no vacío A, se dice que R es una relación en A, si y solamente si, R A A .

2 . 3 . Clases de Relación :

FUNCIONES

Sea R una relación en A ( R podrá ser :

A B ), luego R

I . R ef le xiva

a A

(a ; a) R

II . S i m é t r i ca

(a ; b) R

(b ; a) R

III. Trans itiva

(a ; b) R (b ; c) R

(a ; c) R

I V. De equivalencia Siempre y cuando sea a la vez reflexiva, simétrica y transitiva.

Se define una relación en A de la manera siguiente : R = {(1; 1), (1; 2), (2; 2), (3; 3), (2; 1)} ¿R es una relación de equivalencia? Resolución : Si R es una relación de equivalencia, deberá ser reflexiva, simétrica y transitiva a la vez.

a R

(a ; a) R

1

A

(1; 1) R

¡Correcto!

2

A

(2; 2) R

¡Correcto!

3

A

(3; 3) R

¡Correcto!

Evidentemente, R es reflexiva. Simét rica (a ; b) R

(1 ; 2) R

(2 ;1) R

(b ; a) R ¡Correcto!

Evidentemente, R es simétrica. Transitiva (a ; b) R (b ; c) R

Dada una relación F de A en B (F A B) , se dice que F es una función de A en B si y sólo si para cada x A existe a lo más un elemento y B , tal que el par (x ; y) F , es decir, que dos pares ordenados distintos no pueden tener la misma primera componente. Ejemplo : ¿Cuál o cuáles de las siguientes relaciones,

R1

{(2 ;1), (0 ; 3), ( 1 ; 7)}

R2

{(3 ; 0), (4 ; 0), (5 ;1)}

R3

{(5 ;1),(4; 1),(4; 2)}

son funciones?

Ejemplo : Dado el conjunto A = {1; 2; 3}

Reflexiva

1 . Definición :

(a ; c) R

(1;1) R (1; 2) R

(1; 2) R ¡Correcto!

(1 ; 2) R (2 ; 2) R

(1 ; 2) R ¡Correcto!

(1; 2) R (2 ; 1) R

(1 ;1) R ¡Correcto!

Evidentemente, R es transitiva. R es una relación de equivalencia.

Resolución : De acuerdo con la definición, se observa que:

R1 es función R 2 es función R 3 no es función, ¿por qué? Porque

(4 ; 1) R 3

(4 ; 2) R 3 ,

siendo

pares

ordenados distintos. 1 . 1 . Propiedad Siendo F una función, se verifica lo siguiente :

(x ; y) F (x ; z) F

y

z

2 . Dominio omi om m i io y Rango de una función F 2 . 1 . Dominio om m de F = Dom(F) (DF ) denominado también pre imagen, es el conjunto de los primeros elementos de la correspondencia que pertenece al conjunto de partida. 2 . 2 . Rango de F = Ran(F)

(RF ) denominado también imagen, recorrido o contra dominio, es el conjunto de segundos elementos de la correspondencia que pertenece al conjunto de llegada. Ejemplo : Dada la relación funcional representada por el diagrama digital.

A

A

1 2 3 4

0 -1 2 4

Determinar la función, indicando su dominio y rango.

Resolución :

1 . 1 . Teorema Toda recta vertical, trazada a la gráfica de una función, la corta sólo en un punto.

Del diagrama, se tiene : F = {(1; 2), (3; 0), (4; 2)}

Fig. (1)

y

F

De donde es evidente que :

DF = {1; 3; 4}

R F = {2; 0}

2 . 3 . Propiedad : Sea F una función de A en B, luego se denota por: F:A B y se cumple lo siguiente :

DF

A RF

x

F corresponde a la gráfica de una función. y

Fig. (2)

B

H

3 . Apli ca ci ó n 3 . 1 . De fi ni ci ón Dada una función F de A en B, F : A B . Se dice que F es una aplicación, si y sólo si, su dominio es igual al conjunto de partida.

F es aplicación

DF = A

FUNCIÓN REAL DE VARIABLE REAL L 1 . Definición : Dada una función F de A en B, F : A B , si A y B son subconjuntos de los números reales R, se afirmará que F es una función real de variable real.

F:A

B,A

R B

R

Debido a ello, F tendrá una representación gráfica en el plano cartesiano (x.y), la cual viene dada por un conjunto de puntos generados al establecer la relación de correspondencia entre la variable independiente "x" y su imagen la variable dependiente "y", es decir : F

{(x ; y) R2 / x DF

y

F(x)}

la igualdad mostrada : y = F(x) expresa la regla de correspondencia de la función real F.

x

H no corresponde a la gráfica de una función. 1 . 2 . Criterios para determinar el dominio y el rango I . Para el Dominio : Se despeja la variable "y", para luego analizar la existencia de su equivalente. II . Para el Rango : Se despeja la variable "x", para luego analizar la existencia de su equivalente. A veces, el rango se determina a partir del dominio. Observación : Frecuentemente, para determinar dominios y rangos es necesario reconocer la existencia de las expresiones dadas dentro del conjunto de los números reales, así pues, tenemos : Ejemplo : Determinar el dominio y el rango de la función F, donde : * *

A R B A R

B A

0 0

Ejemplo : Determinar el dominio y el rango de la función F, donde :

F:R

R/y

F(x)

2x 1 x 3

Resolución :

Ejemplo :

De acuerdo con los criterios para el dominio : y 2x 1 x 3 y R x 3 0

Dadas las funciones :

x x R {3} DF R {3}

3

F:R

R/y

F(x)

x x2

G:R

R/y

G(x)

1 x

¿son iguales? Resolución :

para el rango :

2x 1 x 3 xy - 3y = 2x + 1 xy - 2x = 3y + 1 (y - 2)x = 3y + 1

De acuerdo con la definición, veamos si se verifican las condiciones :

y

3y 1 y 2

x

x R

y 2 0 y

x x2

I. Para F : y

y R

x2

x

x R {0}

0 DF

2

0

R {0}

y R {2} RF

R {2}

Ejemplo : Determinar el rango de la función, la cual viene dada por : F

R

R/y

F(x )

2x 3 ; x

5 ;10]

Resolución :

7

y RF

y

17

7 ; 17] 7 ;17]

2 . Igualdad de Funciones 2 . 1 . De fi ni ci ón Dadas las funciones F y G, tal que : F:R

R/y

F(x )

G:R

R/y

G(x )

se dice que éstas son iguales : F = G, si y solo si verifican simultáneamente las condiciones : I . DF II . F(x )

DG G(x) ;

x

x

x R {0}

DG

0

0

R {0}

Observar que : DF

DG .

II . Regla de correspondencia para F.

Observar que el rango se puede encontrar a partir del dominio, pues con x 5 ;10] bastará deter-minar la extensión de : y = 2x - 3. Veamos : x 5 ;10] Por condición : de donde tenemos : 5 x 10 multiplicando por 2 10 2x 20 sumando -3 7 2x 3 17

observar que :

1 x y R

II . Para G : y

x DF

DG

F:y

F(x)

x x2

1 x Regla de correspondencia para G. como x

0 : F(x) =

G: y

G(x)

1 x

Observar que : F(x) = G(x).

F

G son iguales

1 . FUNCIONES ESPECIALES

1 . 4 . Función Valor Absoluto

1 . 1 . Función Lineal

F : y = F(x) = |x|

F : y = F(x) = mx + b y |x|

y

x; x 0 0; x 0 x; x

F

m = pendiente m = Tg

0 y

x

F

1 -1

DF

R

FF

x

1

R DF

1 . 2 . Función Identidad

R

RF

[0 ;

1 . 5 . Función Signo

F : y = F(x) = x F : y = F(x) = Sgn(x)

y F

45º

y

x

1; x 0 0; x 0 1; x 0

Sgn(x)

y 1

DF

R

FF

F

R

x

1 . 3 . Función Constante

-1

F : y = F(x) = k; k R

DF

R

RF

{ 1, 0 , 1}

y 1 . 6 . Función Escalón Unitario

k

F

F : y = F(x) = u(x)

x y

DF

R

RF

u(x)

0; x

0

1; x

0

y

{k}

F

x

DF

R

RF

{ 0 ; 1}

1 . 7 . Función Máximo Entero F:y

1.10. Función Raíz Cuadrada :

F(x)

[[x ]]

F:y

[[3 ; 15]]

x

y

Definición : Dado el número real "x", el máximo entero de "x" es la relación funcional denotada por [[x]] y definida como el mayor entero menor o igual que "x", veamos algunos ejemplos : *

F(x)

F

3 ¿por qué?

Porque 3 3 ;15 *

[[4]]

x

4 ¿por qué?

Por que 4

DF

4

[0 ;

RF

[0 ;

1.11. Función Raíz Cúbica

Teorema : [[x]]

y

y

x

F:y

y 1; y Z

3

F(x) y

y F

3

F

2

-3 -2

-1

x

1

-1

1

2

3

x

DF

-2

R

F:y

DF

R

RF

RF

F:y

F(x )

R F

x2

F(x)

y

x

F

x

RF

F(x)

x

3

RF

R {0}

Conociendo la gráfica de la función F, donde: F : y = F(x)

F

x

R

R {0}

y

y

DF

DF

2 . DESPLAZAMIENTOS Y GIROS DE LA GRÁFICA DE UNA FUNCIÓN

[0 ;

1 . 9 . Función Cúbica Simple : F:y

1 x

y

1 . 8 . Función Cuadrática Simple :

R

R

1.12. Función Inverso Multiplicativo

-3

DF

x

RF

R

x

y considerando un número positivo "h", tenemos :

2 . 1 . Despl azamient o Horizo ntal

2 . 4 . Giro con respecto al eje "y"

y y

y

F(x+h)

F(x-h) x

"h" unidades hacia la izquierda

F(-x)

x

x

"h" unidades hacia la derecha

El eje "y" se comporta como si fuese un espejo.

2 . 2 . De spla za mi ento Ver ti ca l 2 . 5 . Giro producido por el valor absoluto y

y

F(x)-h

y

F(x)+h

|F(x)| x

x

x "h" unidades hacia abajo

"h" unidades hacia arriba

La parte de la gráfica debajo del eje "x", se refleja por encima del mismo.

2 . 3 . Giro con respecto al eje "x"

y -F(x)

x

El eje "x" se comporta como si fuese un espejo.

EJERCICIOS PROPUESTOS 01.

Determinar el valor de "m.n", si se cumple que :

06.

¿Cuál o cuáles de las siguientes gráficas representa a una función?

(m+n; 3) = (9; 2m-n) a) 10 d) 40 02.

b) 20 e) 50

c) 30 x

A

{x Z / 12 6x

B

{x Z / x 2

a) 40 d) 25

(I)

18} y

b) 35 e) 20

c) 30

a) Sólo I d) I, III y IV 07.

= = = = =

{(1; 2), (2; 4} {(0; 1), (2; 4), (3; 5)} {(1; 2), (2, 4), (3; 6)} {(1; 2), (2; 4), (4; 8)} {(2; 4), (1; 6)}

08.

05.

b) -6 e) -9

09.

b) 10 e) 16

Dadas las funciones :

Calcular : F( 2) F(d

c) 16

a) 2 d) 8 10.

c) Sólo III

2)

F(d) G(

b) 4 e) 10

)

c) 6

Determinar el dominio de la siguiente función :

I. F = {(2; 3), (2; 4), (3; 4)} II. G = {(3; 1), (-1; 4), (4; 3)} III. H = {(-2; 2), (-1; 3), (2; 3), (4; 2)} b) Sólo II e) II y III

c) 12

F = {(2; 6), (3: b), (3; a-b), (d; a)} G = {(4; d+1), (4; 6), ( ; b)}

¿Cuál o cuáles de los siguientes conjuntos representa a una función?

a) Sólo I d) I y III

c) -7

Del problema anterior, dar la suma de elementos del dominio y rango de la función. a) 8 d) 14

Calcular el valor de : a+b+c+d+e+f+g. b) 14 e) 20

c) Sólo I y IV

representa una función.

Sea el conjunto : A = {1; 2; 3} y sean las relaciones R, S y T definidas en A; donde R, S y T son reflexiva, simétrica y transitiva, respectivamente; si :

a) 12 d) 18

b) Sólo II y III e) II y IV

Calcular el valor de "ab", si el conjunto :

a) -5 d) -8

R = {(1; a), (2; 3), (2; b), (3; c)} S = {(1; 3), (e; d)} T = {(1; 2), (2, 3), (f; g}

(IV)

F = {(2; 5), (-1; 7); (2; a+2b); (3; a-9); (3; 2b)}

R= {(x; y) A B/y =2x} R R R R R

x

(III)

B = {2; 4; 6}

Determinar por extensión la relación R, de A en B, definida por :

y

x

Sean los conjuntos :

a) b) c) d) e)

(II)

y

9}

A = {1; 2; 3}

04.

x

Sean los conjuntos :

Calcular el número de elementos que contiene el producto cartesiano A B.

03.

y

y

f(x)

a) [ 5 ; b)

5;

d) [ 5 ;

x 5 x2

4

{ 2 ; 2} c) R { 2 ; 2} e)

2; 2

11.

Determinar el dominio de la siguiente función : 4x 1 2x 3

g(x) 3 a) R {2 ; } 5 3 1 c) R { ; } 2 5 e) R { 2} 12.

18.

3x 2 5x 1

F:R

x

a) [ 3 ; 7] {1}

3

19.

3

7 x

x

2

b) [ 3 ; 1

c) [ 3 ; 7] { 1; 1} d)

3;7

1 ; 7] { 1 ;1}

c)

x x 3 c) R

{1} 22.

Hallar el rango de la función : x2

a) [ 3 ;

b) [ 3 ; 0

d) [ 0 ;

e)

R/y

5 3 2 2

x2

F(x)

4x 7 ; x

b) [2; 11]

12 ; 39]

5 ; 4]

c) [3; 39]

12 ; 39

e)

F(x) 16 4 x x 2 ; x [ 8 ; 2

R/y

b)

20 ;16]

c) [ 16 ; 20]

d)

16 ; 20

Determinar el rango de la función : g(x ) a)

c) [ 3 ;

c)

;

e) x2

b)

d) R-

d) R {31}

23.

31

; 31]

a) [31 ;

a) [10 ; 13

b) [15 ; 21

d) [15 ; 30

e) [35 ; 65

/y

F(x) c)

; 5

10 ;13 ]

6x

4

b) [ 5 ;

5;5

d) [ 5 ; 5]

; 5]

F

2x 5

x2

Sea la función :

c) R

Determinar el rango de la función F, donde:

[15 ; 30

6;

3

Determinar el rango de la función :

F : [5;8

d)

20 ;16 a) [ 2 e) R

{3}

f(x )

17.

2 3 1 2

Determinar el rango de dicha función.

(x, y) / y

b) R e) R

3

b)

Sea la función : F:R

f(x )

16.

21.

e) R { 5}

{ 3} {0}

con dominio en el

2

Determinar el rango de la función F, donde:

d)

5;

H

3 x

2

a) [12; 39]

Indicar el rango de :

a) R d) R

4

F:R

b) R { 3}

4 d) [ ; 3

15.

20.

4 3x x 5

a) R {3}

14.

6 x x 4

e) 18

f(x )

; 5

{2}

a) 14

Determinar el rango de :

c)

e) [3 ; 5

Sea : f(x )

3 ; 5]

c)

conjunto Z. Hallar la suma de elementos del rango.

1

e) R { 1 ;1} 13.

3 ;11]

b) [3 ; 5

d) R o

Determinar el dominio de : 4

2x 3 ; x

F(x)

3;5

a)

d) R {4 ;1}

h(x)

R/y

Deteminar el rango de F(x).

3 ; 2} 5

b) R {

Sea la función :

(x, y) R 2 / y

se sabe que su rango es : Hallar : 9b + a. a) 2 d) 0

b) 1 e) 4

x2

3 9

a ; b] .

c) 3

24.

Dada la función :

30.

F(x)

2x

2

3x

t| |t| 3 ; redefina la función en los t intervalos de :

2;x R

25.

26.

a) [ 1; 1]

b)

d) [ 0 ;1 ]

e)

2x

F(x )

a) 8 d) 2 31.

c) [ 1 ;

Sea la función : f : R

x 2 2x 3 | x 10 | | x |; 2

Tal que : B

x

10 .

A.

f(x)

a) 80 d) 106 32.

b) 96 e) 115

c) 103

Hallar el rango de :

G {(x , y) R 2 / y

c) 5/2

R / f(x )

c) 4

x [2 ;10] . Hallar : A + B.

x2 1 2x 2 x 2

b) 2/3 e) 1

f(4)

"A" es el menor valor real y "B" es el mayor valor real.

Determinar el menor valor que asume la función real de variable real cuya regla de correspondencia es :

a) 2/5 d) 5/3

1)

Para la función :

;0]

F(x)

f(

5)

b) 6 e) -10

f(x)

x2 1

1; 1

y [0 ;

Luego, calcular : 5 f(

c) 8

Determinar el rango de la función real de variable real, cuya regla de correspondencia es :

y

27.

; 3 ,[ 3 ; 0

b) 7 e) 10

y

|3

f (t )

donde : Ran(F) [ a ; a 1 Calcular "a". a) 6 d) 9

Dada la función :

A

A Q , llamada función constante.

5 x

3 x}

a) y [ 2 ; 4]

b) y [0 ; 4]

c) y R

d) y [2 2 ; 4]

e) y [0 ; 2 2 ]

2

Se sabe que : f (2005) f(1003) 12 . 10

Hallar : E

k 1

a) 40 d) 20

33.

Determinar el dominio de la función F, donde :

f(k ) .

F:R

b) 20 e) 40

c) 30

a)

0;

Si :

( 2x 1 ; x) R 2 / x 2x 3

34.

0 ;10 ]

a) a + 3b = 25 c) 6a + 23b = 25 e) 5a + 6b = 36

x

c) [ 0 ; 4 ]

e) [ 4 ; 4 ]

x2

x6

3x 4

a) [1 ;

Hallar : f(2x 1) f(2x 2 ) . c) -4x

c) 9

c)

2; 2

e)

4 ;1 ]

R

3x 2 12 ; encontrar su dominio, si

su rango es [ 12 ;16

.

b) 2x - 1 e) 2x

b) 8 e) 11

Sea la función polinomial : f(x) : R f(x )

; x [0 ; 2

x , e indicar el número de

3| x

a) Infinitos d) 10 35.

2x 1; x [2 ; 5

si : x [1 ; 3 2

|x

valores enteros que posee.

b) 3a + 6b = 10 d) 6a + 46b = 44

Si tenemos :

a) 14 2 d) x

2

Hallar el dominio de : f(x )

entonces, la relación correcta entre los valores de "a" y "b", es :

f(x)

3

a ; b] es el dominio de la función F, definida por: F

29.

F(x)

b) [ 0 ;

d) [ 0 ; 4 28.

R/y

. b)

16 ;12

d)

1; 4

36.

Dada la función :

41. n

F (x )

a

n

n

x ;n N

a

Dada la gráfica de F(x) :

I. Dom(F) = R; n impar II. Dom(F) = [-a; a] n par III. F(x) = F(-x);

3 -6

n par

-1

0

Indicar el valor de verdad. a) VVV d) FFV 37.

b) VVF e) FFF

Indicar lo correcto : 5 ; 2] a) Dom(F) b) Ran(F) [ 5 ; 2 6; 1 c) Ran(F)

A

{(t 2

B

{(t 5 ; t) / t R}

d) Dom(F)

C

{(t 2 1 ; t) / t R}

e) Ran(F)

3 ; t) / t R}

D {(3t 2 ; t) / t R}

42.

b) A y B. e) B y D.

x

4

-2 -5

c) VFV

¿Qué conjuntos de pares ordenados son funciones?

a) Sólo B. d) Todos.

y

0

6 ; 1] [0; 4 2;0

Graficar : F(x) = 3x - 2 y

a)

c) Sólo B.

0; 3] 0; 3] 0; 4]

y

b)

x

x

38.

Calcular el rango de la función : f(x ) Si : DF a)

39.

x

x [1 ; 9] .

c)

15 ;1 ]

e)

0 ; 15 ]

y

c)

1 ; 15

b)

1 ; 15

d) [

15 ; 1]

a) [ 0 ;

b)

d) R

e)

x

5 | 1 x) 5 x

1;

c)

;0]

;4 ]

Sea la función lineal : f : R correspondencia es :

f(x) | ax 2

43. R cuya regla de

Graficar la función : F(x ) y

a)

c) a

8 ;1 5

e) a

8 ;0 5

3

2 y

b)

b) a

x

3

x

-3

y

c)

0;8 / 5

x

3ax a 2 | ax 2 ax 3

indicar los valores del parámetro real "a", que definen completamente la función "f". a) a

x

y

Determinar el rango de la función : (| x

y

d) x

e)

F(x )

40.

2x

y

d)

1; 5 / 3

d) a R

-3

x

-2

e)

y

2 3

x

-2

2

x

44.

Graficar : F(x) =

x

y

a)

47.

2 y

b)

x

2 y

c)

48.

y

d)

-2

a) 8 d) -8

x

2

2

x

Hallar el área de la región formada por las gráficas de las funciones F y G, tales que :

a) 6 u2 d) 12

b) 8 e) 16

Graficar : F(x ) | x 2

x

-2

y

a)

x 2 ; si : x x ; si : x

Graficar : F(x )

y

c) 9

3| y

b)

3

3

x

0 0 y

c) a)

c) -2

F(x) = |x 5| y G(x) = 3.

49.

45.

b) 2 e) 5

x

y

e)

Luego de graficar : F(x ) x 2 6 x 14 , se obtiene una parábola cuyo vértice está dado por el par ordenado (a; b). Calcular : a + b.

x

d)

y 3

y

b)

x

x -3

x

x

y

e) y

c)

y

d)

x

x

-3

x

50.

Se tiene la gráfica de la función F(x) :

y

e)

y x

x

46.

Graficar : F(x) = |x - 3|+ 2. y

a)

y

b)

3

¿Cuál de las siguientes gráficas corresponde a : H(x) = F(x-3) + 3 ?

2 2

x

x

-3

y

a)

y

b)

3 x y

c)

2 -3

-3 y

c)

2 x

3

x

3 x

y y

e) x

-3

x

x

y

d)

3

e)

3

y

d)

x -3

51.

Obtener la pendiente de : F(x )

56.

Ax B

52.

b) 4 e) 1

abx b 2 ; ab

a) 1945 d) 1960 57.

9 b3 2 u a

d) ab 53.

b)

a) 7 d) 16 c)

3

2b3 9a

58.

2x 2

3x

7x 2

F(x)= x 2 - 4x - 5

59.

6

x

60.

En la función : f(x ) (x a)2 b . El valor de "x" que hace que la función acepte a 7 como mínimo valor, es 7. Hallar "ab".

55.

4

3px

c) ( 1 ; 3)

b) 12 e) 32

c) 14

Graficar : F(x )

a)

x ;x

1

2

1

x ;x y

y

b) x

x

c) 49 c)

y

f(x) 2x 2 12x 1 tiene un máximo o un mínimo. ¿Cuál es su valor? b) Un máximo, 19. d) Un mínimo, 3.

x

e)

y

d)

La función cuadrática :

a) Un mínimo, 19. c) Un máximo, 3. e) Un máximo, 20.

p

Determinar el área de la región formada por la función: F(x) = -|x| + 4 y el eje de las abscisas. d) 16

c) 28

c) 15

b) (1 ; 1) e) (1 ; 3)

a) 8 u2

b) 14 e) 0

5x 9

se elige "p", de manera que sus gráficas tengan un único punto en común. Entonces, las coordenadas (x; y) de dicho punto son : a) (0 ; 0) d) (1 ; 3)

a) 7 d) 49

3 y g(x)

b) 8 e) 20

g(x)

y

54.

2x

Dadas las funciones : f(x )

9 b3 e) 2a

b) 42 e) 24

x2

f(x )

Hallar el área de la región sombreada :

a) 21 u2 d) 14

c) 1955

e indicar la suma de coordenadas de uno de ellos.

2a 9b

b) 1950 e) 1965

Hallar los puntos de intersección de las gráficas de :

0

g(x) 2b 2 con el eje de las ordenadas.

a)

60 x 1500

Encontrar la ganancia máxima.

c) 3

Hallar el área de la región formada por las gráficas de las funciones :

f(x)

2x 2

G(x)

sabiendo que la gráfica F(x) pasa por el punto (8; 38) y por el punto (0; -2). a) -2 d) 5

La ganancia de cierta compañía está dada por :

2

y

x

1 1

x

61.

La gráfica de la función : F(x) = x|x|; es : a)

64.

y

Hallar el área del triángulo sombreado, si "L" es una recta cuya pendiente 3). y

b)

y

A

(-1; 15)

x x

c)

y

y

d)

x L

x

a) 15 u2 d) 28

x

e)

y

65.

b) 21 e) 32

c) 24

Calcular el área de la región sombreada limitada por las funciones indicadas. y

x

H(x) = 6 - x - 2

62.

G(x) = 4

Las gráficas corresponden a las funciones: 1 2 x 2 si la máxima longitud vertical "d" se encuentra en la abscisa "a". Calcular "a". f(x )

x2

2x

g(x)

x

a) 24 d) 16

y

b) 32 e) 20

c) 48

g

66. d

63.

4|

f

a

a) 1 d) 1/3

Graficar : F (x ) | x

x

b) 3/2 e) 3/4

a)

y

y

b)

c) 2/3

Dada la gráfica de F(x) :

c)

y

x

-4

x

16

y

d)

y 4

5

x

-16

x

16 y

2 -7

-2

e)

x

7

1

4

-1 67.

-5

x

Indicar la gráfica de la función : F(x )

se cumple :

Dom (F)

Ran (F) [a ; b

a)

[c ; d]

Calcular : a + b + c + d. a) 0 d) 13

b) 1 e) 13

b)

y

x

c)

3

x

x2 y

x

c)

y

71.

y

d)

Si "h" es una función lineal de pendiente 3 e intersección con el eje "y" igual a 5, hallar la regla de correspondencia de la función g(x), si:

x

x

g(x) - x = h(1) + h(x+1) e)

a) g(x) = 4x + 4 c) g(x) = 4x +12 e) g(x) = 3x + 12

y

x

72. 68.

b) g(x) = 4x + 16 d) g(x) = 3x +13

En el siguiente gráfico : y

Hallar el área de la región sombreada :

y

F(x)= x 2 - 2x - 3

(2; 0) x

Hallar la ecuación de la parábola si el punto (3, 2) pertenece a ella y su rango es el intervalo [

5

a) 36 u2 d) 12 69.

b) 18 e) 25

73.

¿Cuál de los siguientes puntos no está en la gráfica?

x2

Graficar : F(x) Si : m < 0. a)

c) y

2

x 2

2mx

2 3x

3x

b) y

y 2 2

d) 2x

x2 2

3x 3x

y

y

Indicar cuántos puntos de la forma (a; b) donde : a y b Z se encuentran dentro de la zona limitada por las funciones :

a) 21 d) 12 74.

b) 19 e) 17

c) 14

De la gráfica :

y

m2 . b

y

b)

y

S

x

x y

d)

y

a) ab x

d)

y

75. x

x

Si el área "S" del rectángulo es máxima, hallar dicha área.

x

e)

2 2

a

c)

.

F(x) = (x+2)(x-2) y G(x) = (2+x)(2-x) 1 1 c) ( ; ) 2 3

b) (

d) ( 1; 1)

3x

x x 1

1 ; 1) 2 e) ( 2; 2)

a) (0; 0)

2 a) x

e) 2x

c) 24

y

70.

x

1 ; 4

ab 3

b)

ab 2

e)

ab 6

c)

ab 4

Un rectángulo tiene dos de sus lados sobre los ejes coordenados y el cuarto vértice sobre la recta de ecuación y = 2x + 8. El área máxima que puede tener el rectángulo es igual a : a) 8 d) 11

b) 9 e) 12

c) 10

76.

Sea f, una función de proporcionalidad, tal que : f(3) + f(7) =20. Entonces, el valor del producto : f(21/5) f(5) f(7), es : a) 147 d) 1716

77.

b) 1470 e) 1176

79.

Según el gráfico de "f". y

c) 1170

1

f

Dado el gráfico :

x

-2

y

Indicar el gráfico : H(x) = f( x) V

a)

y

b)

y

1

2

c)

x

x

2

x

y

-1

d)

y

1

2

Donde : x2

6x 8

a) 1 u2 d) 4

b) 2 e) 5

x -1

y

e)

Hallar el área de la región sombreada. (V : vértice de la parábola).

78.

x

2

F(x )

1.

1 x

c) 3

-2

Si la gráfica adjunta, representa a : y = f(x)

80.

Dada la función "f" cuya regla de correspondencia es f(x ) x 2 2x a . Entonces, podemos afirmar que los gráficos adjuntos corresponden :

1

I.

II.

f

f

2 x

¿Cuál de las gráficas representa a : y = f( x) ? a)

III.

b)

1

-2 -2

2

c)

2

1 -2

-1

2 -1

x

-1

d) -2

e)

f

a) b) c) d) e)

El gráfico El gráfico El gráfico El gráfico El gráfico

II ocurre cuando a > 1. II ocurre cuando a < 1. III ocurre cuando a = 1. I ocurre cuando a < 1. II ocurre cuando a > 1.

x

Claves 01.

b

21.

c

41.

d

61.

b

02.

c

22.

b

42.

d

62.

c

03.

c

23.

c

43.

e

63.

a

04.

b

24.

b

44.

d

64.

c

05.

e

25.

a

45.

e

65.

b

06.

d

26.

a

46.

d

66.

c

07.

c

27.

a

47.

c

67.

b

08.

d

28.

d

48.

c

68.

b

09.

c

29.

c

49.

a

69.

d

10.

a

30.

a

50.

c

70.

d

11.

c

31.

d

51.

d

71.

b

12.

c

32.

d

52.

e

72.

a

13.

b

33.

c

53. 5 53 3

a

73.

e

14.

c

34.

d

54. 4. 4

c

74.

c

15.

c

35.

c

55.

b

75.

a

16.

a

36.

a

56.

b

76.

e

17.

b

37.

e

57.

c

77.

a

18.

c

38.

d

58.

d

78.

d

19.

c

39.

a

59.

d

79.

d

20.

c

40

e

60

d

80.

b

Capítulo

16

LOGARITMOS EN R

Función Exponencial

Función logarítmica

Siendo "b" un número positivo distinto de la unidad.

F:R

R/y

F(x)

F:R

bx

exp b (x)

R/y

F(x)

Log b x

Donde :

Donde : DF

R

RF

DF

0;

F : F(x)

bx ; 0

0;

RF

1.

b 1

F:y

F(x)

Log b x ; 0

b

1

y

y

1

1 x

F:y

F(x)

bx ; b

x

La función es decreciente.

La función es decreciente. 2.

R

Análisis de la gráfica

Análisis de la gráfica : 1.

Siendo "b" un número positivo distinto de la unidad.

2.

1

F:y

F(x)

Log b x ; b

1

y y

1

1

x

x La función es creciente. La función es creciente. Observación : La función exponencial es monótona e inyectiva, por lo último se afirma que dicha función admite inversa.

Observación : La función logarítmica es la inversa de la función exponencial y viceversa. Logaritmo (Log) Se define logaritmo de un número "N" en una base "b" positiva y distinta de la unidad, como el exponente " " que debe afectar a dicha base, para obtener una potencia igual al número dado inicialmente.

R epr es e nta ció n

Log bN

2.

M, N

........ (1)

0;

b 0;b

Log

Donde : Log = Operador de la logaritmación N = Número propuesto / N > 0 b = Base del logaritmo / b > 0; b 1 = Logaritmo / R.

3.

M

0;

b

M

1

Log

n R; b Log

b

M

b

N

0;b n

M ( ) b N

Log

1

n . Log

b

M

Definición :

b *

*

Log2 8

Log 5 x

x

2

4.

N ...... (2)

0;

2

8

x

3 x

x

25

1.

Log b N

b

*

5

*

12

b

M

b

0;b

Log

2.

N

b

5

x

4

5

x

9

3.

Propiedades generales :

2.

Log b 1

0

b

0; b

Log b b

1

1

(bn )

1

1.

Log 7 ( 10) ¡No existe en R!

M, N

0;

(b m )

m n

b

0;b

n m

1; n R

Log b b n

Un sistema de logaritmos se genera al asumir el parámetro "b" un valor determinado, como : b > 0; b 1, es fácil apreciar que existen infinitos sistemas de logaritmos, siendo los usuales los siguientes :

b 0;b

Log M b

Log

2.

1 b

N

Sistema de logaritmos naturales : También llamado sistema de logaritmos neperianos o hiperbólicos. Aquí, la base es el número inconmensurable "e" cuyo valor aproximado es : 2,7182. Log N e

Propiedades operativas : 1.

n

Sistema dee llogaritmos

Observación : En R no existe el logaritmo para números negativos. *

M

0 ; b 1 ; {m , n} R

3

0; b

bn

Log n (m b ) ( b)

Log 12 (x 4 )

b

Log

1

1 ; {m, n} R {0}

n

1.

0;b

Casos especiales :

2

Teorema : Reemplazando (1) en (2).

Log 5 3

n R {0} ; b Log

x

5

M

Log

b

(M . N )

LnN ; N

0

Sistema de logaritmos decimales : También llamado sistema de logaritmos vulgares o Briggs, aquí la base es el número 10. Log

10

N

LogN ; N

0

Conversión de Sistemas : 1.

a, b,c

De logaritmo natural a decimal

LogN 2.

Propiedad adicional :

0 , 4343 . LnN ; N

a

0

De logaritmo decimal a natural

*

5

Log7 12

R /b

Log bc

c

1

Log ba

Log7 5

12

Ecua cio ne s l oga rí tim icas

Ln N = 2,3026 . LogN ; N > 0 Cambio de base

Analizaremos cada uno de los casos frecuentes, veamos :

Dado un logaritmo en base "b", se le podrá representar en base "m", según la relación.

Primer caso :

x

se cumple : Log

Donde : N *

0

{m , b}

m

Log

R

N

m

se plantea : b a

b

{1}

Log 5 3

Caso especial :

{a , b} Log

b

R

a

se cumple : x 0 y se plantea : x = y

0 b

bx

Log b y 0;b 1

a

se cumple :

a

0

b

se plantea :

Log b bx

0

Log b a

x . Log b b

Log b a

x

Log ba

Inecua a ciones expo nentes Analizaremos cada uno de los casos existentes, veamos :

1 Log18 7

Log 7 18

0; b 1

Log b x

{1} 1 Log b a

b

Segundo caso :

Tercer daso :

Log 512

0

a

x

Log3 12 en base 5, será : Log 312

*

b

Log

N

Log b x

Primer caso : Siendo, 0 < b < 1.

Regla de la cadena :

bx

by

x

y

Verificando la existencia de cada uno de los factores en el conjunto R, se cumple :

bx

by

x

y

Segundo caso : Siendo, b > 1.

Log

*

b

a . Log c . Log d . Log e a c d

Log 2 5 . Log 5 7 . Log 7 8 Log 2 23

3 .1

3

3Log 2 2

Log 2 8

Log

b

e

bx

by

x

y

bx

by

x

y

Inecuaci one s l oga rít micas Analizaremos cada uno de los casos existentes, veamos : Primer caso : Siendo, 0 < b < 1

x>0

Log b x

Log b y

x

Log b x

Log b y

x

y y

y >0

Segundo caso : Siendo, b 1

Log b x Log b x

Antilogaritmo (Antilog)

x

Log b y Log b y

0

y

x

y

N R b

y

la base "b", de la manera siguiente:

x

0

También llamado exponencial, considerando que :

Cologaritmo (Colog)

0 b 1 , se define el logaritmo del número "N" en

Anti log

Teniendo en cuenta que : N

0

b

0,b

1

Se define el cologaritmo del número "N" en la base "b", de la manera siguiente :

Co log

b

N

Log

b

N

Log

1 ( ) b N

*

Antilog 2 4

*

exp 3 ( 2)

b

N

24

3

2

exp

Co log125 25

Log125 25 Log 2 3

(5 3 )

(52 )

N

b

16

1 9

Relación entre Operadores : Teniendo en cuenta que {x; b}

R / b 1;

se cumple : *

b

1.

Antilog b (Log b x)

2.

Antilog b (C o log b x)

3.

Log b(Anti log b x)

4.

Co log b ( Antilog b x)

x x

x x

1

N

EJERCICIOS PROPUESTOS 01.

Hallar :

08. M

Log

2

16 Log

27

9 Log

3 5

Calcular :

25

E a) 11 d) 13 02.

b) 121/12 e) 10

a*b

a 2 b2

a% b

Log 2 (a b)

09.

(5 * 3)

.

b) a4

16 d) a

2 e) 4 a

10.

1 a a b

p2

q2 2

)

20 .

11.

b b

a a

1 b

c)

a

Si : ab ; a , b , c R

{1} ,

ab

b) ab

d) -ab

e)

a) 1/2 d) 4

b) 2 e) 6

b) 1 e) -1/2

a) 2 d) 5 13.

1 Log ( ) 4

c) -5

Log 4 3

5(3

Log 4 x

b) 3 e) 6

)

36

c) 4

Dada la ecuación : xLog4 + Log (Log3) = Log(Log81) El valor de "x" que le verifica es :

2 Log 3 40 2

a b ; ab 1 64 .

ab

Resolver :

c) 3/2

b) -1 e) -1/2

(a b)

c) a b

Hallar "x", de :

7x

Efectuar :

3 Log 2 45 3

Co log c a Colog c b

a) 0

a) 0,5 d) 2 12.

b) 7 e) 3

Log abx

c) -2

3Log ( 2x ) 2Logx

b

Log (1 1) ; Log (1 1 ) ; Log (1 1 ) ; ..... 2 3

Si : a 3b 3

c) 1/2

b) 2 e) -1/2

Logp Logq

Indicar la suma de los 999 primeros términos de la sucesión :

a) 2 d) 1/2

Ln 3

Co log 6 Antilog 3 (Log 3 12 1)

E

c) 7

a b) a

e)

a) 1/2 d) 5

Ln 25

Calcular :

Si : Log2 = a; Log3 = b, hallar el logaritmo de 5 en base 6 en términos de "a" y "b".

d)

.

reducir :

b) 0 e) 4

a) 1

07.

b) 5 e) 1/10

Antilog c Anti log a b

Si se cumple:

a) 22 d) 8

06.

1 Log45 9

a) 1/2 d) 1/4 8 c) a

a) 8a

Calcular : Log q p Log pq

05.

2 Log3 5

E (3 a 2 %2 a 2 )

Log (

04.

1

a) 2 d) 1/5

Si :

Calcular : E

03.

c) 125/12

1

1 Log5 72 1 c) 1

a) 6 d) 5 14.

b) 1 e) 4

Resolver la ecuación :

x Log (1 2x )

a b

0 , hallar "x", de :

c) 8

c) 8

Hallar : a) 0 d) 3

x 1

x Log 5 Log 6

x 1 b) 1 e) 8

c) 2

15.

Hallar "x".

Log b xLog c xLog a c

Log a x Log bc a) ab d) a 16.

17.

24.

Dar la suma de soluciones de :

b) 1 e) 4

c) 2

d)

10

5 1

3

Log 2 x 1

10

10

5 1

26.

3

66

b)

ac b

c) abc

e) 2

Si : 10a

27 ; 10b

15 , hallar : Log2, en términos

a)

1 (a 3

3b

3)

b)

1 (a 3

c)

1 (3 b 3

a

3)

d)

1 (3 b 3

e)

1 (a 3

3b

3)

3b

3)

a

3)

k 1 . k

Si : a k

Log b a 2 ... Log ba 99 ,

Log 2 x Log 2 x 2 4 2

(

b) 4

10 7 .

a) 3 d) 4

c) 2/3 27.

b) 2 e) 2,5

Si : Log a bc

c) 3,5

x n ; Log bac

yn ,

z n , para todo n N .

Log c ab

x ) 64

Calcular el valor de :

c) 16

E

e) 2

1 n

n

1 x

n

1 1

y

n

1 1

z

n

1

Calcular el logaritmo en base 16 del logaritmo de

2 2 en base 8. a) -1/4 d) 1/2 22.

1 2

donde : b

Señalar el producto de las tres raíces de :

a) 4 2 d) 8

Log c b . Log ba 2c 2

4

x(1 Log99x)

b) 3/2 e) 1/27

x

c) 15

Log c a 1

Calcular : Log ba1

Hallar "x", en :

a) 1/3 d) 1/9

21.

c)

0,

de "a" y "b".

e) 100 10

99

20.

25.

Hallar la mayor solución : 1 Log x

19.

-2

b

.

b) 45 e) 9

d) 1 = 10

0

Si : a > b > c > 1, reducir :

a)

(Log x)

b)

9

E

Resolver la ecuación :

a) 10 10

5

c) 6

Co log Antilog x Log Logx

18.

3

9

b) 8 e) 10

a) 0 d) 3

(Log 3a)(Log 3 b) ; a

b

a) 27 d) 25

c) ac

9 Log 8 x 2Log x 8

Si : a

hallar : E

1 Log c a

b) bc e) b

a) 10 d) 12

23.

a) 2n

b) 4 e) -8

d)

c) -4 28.

1 n

9Log 8 a) 9 d) 18

b) 12 e) 13

3

c) 15

n 2

Log

4

Log 4 2

e)

Resolver :

Calcular : 1 3

2 c) n

b) n

y Log ( ) x x y .y x

z

Si : 3Logy = 6Logx = 2Logz, indicar : xyz. a) 4 d) 1

b) 16 e) 0

c) 64

29.

Resolver :

36.

Hallar "x" :

Log 5 (Log 4 x ) Log 5 x

x a) 1 d) 27 30.

xx

Si

Log Log x Log Log x LogLog x Log x

a) 4 d) 1

b) 3 e) 0

37.

b)

3 a a

d)

a

e)

5 a a

... Log 3 9 n

b) 6 e) 9

Log 3 9 28

38.

b) 12 e) 10

Log 2 x

39.

4b

33.

b

d)

b

2 8

b)

ab

2

e)

ab

8

c)

a

8

Log 2 3

6

10

Logx

3

40.

Log 2 6

Log

x

x

El valor de "x" es : a) 1 d) 4 34.

b) 2 e) 5

c) 3 41.

Log 5x

a) 1 d) 4 35.

x

d)

Resolver : x Logx 9 . Indicar la mayor solución : a) 1

b) 1000

d) 100

e) 10

c) 10 9

Log 9

Resolver :

a)

42.

5

c) 72

x 2 1)

0

1; 1

b)

; 1] [1;

c)

; 1

1;

d) [ 1; 1]

x , si :

e)

Ln (x 2 8) Ln (2 x )

e) { 1; 1}

b) 7

5

2Log 5x

c) 3

Log7 5Log7 a) 5

4)

.

b) 2 e) 0

Calcular

8). Log (2 x )

Log (1

Log x (3x Indicar : x

c) 8

b) -9 e) -171/10

Resolver : Log 5 x

1 2

Indicar el producto de todas las soluciones de :

a) -76/5 d) 24

Si :

3

b) 4 e) 1/2

Log (x 2 a)

Log x 2

Log 2 x

Resolver para "x" : Log 8 x ab

c) 24

Indicar una solución de :

a) 1 d) 16

2

a a

c)

Dar la suma de soluciones de la ecuación logarítmica:

c) 7

Log b x a

Loga 2

Log Log(x - 5) + Log2 = Log Log(x + 1)

Hallar el valor de "n", si :

a) 5 d) 8

2 2 a

a) 11 d) 8

Log 3 9 3

1

x ) Log 2a 10

a)

c) 2

Log 3 9 Log 3 9 2

32.

c) 9

10 , calcular :

M

31.

Colog 2 3

b) 243 e) 81 x

Log (

Log 5x

Log 3 (Log 1 (x

Log Log 5x c)

5

Resolver :

5

7

2))

0

2

a)

2; 5

b)

d)

2; 4

e)

2; 5 2 5 1; 2

c)

1; 2

43.

49.

Resolver :

3 (9 x ) 10(3x ) 3

Resolver :

0 Log

a) x [ 1;1]

b) x

c) x R

d) x

1; 1 0; 1

44.

1; 4]

a)

e)

27 x

1

9x

1

3x

1

50.

3x

a)

; 1]

b) [ 2 ;1]

c)

; 2]

d) [ 1; 1]

1 a) [

5

; 2]

3 ; 2]

c) [ 3 ; 2]

b) [ 2 ; 25

x 2

b) [

3 ; 3]

d) [

2 ; 3]

( 2)

a) F ( c) F (

a > 1; 0 < b < 1, resolver el sistema adjunto :

b

a

48.

52.

a) x

1; 2

b) x

c) x

2; 3

d) x

1; 1

Resolver :

a)

1;

d) [2 ;

función cuya regla de Log (

1 x ) 1 x

e)

0 ; 2]

1 a

x) 1 c)

1 ; 2]

2

)

b) F (

a b ) 1 ab

d) F (

2ab ) a b

a b ) 1 ab

Obtener el dominio de la función definida por : f (x )

Ln (Ln (

a)

e ;1

b)

c)

1; 1;1 e

d)

e)

b) [1 ;

b

0; 4

0; 5

Log x (x 2

a b ) 1 ab

a

e) F (

..... (1)

b6x ..... (2)

e) x

11 ; 4] 4

E = F(a) + F(b)

Si :

x2

1

Hallar el equivalente de :

e) x R

a

1; 2]

c)

2x 2 4 x 6 ) 4 x 11

Si se define una correspondencia es :

1; 5

3x 8

(

F (x )

d) x { 1 ; 5}

x

1

[4 ;

x R [ 1 ; 5]

c)

47.

11 4

x2 4x 5 2

a) x [ 1; 5] b) x R

)

11 } 4 11 d) [ 2 ; 4 ] { } 4 11 { } e) [ 2 ; 4

51.

Resolver : 4x x2 5 3

6 4

c) [ 2 ; 4 ] {

e) [ 2 ; 2] 46.

1 2

a) x2 3

x x

Resolver :

Log

Resolver :

x2

3;

e)

e) [ 1; 2] 45.

(

b) [5 ;

0 ;1

d)

Resolver :

x

x x

1 )) 1

; 1 1; 1

53.

Indicar la gráfica de :

F(x)

56.

Hallar la gráfica de :

3 Log 1 x

F (x )

3

| Log 1 (x

2) 3 |

2 y

a)

y

b)

x

3

y

x

1

y

a)

b)

x

0

y

y

y

c) c)

27

27

d)

x

x

0

d)

x

0

x

x

0

y

y

e) 0 1

e)

x

x

57.

La gráfica de cierta función exponencial contiene al 3 2

punto P ( ; 27 ) . Indicar la base y la regla de la función. 54.

Hallar el rango de la función definida por: f (x )

Log 1 (x 2

x

a) b = 3; 3 16 )

b) b

c) b = 9; 9 x

4

d) b

1

a)

; 4]

b)

c) [ 2 ;

e) b = 3; 3 x

; 2]

d) [4 ;

58.

e) [2 ; 55.

Obtener la gráfica de : F (x )

Si se grafica : F:R

R/y

F(x )

H:R

R/y

H(x )

Log 4 x Log 16x ,

a)

x2 1

y

(0; e)

b)

(0; e)

x

y

x

y

c

c)

d

y

d)

(0; e) x

1

a) 1/3 d) 2

e

y

se obtiene :

Calcular :

1 1 2x ;( ) 3 3 1 1 ; ( )2 x 9 9

16

c . d

b) 1/2 e) 3

c) 1

x

e)

(0; e) x

59.

Hallar la gráfica de la función :

y

y

f(x) = |Ln|x||-1 c) y

a)

x -1

y

x

64.

Log 3 x Log 2 x a)

Hallar el campo de definición de : 4

H(x)

c)

Ln Ln[F[F[F(x 3]]]

e)

siendo : F(x) = x - 1. a) x > 0

b) x

e

d) x 61.

c) x

1

e

65.

e) x R

e

1 Log 2 2x

a) b) c) d) e)

0; 2

Log 3 2

b)

0;

Log 2 3 ( ) 3 2

0;

Log 2 3 ( ) 3 3

0; 3

d)

2 Resolver : Ln (e

e2

x

1)

1

0;

x

Log 2 3

Log 3 3 ( ) 2 2

x Z

indicando como respuesta el cardinal de su conjunto solución.

Resolver :

(1,25)

62.

Resolver :

-1

e) N.A. 60.

x

4

d)

x

1

e)

y

-1

x

4

y

x

c)

x

4

b)

1

1

d)

y

(0,64)2

Log

2

a) 2 d) 7

x

66.

x>5 0 3/2 d) x > 4

x

b) x 3/2 e) x < 4

3x

3

0 c) x

1

Claves 01.

e

31. 26.

c

51.

b

02.

c

32. 27.

d

52.

b

03.

a

33. 28.

c

53.

d

04.

d

34. 29.

c

54.

b

05.

e

35. 30.

e

55.

d

06.

c

36. 31.

c

56.

a

07.

d

37. 32.

c

57.

c

08.

a

38. 33.

b

58.

a

09.

c

39. 34.

d

59.

b

10.

e

40. 35.

a

60.

c

11.

a

41. 36. 36 3 6

e

61.

d

12.

c

42. 37. 37 3 7

d

62.

b

13.

b

43. 38.

b

63.

d

14.

a

44. 39.

d

64.

c

15.

c

45. 40.

e

65.

a

16.

c

46. 41.

e

66.

b

17.

c

47. 42.

b

67.

b

18.

c

48. 43.

b

68.

c

19.

c

49. 44.

c

69.

a

20.

b

50. 45.

b

70.

d

21.

a

51. 46.

d

22.

b

52. 47.

b

23.

d

53. 48.

d

24.

a

54. 49.

e

25.

b

55. 50.

b

Capítulo

17

Progresión aritmética (P.A.)

PROGRESIONES 3.

Número de términos (n)

Es aquella sucesión ordenada en la que cada término, excepto el primero, es igual al término anterior aumentado en un valor constante llamado razón de la progresión. Representación de una P.A.

4.

r . a1

2r . ....... . a1 (n 1) r

Términos equidistantes de los extremos

"m" términos

"m" términos

= Inicio de la P.A.

ax

= Separación de términos = Primer término

an

1

a1 . ... . a x . ... . a y . ... . a n

Donde :

. a1

r

( a x y ay )

a1 . a 2 . a 3 . ...... . a n a1 . a1

a n a1

n

5.

= Término n-ésimo

n

= número de términos

r

= razón de la P.A.

ay

a1

an

Término central ( a c ) Siendo "n" impar, la P.A. admite término central.

Clases de P.A.

a1 a n

ac

1.

Si : r > 0, la P.A. es creciente.

2.

Si : r < 0, la P.A. es decreciente.

6.

2

Suma de los "n" primeros términos de una P.A. ( Sn )

Observación : Si, r = 0, se dice que la progresión aritmética es trivial.

6.1.

Sn

6.2.

Sn

a1

2

an

.n

Propiedades de una P.A. Dada la siguiente progresión aritmética, a1 . a 2 . a 3 . ...... . a n

1

an

2a1

(n 1) . r 2

.n

se cumple :

Me di os Ari tm ét icos

1.

Son los términos de una P.A. comprendido entre sus extremos, veamos un ejemplo :

Razón (r)

r 2.

a 2 a1

a 3a 2

....

an

an

Término n-ésimo ( a n ) an

a1 (n 1) r

1

3 . 7 . 11. 15 . 19 . 23 . 27 . 31 Medios aritméti cos

Interpolación de Medios Aritméticos

Clases de P.G.

Consiste en formar una P.A., para lo cual se debe conocer los términos extremos y el número de medios que se quiere interpolar.

1. 2. 3.

Sea la progresión aritmética :

Propiedades de una P.G.

a . .......... .......... ... . b

Si : q > 1, la PG. es creciente. Si : 0 < q < 1, la P.G. es decreciente. Si : q < 0, la P.G. es oscilante.

Dada la siguiente progresión geométrica,

Medios aritméti cos

Por fórmula :

an

a1 (n 1) r

Reemplazando :

b = a + (m+1)r

t1 : t 2 : t 3 : ...... : t n

Razón (q)

b a m 1 q

Fórmula cuyo nombre es razón de interpolación. Progresión armónica (P. H.)

: tn

se cumple : 1.

r

1

2.

Es aquella sucesión ordenada, donde ninguno de sus términos es cero y los recíprocos de los mismos forman una progresión aritmética.

t2

t3

t1

t2

tn

....

tn

1

Término n-ésimo ( t n ) tn

t1 . q n

1

Si la sucesión : a1 ; a 2 ; a 3 ; ...... ; a n

3.

Número de términos (n) Teniendo en cuenta que t n , t y q son positivos. 1

es una progresión armónica, se verifica lo siguiente: 1 . 1 . 1 . ..... . 1 a1 a 2 a 3 an Progresión geométrica (P.G.)

Log (t n ) Log (t1)

n

4.

T é rm i n nos o

Log (q)

e qui d is t a nt e s

ext r em o s

t 1 : ... : t x : ... : t y : ... : t n "m" términos

"m" términos

tx . t y

Representación de una P.G.

t1 : t1q : t 2q 2 : ...... : t1 q n

lo s

( tx y ty )

Es aquella sucesión ordenada en la cual el primer término es diferente de cero y se caracteriza porque cualquier término, excepto el primero, es igual al término anterior multiplicado por un valor constante llamado razón de la progresión.

5.

t1 : t 2 : t 3 : ...... : t n

de

1

t1 . t n

Término Central ( t c ), siendo "n" impar, la P.G. admite término central.

1

tc

t1 . t n

Donde : = Inicio de la progresión. : t1

= Separación de términos.

6.

Suma de los "n" primeros términos de una P.G. (S ) n

= Primer término.

tn

= Término n-ésimo.

n

= Número de términos.

q

= Razón de la P.G.

Sn

t1 .

qn 1 q 1

;q

1

7.

Suma límite ( S

Lím

)

Interpolación de medios geométricos

Para P.G. de infinitos términos, es decir en caso de que n . S Lím 8.

t1

1 q

; 1

q

1

Consiste en formar una P.G., para lo cual se debe conocer los términos extremos y el número de medios que se quiere interpolar. Sea la progresión geométrica : a : .......... .......... .......... . : b

Producto de los "n" primeros términos de una

" m " medios geométri cos

P.G. ( Pn )

Pn

(t1 . t n )c

Med ios ge ométri cos Son los términos de una P.G. comprendidos entre sus extremos, veamos un ejemplo :

1 : 2 : 4 : 8 : 16 : 32 : 64 Medios geométri cos

Por fórmula :

tn

t1 q n

1

Reemplazando :

b

a . qm

1

m 1

b a

q

Fórmula cuyo nombre es razón de interpolación.

EJERCICIOS PROPUESTOS 01.

En la siguiente P.A. :

08.

:(

7). 5 . (

¿Cuál es el valor de " a) 1 d) 7 02.

S

b) -1 e) F.D.

a) 1 d) 2/9 09.

c) 0

Sea la progresión aritmética

a.b.c.d.

Si la suma de sus términos es "n" y la razón es "2n".

a)

3n 2

d) 4n 04.

05.

b) 7 e) 12

b) 1412 e) 1914

Sn

07.

b) 35 e) 41

11.

c) 1705

n (n 8)

13.

c) 37

b) 10 c) 2 e) No se puede determinar

b) 1/9 e) 4/9

b) 4 e) 12

b) 15 e) 22

b) 77 e) 98

y

z)

z 2 (x

y)

3

c) 7/9

c) 6

c) 19

c) 87

Una progresión aritmética está formada del 4 al 55. La suma de los 6 primeros números es 69, de los 6 siguientes es 177 y la suma de los 6 últimos es 285. El segundo y el décimo término de la progresión será : a) 7 y 31 c) 10 y 28 e) 8 y 32

En una progresión aritmética, el término de lugar A es B y el término de lugar B es A. Calcular el valor de (A+B), sabiendo que el segundo término de la progresión es el doble de su sexto término. a) 11 d) 3

(x

x)

De los tres primeros términos de una progresión aritmética, el término intermedio es 15 y el producto de los mismos es 2415. Entonces, el término del décimo primer lugar es : a) 76 d) 97

12.

y 2(z

El quinto término de una P.A. es igual a 19 y el décimo es 39. ¿Cuántos términos hay que tomar para que su suma sea 465? a) 12 d) 32

c) 9

Determinar el décimo quinto término de una P.A., si la suma de los primeros "n" términos está determinada por :

a) 33 d) 39

10.

n2

En la siguiente P.A. : : 10 . ........ . 76 . ........ . 100 el número de términos comprendidos entre 10 y 76 es el triple del número de términos comprendidos entre 76 y 100. ¿Cuál es la suma de los términos de la P.A.? a) 1031 d) 1836

06.

c) 6 n2

b) 12n

En una P.A. la diferencia de dos términos es 96 y la diferencia de sus respectivos lugares es 8. La razón de la progresión es : a) 5 d) 10

z)

Dos cuerpos que se encuentran a la distancia de 153 m uno del otro, se mueven al encuentro mutuo, el primero recorre 10 m/s y el segundo recorrió 3 m el primer segundo, en cada segundo siguiente recorre 5 m más que el segundo anterior. ¿Después de cuántos segundos los cuerpos se encuentran? a) 2 s d) 10

a2 d2 .

e)

x 2 (y

c) 5

Si la suma de los 6 primeros términos de una P.A. es igual a la suma de los 10 primeros términos, calcular la suma de los 16 primeros términos.

Calcular : E

Si : x, y, z; son elementos consecutivos de una progresión aritmética, simplificar :

"?

b) 3 e) 8

a) 1 d) 2 03.

3)

b) 10 y 34 d) 13 y 37

En una progresión aritmética, los elementos de los lugares j, k y (j+k), son tales, que la suma de los primeros es igual al último menos 1. Si la suma de los primeros es "x", hallar la razón de la progresión. x a) ( j k 1)

(x 2) b) ( j k )

(x 1) c) ( j k 1)

(x 2) d) ( j k 1

(x 2) e) ( j k 1) 14.

Determinar el término central de una progresión aritmética de 7 términos, sabiendo que la suma de los términos de lugar impar es 77 y los de lugar par 56. a) 21 d) 19

b) 15 e) 18

c) 25

15.

Indicar las raíces de la ecuación : x 3 px q 0 , si están en progresión aritmética (p 0).

22.

a) -q; 0; q b)

p ; 0;

d)

p

q; p; p

23.

q

Indicar la razón entre "x" e "y", de tal manera que el medio de lugar "r" entre "x" y "2y" sea el mismo que el medio de lugar "r" entre "2x" e "y". Habiendo "n" medios aritméticos interpolados en cada caso. a)

1 n

b)

r

r d) n r

17.

p

p ; 0; p

e) 16.

a) 1 d) 4

q ; 0; q

c)

1

n r 1

n

r e) n 1

c)

n

18.

19.

r

25.

a) 8

b)

d) 16

e) 16 2

S 5n

S4 n

2

b) 3 e) 11

c) 8 2

26.

b) 648 e) 625

27.

c) 3/4

c) 729

b) 234 e) 5/9

c) 243

La suma de los términos de una progresión geométrica decreciente de infinitos términos es "m" veces la suma de sus "n" primeros términos. Hallar la razón de la progresión geométrica.

a)

m 1 m

c)

m 1 m

e)

m 1 n 1

1/ n

b)

m 1 m 1

d)

1 mn

1/ m

1/ m

1/ m

1/ n

El primer término de una sucesión geométrica es igual a x-2, el tercer término es igual a x+6, y la media aritmética de los términos primero y tercero es al segundo término de la sucesión como 5 es a 3. Hallar el sexto término de la sucesión y dar como respuesta la suma de sus cifras. a) 6 d) 24

b) 6

Si se interpolan 5 medios geométricos entre 8 y 5832. ¿Cuál es el quinto término de la progresión total? a) 1944 d) 2916

La diferencia del tercer término con el sexto término de una P.G. es 26, si su cociente es 27. ¿Cuál es el primer término de la P.G.?

c) 9

Hallar la razón de una P.A. cuyo primer término sea la unidad, tal que los términos de lugares : 2, 10 y 34 formen una P.G. b) 1/3 e) 2/3

b) -1/8 c) -1/16 e) No se puede determinar

.

La suma de 3 números positivos en P.A. es 18. Si a estos números, se les suma 2, 4 y 11, respectivamente; los nuevos números forman una P.G. ¿Cuál es el mayor de los números primitivos?

a) 2/5 d) 5/7 21.

b) 6 e) 15

S9 n

c) 3

La suma de los 3 primeros términos de una progresión geométrica es igual a 6 y la suma del segundo, tercero y cuarto términos es igual a -3. Calcular el décimo término.

a) 245 d) 1/9

Hallar un número tal que al restarle 8, multiplicarlo por 2 y por 4 se obtienen tres resultados que se encuentran en progresión geométrica.

a) 1 d) 9 20.

24.

1 r 1

términos de una P.A., calcular el valor de :

b) 2 e) 7

a) -1/2 d) -1/64

Asumiendo que S kn es la suma de las "kn" primeros

a) 3 d) 12

El primer término de una progresión geométrica es igual a (x - 2), el tercer término es igual a (x + 6), y la media aritmética de los términos primero y tercero es al segundo como 5 es a 3. Determinar el valor de "x".

b) 9 e) 23

c) 18

Tres números enteros están en P.G. Si al último término se le resta 32, se forma una P.A.; pero si al segundo término de esta P.A., se le resta 4 se forma una nueva P.G. Según ello, señale la suma de los tres números enteros. a) 50 d) 72

b) 12 e) 60

c) 62

28.

En una P.G., el primer término es "a", la razón "q"; S 4 la suma de las cuartas potencias de los "n" términos de la progresión. Señale el equivalente de :

S 4 (q

4

1)

35.

2 : .......... ..... : a : .......... .......128 4 8 " m " medios

29.

a) 16 d) 4

b) Primer término d) Segundo término

36.

respectivamente, calcular : a q r . br p . c p q .

30.

b) 1 e) abc

31.

b) 200 e) 15

38.

c) 60

Calcular el límite de la suma : 3 4

1

a) 9/4 d) 6

7 16

15 64

b) 8/3 e) 4

31 256

39.

...

c) 7/2

Calcular el límite de la suma : S

2 3

a) 9/2 d) 9/8 34.

c) ab

Se interpolan cuatro medios geométricos entre 160 y 5. Hallar la suma de los dos últimos términos de la progresión geométrica formada.

S

33.

37.

b) 3ab e) 6K

a) 240 d) 35 32.

c) 2

En una P.G. no oscilante el término de lugar "6a" es 3 K 2 y el término de lugar "4b" es 12, hallar el término de lugar "3a+2b". a) 2abK d) 3K

3

2

3

2

3

32

33

34

35

36

b) 2/9 e) 7/4

6, a4

48

b) a1

8 , a4

64

c)

7, a4

56

a1

d) a 1

10 , a 4

e) a1

8 , a4

270 216

2.

c) 32

En una serie geométrica de números naturales de

no .

a) 4 d) 8

b) 6 e) 9

c) n2

b)

d) n + 1

e) 2n - 3

3

Dados los términos : a m n A y am n B de una progresión geométrica {a n } . Hallar : a m . a)

A

d)

Am

b)

AB e) AB

c)

Bm

En una P.G. de términos positivos, se observa que cada término es igual a la suma de los dos términos siguientes. ¿Cuál es la razón de la progresión?

1 2

b) 5 1 2

e)

2 7

c)

1

2 5

5 2

Se deja caer una pelota desde una altura de 90 m, en cada rebote la pelota se eleva 1/3 de la altura, de la cual cayó la última vez. ¿Qué distancia recorre la pelota hasta quedar en reposo? a) 120 m d) 150

41.

n 2

a) n

d) 40.

c) 7

En una P.A. de "n" términos, la suma de los (n-1) primeros términos es "n" y la suma de los (n-1) últimos términos es " n2 ". Hallar la razón de dicha progresión.

...

c) 80/81

o

Calcular : a o

a)

La suma de los medios geométricos de una serie de 4 términos es 42 y su diferencia 14. Si la razón es mayor que uno, al calcular el primer y cuarto término se obtiene : a) a1

b) 8 e) 64

4

razón r > 1, r N, la suma de los no primeros términos es 31, ( no >3). Si a es el primer término de la serie.

Si los términos de lugar p; q y r de una P.G. son a, b, c,

a) 1/2 d) 3

" 2m " medios

Hallar "a", si la razón de la progresión es

1/ 2

a 2 (q 4 n 1) a) 1 c) n e) Término central

En la P.G. :

b) 180 e) 140

c) 90

Si : 2 + 14 + 26 + 38 + ..... + x = 816, entonces, el valor de "x" es : a) 110 d) 146

b) 122 e) 158

c) 134

42.

A lo largo de un camino había un número impar de piedras a 10 m una de otra. Se quiso juntar éstas en un lugar donde se encontraba la piedra central. El hombre encargado podía llevar una sola piedra. Empezó por uno de los extremos y los trasladaba sucesivamente. Al recoger todas las piedras, el hombre caminó 3 km. ¿Cuántas piedras había en el camino? a) 17 d) 13

43.

b) 41 e) 25

b) 4/3 e) 18

Los números reales a1 , a 2 , .... a n , son positivos y están en progresión aritmética de razón "r". Si :

1

Tn

45.

an

an

a)

48.

a1

46.

Sn

Tn

Donde : S n : suma de los "n" primeros términos. Tn : término general. Si : "n" es impar, indicar el término central. a) n + 1 d) n + 4

b) n + 2 e) n + 5

c) n + 3

n 1 an a1

d)

1 n a1 an

Si, en una P.G. de cuatro términos se cumple que la suma del primero con el tercero es 117, además, la suma del cuarto con el segundo es 78. Hallar la diferencia entre el cuarto y segundo término. b) -54 e) -45

c) -81

En una P.A., el ultimo término es "u", la razón es "r" y sus valores se obtienen al resolver el siguiente sistema: 2

335 2

ur u r 70 . Si : r > 0. Si la suma de términos es 16, hallar el número de términos.

c) 600

(n 3)(n 1)

b)

a1

a) -30 d) -36

B

De una progresión aritmética, se sabe que:

a4

n an

u3 r 3

b) 540 e) 1800

a3

an

n 1 an a1

c)

49.

a) 200 d) 900

1

n 1

c) 6

A

a2

de "n", a1 y a n , es :

c) 1/2

La figura representa a una persona en su "skate" que va a recorrer una rampa semicircular de longitud 180 m de A hasta B, en cada viaje (de un extremo a otro), sólo recorre el 70% de lo recorrido en el viaje anterior. Calcular la distancia total que "barrió" con el skate hasta detenerse en el centro de la rampa.

a2

entonces, la expresión simplificada de Tn en términos

e) b) 4 e) 10

a2

1

1

...

Entre 2 y 162, entre 3 y 19683 se han interpolado el mismo número de medios geométricos. Calcular la diferencia de las razones, sabiendo que la razón de la primera es 1/3 de la razón de la segunda. a) 2 d) 8

1

a1

c) 29

Dados los números, x, y, z, w; se observa que los tres primeros están en P.A. y los tres últimos en P.G. siendo la suma de los extremos 14 y la suma de los medios 12. Hallar "x". a) 3/4 d) 12

44.

47.

a) 9 d) 12 50.

b) 7 e) 5

c) 4

Una persona nació en la segunda mitad del siglo pasado; un año que goza de la propiedad de que las cuatro cifras son tales que las tres diferencias formadas restando la primera cifra de la segunda, la segunda de la tercera y la tercera de la cuarta estén en P.G. ¿Cuántos años cumplirá el 2006? a) 49 d) 57

b) 54 e) 51

c) 56

51.

Dada la siguiente progresión geométrica,

56.

P. G. a:b:c . Calcular :

1 a3

a 2 b 2c 2

E

a) a + b + c c) a

2

e) a 3 52.

b

2

c

2

1 b3

1 c3 b4

c4

4

3

2

b

c

b3 c 3

a) 9n d) 9 57.

En una P.G. de seis términos decrecientes, se cumple que la suma de los términos extremos es 5 a y el producto de los medios es a 2 .

b) 3n e) 3

c)

n

d)

5

3

Del gráfico, hallar la suma de todas las longitudes de las perpendiculares que se proyectan ilimitadamente a partir del punto "P".

Sen

40 . 41 P

5 2

3

b;

3

Calcular la razón. a) 5

c

a c y b c términos, formando tres progresiones b a geométricas diferentes. Hallar el producto de las tres razones geométricas obtenidas. Si : a + b + c = n.

b) a 4 d) a

Entre 2 y 18 se interpolan, en forma separada a

5

b)

5

d)

5

3

5

48

2 3

5

e) Hay 2 respuestas 53.

Sean : t12 y S1 el primer término y la suma límite de una P.G. decreciente. Si t1 es el primer término de una nueva P.G. en la cual la razón es la mitad de la razón de la anterior P.G. El equivalente en la razón entre las sumas límites de la primera y la segunda P.G. expresada en términos de t1 y S es : 1

a) t 1 d) 54.

2

S1)

b) S 1 e)

t 2 S1 c) 1 2t1

t1

(t1 S1)

x 2 6x 8 0 x 2 6x 8 0 x 2 6x 8 0

b) x 2 6x 8 0 d) x 2 6 x 10 0

Se tiene 2 progresiones, una aritmética y otra geométrica, cuyos primeros términos son iguales e igual a la razón común, sabiendo que la suma de los 8 primeros términos de la progresión aritmética es igual a la suma de los infinitos términos de la progresión geométrica. Hallar el noveno término de la progresión aritmética. a) 35/41 d) 35/4

58.

b) 35/26 e) 35/36

c) 36/35

b) 20 e) 60

c) 30

Dada la progresión aritmética creciente : a1 . a 2 . a 3 .... a n sabiendo, que la suma de sus términos es "S" y que la suma de sus cuadrados es S 2 , su razón "r", será : 1

a)

2

Hallar la ecuación de segundo grado, cuyas raíces y el producto de ellas están en progresión geométrica creciente, además, el producto de sus raíces, la suma de ellas y la mayor de las raíces están en progresión aritmética. a) c) e)

55.

(t1

S1

a) 10 d) 50

b)

c)

d)

e)

n S12

S2

n2(n 2 1) 12 (n S12 2

n (n

2

2 (n S12

S2 ) 1) S2)

n2 (n2 1)

24 (n S12

S2)

n2(n 2 1)

6 (n S12

S2)

n2 (n2 1)

59.

Dadas las relaciones :

60.

Si la expresión :

Log a x . Log b y . Log c z

a (b c ) x 2

x:y:z Calcular el valor que debe tomar el logaritmo de "z" en base "x". a) Log a c c)

Log a (a / b)

e)

1 Log a b

Log c (c / b)

Log c b 1

b)

d)

Log (a / b) Log(b / c) Log a b

1 Log a

b(c a)xy c(a b)y 2

es un cuadrado perfecto, entonces, a, b, c; se encuentran formando : a) b) c) d) e)

Progresión aritmética. Progresión geométrica. Progresión armónica. Progresión hipergeométrica. Progresión aritmética de orden superior.

Claves 01.

d

31.

a

02.

c

32.

b

03.

a

33.

d

04.

e

34.

c

05.

c

35.

b

06.

c

36.

c

07.

b

37.

a

08.

d

38.

b

09.

c

39.

d

10.

b

40.

b

11.

c

41.

c

12.

a

42.

e

13.

b

43.

d

14.

d

44.

c

15.

c

45.

c

16.

e

46.

b

17.

c

47.

c

18.

d

48.

a

19.

d

49.

c

20.

b

50.

a

21.

b

51.

e

22.

c

52.

b

23.

d

53.

c

24.

c

54.

b

25.

a

55.

e

26.

b

56.

d

27.

c

57.

e

28.

b

58.

b

29.

b

59.

e

30.

e

60.

c

Í N D I C E

View more...

Comments

Copyright ©2017 KUPDF Inc.
SUPPORT KUPDF